Family Medicine - fmCases

अब Quizwiz के साथ अपने होमवर्क और परीक्षाओं को एस करें!

What are other hormonal risk factors for BC

Advanced age at first pregnancy Exposure to diethylstilbestrol Hormone therapy

What are the common signs and symptoms of hyperthyroidism? Select all that apply. A. Heat intolerance B. Tachycardia/Palpitations C. Constipation D. Fatigue E. Heavy periods F. Weight loss G. Tremor H. Sweating I. Depression J. Hyperreflexia

All except constipation, heavy periods

What are lung cancer screening recs

As of 2013, the USPSTF recommends annual screening with a low-dose CT scan to screen for lung cancer in patients ages 55 to 80 who have smoked for 30-plus years. To be considered, the patient should also be currently smoking or have quit within the prior 15 years.

Cervical cancer screening recommendations

At age 21: cervical cancer screening should begin. Between ages 21 and 29: screening should be performed every three years. Between ages 30 and 65: screening can be done every five years if co-tested for HPV (preferred) or every three years with cytology alone (acceptable).

For RUQ pain, What is the most practical imaging study to order at this time, considering cost, availability, and sensitivity? Choose the single best answer. A. Abdominal upright and flat plate x-ray B. Abdominal ultrasound C. Abdominal CT Scan with contrast D. Abdominal MRI

B In general, real-time abdominal ultrasonography is the preferred imaging study to evaluate the right upper quadrant because it is inexpensive, noninvasive, and widely available. It provides a good evaluation of the liver and other viscera such as the gallbladder, and it is accurate in the detection of gallstones and dilation of the biliary tree.

Which of the following would be considered treatment(s) of choice in this clinical scenario? Select all that apply. A. Amitriptyline - a tricyclic antidepressant B. Sertraline - a selective serotonergic reuptake inhibitor (SSRI) C. Cognitive-behavioral therapy D. Electroconvulsive therapy (ECT) E. Exercise

B, C, E

What is official USPSTF magazine for women 50-74

Biennial screening mammography for women age 50 to 74 years (Grade B recommendation)

Which is/are most effective for ankle injury? Choose the single best answer. A. Tape B. Elastic wrap C. Semi-rigid support

C

Which of the following diagnostic tests is the best initial test with high predictive value for determining whether your patient has cellulitis or DVT? Choose the single best answer. A. Complete blood count B. Culture and sensitivity of the ulcer C. Venous Doppler of the lower extremity D. D-dimer E. MRI of the affected extremity

C

What are the 3 C's of addiction

Compulsion to use Lack of Control Continued use despite adverse Consequences

Of the following risk factors which causes the most deaths in the U.S.? Choose the single best answer. A. Hypertension B. Diabetes C. Obesity D. Smoking

D

Seeing the group going into the exam room, Dr. Nayar asks you, "How do you approach a patient when there are additional people present in the examining room?" Choose the single best answer. A. Greet everyone in the room and begin obtaining the history. B. Greet everyone in the room and say, "Is everyone OK with this?" before beginning the interview. C. Greet the patient only and begin the discussion. D. Greet the patient, identify the other individuals, and ensure that the patient is comfortable with their presence during the discussion.

D

What is effect of menopause on breast cancer

Early menopause decreases BC risk - menopause after 55 increases risk

What does Convex ST segment elevation suggest

Suggests acute myocardial injury

You are seeing a 19-year-old woman who suffered a knee injury playing soccer one day ago. The injury involved a sudden deceleration in which she planted her right foot in front of her while running, whereupon another athlete fell against her shin. She felt a pop and sudden pain. She had to be helped off the field and her knee swelled immediately. Today, she reports that she has considerable right knee pain with bearing weight and that her knee feels unstable at times. Her past medical history is unremarkable, and she takes no medications. On exam, her vital signs are perfectly normal. You conduct a knee exam Which of the following exam maneuvers is most likely to be abnormal in this patient? A. Lachman test B. Laxity to varus stress C. Posterior drawer test D. McMurray test E. Laxity to valgus stress

The correct answer is A. This patient most likely has experienced a tear of her ACL, which can often happen with a sudden deceleration injury. Patients typically describe a "popping" sensation when the injury occurs. Common exam findings for a patient with an ACL tear include a positive anterior drawer test and a positive Lochman's test. Laxity to varus (outward) stress would suggest a tear of the lateral collateral ligament, which is a relatively uncommon sports injury. Laxity to valgus (inward) stress would indicate a possible tear of the medial collateral ligament. This can sometimes co-occur with an ACL tear and is a possible finding in this patient, though it is not the most likely finding. McMurray test is positive in the setting of meniscal tears.

What benzo agents are preferred for sleeping in elderly

zolpidem (Ambien) eszopiclone (Lunesta)

What does the AUDIT score consist of

"How often do you have a drink containing alcohol?" "How many standard drinks containing alcohol do you have in a typical day?" "How often do you have six or more drinks on one occasion?"

When give zoster vaccine

50

Sammy is a healthy male child brought into your office by his mother for a well-child examination. As part of your evaluation you assess his developmental milestones. He is able to run, make a tower of 2 cubes, has 6 words in his vocabulary, and can remove his own garments. What would you estimate Sammy's age to be based upon his developmental milestones? A. 12 months B. 15 months C. 18 months D. 30 months E. 36 months

> The correcgt answer is C. A. At age one year, gross motor skills include pulling to stand, standing alone, and perhaps first steps. Fine motor skills including putting a block in a cup and banging 2 cubes held in hands. At this age a child should be able to imitate vocalizations/sounds and babble. The majority of children this age will know 1 or 2 words in addition to "mama" and "dada." Social-emotional milestones at age one year are waving bye-bye and playing pat-a-cake. Running, building towers of blocks, removing clothing, and a 6-word vocabulary are more advanced skills than a 12-month-old would be expected to have. B. At 15 months of age, a child should be able to stoop and recover and walk well, put a block in a cup, have a vocabulary of a few words, wave bye-bye, and drink from a cup. Running, building towers of blocks, removing clothing, and a 6-word vocabulary are more advanced skills than a 15-month-old would be expected to have. C. At 18 months, a child should be able to walk backward, and 50-90% of children can run at this stage. An 18-month-old should be able to scribble, build a tower of 2 cubes, have 3-6 words in her or his vocabulary, and be able to help in the house and remove garments. D. At 2 ½ years of age, kids can jump up and throw a ball overhand. They can build a tower of 6-8 cubes, point to 6 body parts, name 1 picture, put on clothing, and wash and dry their hands. Sammy is only able to build a tower of 2 cubes, can remove his clothing but does not yet put clothing on, and his vocabulary is limited to 6 words-leading us to believe he is not 2 ½ years old. E. At age 3, children can balance on each foot for 1 second, wiggle their thumbs, name 4 pictures, name 1 color, name a friend, and brush their teeth with help. Sammy's vocabulary is only 6 words, he is not able to name a friend, he is only able to stack 2 cubes, and he has just starting running, but is unable to balance on each foot for 1 second.

A 9-month old baby boy comes to the clinic for a well-child visit. The child is at the 50th percentile for weight, length, and head circumference. He is reaching all developmental milestones appropriately. The mother has no concerns at this visit. The child has previously received the following vaccines: 3 doses of DTaP, 3 doses of Hib, 2 doses of HepB, 3 doses of RotaV, 2 doses of IPV and 3 doses of PCV13, and no influenza vaccines. Which vaccines should the child receive at today's visit? A. Influenza, Hep B, IPV, DTaP B. Influenza, IPV C. Influenza, Hep B, IPV D. Hep B, DTaP, IPV E. Hep B, IPV, and MMR SUBMIT

> The correct answer ic C. A. Influenza, Hep B, IPV, DTaP is incorrect. All three doses of DTaP have been given. B. Influenza, IPV is incorrect. The patient needs the third Hep B shot. C. Influenza, Hep B, IPV is correct. The patient needs a third Hep B, a third IPV, and a yearly flu shot starting at 6 months of age. D. Hep B, DTaP, IPV is incorrect. All three doses of DTaP have been given and the patient now needs a yearly flu shot starting at 6 months of age. E. Hep B, IPV, and MMR is incorrect. The patient also needs a yearly flu shot starting at 6 months of age and MMR is not given before 12 months of age.

An 8-year-old obese male with no past medical history is found to have a blood pressure of 125/90 mmHg on routine evaluation during an office visit for well-child care. Review of symptoms is negative. A physical exam and screening bloodwork are performed. Both are normal, with the exception of his blood pressure and obesity. What is the most likely diagnosis? A. Primary hypertension B. Renal artery stenosis C. Coarctation of the aorta D. Pheochromocytoma E. Hyperthyroidism

> The correct answer is A. A. The sole physical finding is hypertension. Given the mild hypertension and the patient's age, symptoms are unlikely to be present. Other etiologies should be ruled out, but review of symptoms, physical examination, and laboratory studies do not suggest other etiologies. B. Patients with renal artery stenosis are largely asymptomatic, but as the kidney function deteriorates, they may experience edema and dyspnea. On physical examination, patients will have hypertension (as a consequence of elevated angiotensin II and aldosterone) and possibly abdominal bruits. For laboratory studies, the patient will have elevated serum angiotensin II and serum aldosterone, as the kidneys attempt to compensate for a perceived decrease in glomerular filtration rate. C. Assuming sufficient severity of the coarctation, symptoms include chest pain, cold lower extremities, dizziness, syncope, exercise intolerance, failure to thrive, poor growth, headache, and dyspnea, among others. Distal to the coarctation, pulses will be diminished. Blood pressure will be lower in the lower extremities compared to the upper extremities. Murmurs may also be heart on auscultation. Coarctation of the aorta may also be associated with other congenital heart defects (bicuspid aortic valve in 50% of patients) and chromosomal abnormalities (Turner syndrome). D. Symptoms of pheochromocytoma include headache, diaphoresis, palpitations, tremor, nausea, weakness, anxiety, nervousness, irritability, and weight loss, among other symptoms. On physical examination, patients typically present with tachycardia and severely elevated blood pressure. For laboratory studies, the patient will have elevated urinary VMA, urinary HVA, urinary metanephrines, and serum glucose. Pheochromocytoma may be associated with various syndromes, such as multiple endocrine neoplasia and Von Hippel-Lindau disease. E. Symptoms include heat intolerance, frequent bowel movements, increased appetite, diaphoresis, nervousness, restlessness, weight loss, tremor, hair loss, and palpitations, among others. On examination, patients typically present with tachycardia, hypertension, hyperreflexia, and goiter. The patient will have elevated T3, T4, and serum glucose, and TSH may be depressed or elevated depending on the etiology.

Mark is a 5-month-old male who is brought to the urgent care clinic with a three-day history of rhinorrhea and non-productive cough. When he was born he was large for gestational age, and his exam then was notable for macrocephaly, macroglossia, and hypospadias. On physical exam now his vitals signs are stable. He has copious nasal discharge, but his lungs are clear to auscultation. On abdominal exam, you palpate an abdominal mass on the right side just below the subcostal margin. It is 7 cm in diameter and does not cross the midline. The abdomen is soft and non-tender with active bowel sounds. What is the most likely cause of his mass? A. Wilms' tumor B. Teratoma C. Renal cell carcinoma D. Hepatoblastoma SUBMIT

> The correct answer is A. A. Wilms' tumor is commonly associated with Beckwith-Wiedemann syndrome, a genetic overgrowth syndrome. Other features that may be seen in children with this syndrome include omphalocele, hemihypertrophy, hypoglycemia, large for gestational age, and other dysmorphic features. B. Teratomas are congenital tumors that are present at birth. These benign tumors that are often identified incidentally, or may become symptomatic due to mass effect of the lesion within the abdominal cavity. The aggressiveness of the tumor depends on the degree of differentiation. C. Renal cell carcinomas are much more common in adulthood. Risk factors include cigarette smoking and obesity. D. While children with Beckwith-Wiedemann syndrome can have hepatoblastoma (in addition to other types of tumors), this is not the most common tumor in this genetic condition. Note that hepatoblastoma may also be associated with familial adenomatous polyposis.

An otherwise healthy 58-year-old female presents in your office with a cough, sore throat and fevers for the past five days. Which of the following clinical details would lead you to treat with an antibacterial agent? A. Dullness to percussion and crackles on lung exam consistent with community-acquired pneumonia B. Myalgias consistent with influenza C. Non-erythematous tympanic membrane with clear effusion D. Purulent discharge and sinus tenderness consistent with acute sinusitis E. Wheezing and productive cough consistent with acute bronchitis

> The correct answer is A. The differential diagnosis for a patient presenting with cough, sore throat and fevers includes the common cold, influenza, acute sinusitis, acute bronchitis, and community acquired pneumonia. Option A describes otitis media with effusion (OME), a common and painless finding in adults and children in the setting of a cold or its aftermath. Unlike some cases of acute otitis media (which features a bulging, red, and painful tympanic membrane), OME is not treated with antibiotics. Influenza (B) is treated with antiviral medications, such as oseltamivir. Ninety percent of cases of acute bronchitis (E) are viral, and Choosing Wisely recommends against the use of antibiotics for this diagnosis because symptoms have to be present for longer than seven days without resolution to conservative management. The latter is true also for most episodes of acute sinusitis (D). In this patient with only five days of symptoms, antibiotics would not be appropriate for the diagnosis of sinusitis. Community-acquired pneumonia (A) is always treated with antibiotics.

Of the following, which best reflects the caloric requirement of most healthy term babies in the first 1 to 2 months of life? Choose the single best answer. The best option is indicated below. Your selections are indicated by the shaded boxes. A. 50 kcal/kg/day B. 100 kcal/kg/day C. 150 kcal/kg/day D. 200 kcal/kg/day

> The correct answer is B.

A 10-month-old asymptomatic infant presents with a RUQ mass. Work-up reveals a normocytic anemia, elevated urinary HVA/VMA, and a large heterogeneous mass with scant calcifications on CT. A bone marrow biopsy is performed. Which of the following histologic findings on bone marrow biopsy is most consistent with your suspected diagnosis? A. Sheets of lymphocytes with interspersed macrophages B. Small round blue cells with dense nuclei forming small rosettes C. Hypersegmented neutrophils D. Stacks of RBCs E. Enlarged cells with intranuclear inclusion bodies

> The correct answer is B. A. This is incorrect, as sheets of lymphocytes with interspersed macrophages are associated with Burkitt lymphoma. B. This is the correct response. In addition to neuroblastoma, other tumors associated with small blue cells include Ewing's sarcoma and medulloblastoma, both of which tumors are seen in children. C. Hypersegmented neutrophils are characteristic of megaloblastic anemia, a condition associated with a vitamin B12 and/or folate deficiency, not malignancy. D. Stacks of RBCs suggest rouleaux formation, a phenomenon seen in multiple myeloma, a condition not seen in young infants. E. This describes the classic "owl's eyes" seen in CMV and other viral infections.

You are seeing a 60-year-old male brought into your office because of "dizzy spells where he almost passes out" for a week. Your records indicate a history of back pain and diabetes. Upon further questioning, he says he sometimes feels like he is going to pass out and gets short of breath after walking about half a block. He's never felt this way before. He denies having a spinning sensation when he is dizzy. His back has also been really bothering him for the past several weeks so he has been taking ibuprofen "all the time." His temperature is 98.6 Fahrenheit; blood pressure is 105/65 mmHg; heart rate is 100 beats/minute; and respiratory rate is 18 breaths/minute. On physical exam, you note no nystagmus, but he does have conjunctival pallor. His lungs are clear to auscultation, and his cardiac exam reveals mild tachycardia with a regular rate and no murmurs. His abdomen is mildly tender and non-distended. As you think through your differential diagnosis and tests to order, which of the following best describes the likely etiology of his symptoms? A. Acute labrynthitis B. Anemia C. Aortic stenosis D. Myocardial infarction E. Thyroid storm

> The correct answer is B. This patient is likely bleeding from his stomach from taking too many NSAIDs for his back pain resulting in a gastritis or ulcer. The resulting blood loss and anemia leads to decreased preload and episodes of presyncope when his cardiac output is unable to keep pace with the increase in metabolic needs. As a result, his brain is inadequately perfused. Though his age and diabetes put him at risk for a myocardial infarction (D), it is unlikely to present in such a subacute way and his physical exam findings do not point in that direction. Choices A, C and E would likely be present with different physical exam findings: Acute labrynthitis (A) is a cause of peripheral vertigo. Patients typically report acute disequilibrium with a prominent sensation of the room spinning (vertigo), exacerbated by head movement. Nystagmus with eye movement that improves with fixed gaze is typical for a peripheral source of vertigo such as this. Aortic stenosis (C) can be appreciated on physical exam as a systolic ejection murmur, which this patient does not have. Thyroid storm (E), a life-threatening condition in someone with hyperthyroidism, typically presents with elevated blood pressure, tachycardia and fever. Conjunctival pallor would not be expected in this condition.

By what ages should an infant double and triple his or her birth weight? Choose the single best answer. The best option is indicated below. Your selections are indicated by the shaded boxes. A. Double by 3 months, triple by 6 months B. Double by 4 months, triple by 6 months C. Double by 5 months, triple by 12 months D. Double by 9 months, triple by 15 months

> The correct answer is C.

Which of the following positions is most appropriate for Asia at this age? Choose the single best answer. The best option is indicated below. Your selections are indicated by the shaded boxes. A. Front seat, facing the rear B. Front seat, facing the front C. Back seat, facing the rear D. Back seat, facing the front

> The correct answer is C.

You are seeing a 36-month-old boy for his well-child visit. His parents are anxious about ensuring that his development is appropriate. He passed a hearing screen at birth and, other than a few colds, has been generally healthy. He has never been hospitalized or had any serious illness. He is able to run well, walk up stairs, and walk slowly down stairs. He uses more words than the parents are able to count, but can use them only in short, two or three-word sentences. His speech is understandable. He can draw a circle, but not a cross. Neurologic examination shows normal cranial nerves, normal sensitivity, normal motor reflexes, and no Babinski sign. Which of the following is the most appropriate next step in the management of this patient? A. Perform a brain-stem auditory evoked potential hearing screen B. Perform a screening exam for autism C. Reassure the parents that the boy's development appears normal D. Refer the child to a developmental specialist for comprehensive evaluation E. Refer the child to a specialist for evaluation of his delayed motor development

> The correct answer is C. A. A brain-stem auditory evoked potential hearing test (BAER) may be indicated in infants who fail to meet language milestones if they cannot cooperate with other more comprehensive testing. A 36-month-old should be able to cooperate with behavioral audiometry, so a BAER is not indicated. In addition, this child has no evidence of language delay and does not require referral at this point. B. Autism is an increasingly diagnosed cause of developmental delay, but this child is not delayed and no mention is given of any autistic features, such as a lack of symbolic play, repetitive movements, or poor sociability. C. The developmental milestones mentioned in the vignette are within the range of normal for a 36-month-old child. In the absence of any other evidence of significant impairment, there is no indication for referral at this point. D. If there are reasons for concern on developmental screening tests, a referral may be indicated. However, the developmental milestones mentioned in the vignette are within the range of normal for a 36-month-old child. E. This child's motor milestones are not delayed, and no referral is indicated.

An 8-year-old female presents to your clinic for follow-up after being hospitalized for status asthmaticus. She has just completed a 10-day course of systemic steroids. Given her history of moderate persistent asthma, her outpatient regimen includes daily use of inhaled fluticasone / salmeterol. She was also diagnosed with ADHD one year ago and was started on Adderall XR daily. Her BMI today is at the 83rd percentile for her age, and her blood pressure is at the 98th percentile for her age. What is the most likely cause of her stage I hypertension? A. Obesity B. The blood pressure cuff is too big C. Medications D. Renal insufficiency E. Neurofibromatosis 1 SUBMIT

> The correct answer is C. A. While obesity is a risk factor for hypertension, Jane is not overweight. A child is considered overweight when his or her BMI is between the 85th and 95th percentile for age. Obesity is considered > 95th percentile. B. Blood pressure would be falsely decreased if the cuff was too big, and, inversely, falsely elevated if the BP cuff was too small. C. Both steroids and CNS stimulants can cause increases in blood pressure, especially when used in combination. Steroids increase blood pressure by mimicking endogenous cortisol and the sympathetic fight or flight response. Stimulants mimic norepinephrine, stimulating alpha and beta adrenergic receptors, causing an overall increase in blood pressure. D. Renal insufficiency can be a cause of secondary hypertension, but is unlikely in this patient. Risk factors that warrant investigation of renal causes for hypertension include recurrent UTIs, umbilical arterial/venous lines placed while a child was in the ICU, and a family history of renal disease. E. NF-1 can be associated with hypertension as a result of vascular malformations that affect blood supply to the kidneys; however, this child does not present with any signs of NF-1 (café au lait macules, neurofibromas, optic gliomas, lisch nodules).

Which of the following physical findings indicate risk factors for coronary heart disease? Select all that apply. The best options are indicated below. Your selections are indicated by the shaded boxes. A. Xanthelasma B. Carotid bruits C. Roth's spots D. Diminished peripheral pulses E. Buffalo hump F. Striae G. Increased waist circumference

> The correct answers are A, B, D, G

Which of the following vaccines will you give Asia today at her 2-month visit? Select all that apply. A. DTaP (diphtheria/tetanus/acellular pertussis) B. MMR (measles/mumps/rubella) C. HepB (hepatitis B) D. Hib (Haemophilus influenzae) E. IPV (inactivated polio vaccine) F. PCV13 (pneumococcal conjugate vaccine - 13 serotypes) G. RotaV (pentavalent rotavirus vaccine) H. HepA (hepatitis A vaccine)

> The correct answers are A, C, D, E, F, G. The appropriate immunizations for Asia today are her first doses of: DTaP (A) Hib (D) IPV (E) PCV13 (F) and RotaV (G)

Which of the following are signs/symptoms of limb-threatening injury to look for when evaluating Christina? Select all that apply. A. Pain B. Pallor C. Pulselessness D. Paresthesia E. Perishing cold F. Paralysis

ALl of the above - compartment syndrome

You are seeing a 54-year-old female with a past medical history of kidney stones who presents with a chief complaint of "I have a terrible summer cold." She reports three days of low grade fevers (peak of 100.0 Fahrenheit), cough, sore throat, headaches, and nasal congestion. She denies myalgias. Her temperature today is 99.2 Fahrenheit, respirations 14/minute, pulse 78 beats/minute, and blood pressure of 128/74 mmHg. Her head and neck exam reveals normal tympanic membranes, mildly congested nasal turbinates with thin mucous, erythema of the tonsillar pillars and soft palate without tonsillar enlargement or exudate, and mild anterior cervical lymphadenopathy. Her lungs are clear to auscultation. Which of the following options would be the most appropriate therapeutic option for this patient? A. Echinacea supplementation B. Oseltamivir (Tamiflu) 75 mg twice daily for five days C. Pseudoephedrine (Sudafed) as needed for nasal congestion D. Vitamin C supplementation E. Zinc supplementation

> The correct answer is C. This patient has the symptoms and physical exam consistent with the acute infectious rhinosinusitis (ie. the common cold). Treatments that have been demonstrated to help symptoms of the common cold include decongestants such as pseudoephedrine (C), nasal ipratropium spray, and (with weaker evidence supporting it) vitamin C. Vitamin C (D) would be inappropriate in this patient because it can cause kidney stones, which she has had in the past. Zinc (E) and echinacea (A) have not been demonstrated to consistently improve symptoms of the common cold. Oseltamivir (B) is a treatment for influenza, which typically has outbreaks in the winter and typically causes higher fevers and prominent myalgias.

A 7-year-old male presents to the clinic with complaints of headaches and episodes of feeling sweaty and flushed. He also reports that at times he feels as if his heart is racing. He was full term, had an uncomplicated birth, and has been otherwise healthy until now. On exam his BP is 120/80 mmHg and is the same in his upper and lower extremities. His weight and height are in the 50th percentile for his age. What is a likely cause of Billy's hypertension? A. Coarctation of the aorta B. Renal vascular disease C. Renal insufficiency due to renal scarring D. Catecholamine excess E. Primary hypertension

> The correct answer is D. A. Coarctation of the aorta should be suspected in a child with elevated BP (usually > 99th percentile), little family history of HTN, and a discrepancy between upper and lower extremity BPs. Some children with coarctation of the aorta may go undetected until presenting with hypertension at a school-age visit. It is important to pay special attention to the femoral pulses and to document BP measurement in a lower extremity. This patient's BP in his upper and lower extremities are the same, making this diagnosis less likely. B. Umbilical arterial or venous lines as neonate (most often in premature infants) can predispose a child to renal vascular disease. This patient had an uncomplicated birth and did not go the neonatal ICU, making it less likely that his hypertension is due to renal vascular disease secondary to an umbilical arterial or venous line as a neonate. C. Recurrent urinary tract infections in childhood are one of the leading causes of hypertension and renal insufficiency later in life due to renal scarring following infections. UTIs are more common in girls. This patient does not have a stated history of urinary tract infections, and his palpitations and flushing are not consistent with hypertension secondary to UTIs. D. Catecholamine excess (pheochromocytoma or neuroblastoma) should be suspected in a child who is hypertensive and has episodes of sudden sweating, flushing, or feels that his heart is racing. This patient is exhibiting these signs and a urine catecholamine testing would be appropriate in this case. E. Most hypertension in children over 6 years of age, and in adolescents, is due to primary HTN. Obesity is an important correlate. This patient's flushing and racing heart would not be fully explained by primary hypertension. Also, this patient is an appropriate weight, making this diagnosis less likely.

A 7-year-old male frequently gets in trouble at school for being disruptive and talking inappropriately in class. He does not follow directions and does not work well with classmates during group exercises. His mother relates that at home, he is always on the go, sleeping only 6 to 7 hours each night, and he does not follow the rules at home. He often skips his homework and sometimes puts himself in danger, such as by running away unaccompanied. Which of the following is the most likely diagnosis? A. Bipolar mood disorder B. Anti-social personality disorder C. Functional neurologic symptom disorder D. Attention deficit hyperactivity disorder E. Rett syndrome SUBMIT

> The correct answer is D. A. Depression may be responsible for the inattention this child exhibits in school. It is not uncommon for childhood depression to lead to bipolar disorder, in which hyperactivity and impulsivity comprise the manic phase of the disorder leading to a decreased need for sleep. However, the symptoms of depression and mania present in separate phases, not concurrently, as is described in this child. The mnemonic commonly used for mania is DIGFAST (distractability, irresponsibility, grandiosity, flight of ideas, agitation, sleep decrease, talkativeness). The mnemonic for depression is SIGECAPS (sleep disturbance, loss of interest, guilt, energy loss, concentration impairment, appetite changes, psychomotor retardation, suicidal ideation). B. Symptoms of anti-social personality disorder include inability to conform to societal norms, disregard of the rights of others, and often criminality. These individuals often exhibit impulsiveness due to their lack of consideration of the consequences of their actions. Males are affected more than females. However, a diagnosis of anti-social personality disorder can be made only in individuals older than 15 years, earlier than which a diagnosis of conduct disorder is appropriate. The patient described does not display the above characteristics. C. Functional neurologic disorder (formerly called Conversion disorder) would be suspected if no cause could be identified for reported physical symptoms. The drive is unconscious on the part of the patient, and symptoms are not intentionally produced or faked. Functional neurologic disorder is more common in adolescents and involves a sudden loss of sensory or motor functioning. When the patient consciously creates physical symptoms this is referred to as factitious disorder. D. ADHD is characterized by the triad of impulsivity, hyperactivity, and inattention- all of which are present in the child described above. Other symptoms include motor impairment and emotional lability. ADHD is typically diagnosed before the age of 7 but persists into adulthood. Intelligence is usually normal, but individuals with ADHD commonly perform more poorly academically than would be expected for their IQ. E. Rett syndrome is an X-linked pervasive developmental disorder seen only in females; affected males die in utero or at birth. The characteristic symptoms involve regression of language and development, intellectual disability, ataxia, and hand-wringing. This disorder is typically diagnosed earlier, at about age 1-4. This patient is male and does not have symptoms of regression.

Mr. Jones is a 67-year-old male brought into your office because he has been having "dizzy spells and room spinning" for the past two days that are intermittent. Your records indicate a history of back pain, diabetes and hypertension. Upon further questioning, Mr. Jones cannot identify when these spells come on and nothing seems to relieve them. His temperature is 98.6 Fahrenheit; blood pressure is 165/95 mmHg; heart rate is 78 beats/minute; and respiratory rate is 18 breaths/minute. On physical exam, you notice a slight nystagmus. You ask him to focus on your nose but the nystagmus continues. What is the most likely cause of his "dizzy spells"? A. Anemia B. Bleeding gastric ulcer C. Hyperthyroidism with thyroid storm D. Stroke E. Vestibular neuritis

> The correct answer is D. This patient displays symptoms of vertigo and findings of nystagmus suggesting a neurologic problem with balance. Thus, anemia (A), bleeding ulcer (B), and thyroid storm (C) - all of which may present with presyncope (but should not cause vertigo) - are not likely diagnoses. He is likely suffering from a central lesion (ie: TIA/stroke) causing vertigo. The acute onset of symptoms, nystagmus that changes direction and nystagmus that does not resolve with gaze fixation are consistent with a central lesion as opposed to a peripheral lesion. In a peripheral cause, such as vestibular neuritis (E), one would expect the patient's nystagmus to resolve upon focusing on your nose.

What caloric deficit is needed to lose one pound of body weight? Choose the single best answer. The best option is indicated below. Your selections are indicated by the shaded boxes. A. 1,500 B. 2,000 C. 2,500 D. 3,000 E. 3,500 F. 4,000 G. 4,500

> The correct answer is E A patient needs to have approximately a 3500 calorie deficit to lose 1 pound of weight.

Which of the following may elevate blood pressure measurements? Select all that apply. The best options are indicated below. Your selections are indicated by the shaded boxes. A. Anxiety over being at the doctor's office B. Use of a BP cuff that is too large C. Taking a BP below the level of the heart D. Patient is in pain SUBMIT

> The correct answers are A, C, D.

Other than dietary modifications and exercise, which therapies are approved for management of obesity? Select all that apply. A. Oral drug therapy B. Liposuction C. Insulin injection D. Bariatric surgery E. Laxative usage

> The correct answers are A, D

Which laboratory tests would you like to obtain for Mr. James? Select all that apply. The best options are indicated below. Your selections are indicated by the shaded boxes. A. Fasting glucose B. Complete blood count C. Renal function (BUN and creatinine) D. Fasting lipid profile E. Serum cortisol F. None at this time

> The correct answers are A, D

Which of the following are characteristic symptoms of seasonal flu? Select all that apply. A. Bloody diarrhea B. Fever C. Myalgia D. Scarlatiniform rash E. Shortness of breath F. Sore throat

> The correct answers are B, C and F.

An asymptomatic, healthy 9-month-old female is found to have a palpable RUQ mass on exam. After further imaging and lab studies, the mass is diagnosed as a neuroblastoma that has involvement in the bone marrow as well. The mother is worried about the prognosis. Which of the following is true about the prognosis of neuroblastoma in this child? A. Lymph node involvement is a poor prognostic factor B. Prognosis of neuroblastoma is predictable C. Children who are older than 12 months have a better prognosis than younger children D. Favorable histology does not play a role in prognosis E. Non-amplification of the n-myc gene is a favorable prognostic factor.

> The correct answer is E. A. Due to the effectiveness of chemotherapy, neuroblastomas with lymph node involvement are still considered favorable, especially in the setting of other favorable factors, such as young age and differentiating histology. Though distant metastasis is a significant poor prognostic factor, regional lymph nodes do not significantly affect the outcome. B. Neuroblastoma has a broad spectrum of clinical courses. Some tumors may spontaneously regress, some may mature to a benign type, and yet other tumors can be very aggressive with metastases. Age plays a role in the prognosis, as most infants have a good prognosis even with disseminated disease, while infants over 18 months of age do not do as well. C. In infants less than one year of age, neuroblastoma tumors may spontaneously regress. Stage 4S neuroblastoma is a special category that is reserved for infants less than 12 months who have resectable primary tumors and metastases to the liver, skin, and bone marrow. Overall survival is over 85 percent for babies over 6 weeks of age with Stage 4S. D. Favorable histology is a good prognostic factor in neuroblastoma, and is based on the differentiation of the cells involved. E. Non-amplification of the n-myc gene is one of the favorable genetics in neuroblastoma.

An 8-year-old boy is brought to clinic by his parents because they are concerned that he has not been doing his homework. His teacher recently called the parents to say that their son seems distracted in class, constantly interrupts other children when they are speaking, and is very fidgety. When you speak with the boy, he tells you that he did not know about the homework assignments and that he tries hard to pay attention in class. What is the next best step in management? A. Prescribe a stimulant medication for ADHD B. Suggest behavior modification for the child and parenting classes C. Recommend group therapy for the child D. Do nothing, as this child's behavior is normal E. Find out more about his behavior at home and at school

> The correct answer is E. A. Pharmacotherapy is often used in combination with behavioral modification/group therapy for children diagnosed with ADHD. However, to be diagnosed with ADHD, one must have 6 or more symptoms in 2 or more settings for at least 6 months, and several of these symptoms must be present before the age of 12. The symptoms fall within the three categories of inattention, hyperactivity, and impulsivity. A stimulant medication may be prescribed once a diagnosis of ADHD is made. B. Behavior modification for the child and parenting classes for the parents are both used as treatment modalities in ADHD. Parents should be counseled on positive reinforcement, firm non-punitive limit setting, and how to reduce external stimuli. However, this child first requires further evaluation. C. Children with ADHD often learn best in group therapy, particularly social skills and self-esteem. Again, this child first requires further evaluation. D. Many school-aged children are easily distractible, impatient, and hyperactive. It is important to distinguish those who truly have ADHD from those who do not. Again, to have a diagnosis of ADHD, symptoms must be present in two or more settings. E. Contacting the teacher to find out more about the child's behavior at school and learning more about his behavior at home are the best ways to determine if 6 of the symptoms are present in 2 or more settings, which is required to make the diagnosis of ADHD. It also will be important to learn more about other aspects of this child's life, as there are several factors that can lead to acting out (including learning disability, hearing disability, family stress, and abuse).

You are seeing is a 35-year-old female with no past medical history who presents with dizziness for the past week. She says these episodes of dizziness in which the room feels as though it is spinning last for a minute or two at most but she cannot seem to identify what is causing them. She denies any change in diet, headaches or recent illness. Her father passed away from a stroke at the age of 60 but she denies any other significant family medical history. Her vital signs are all normal, as is her head and neck exam. Her lungs are clear to auscultation bilaterally, and her cardiac exam reveals a regular rate and rhythm with no murmurs. You are unable to elicit saccades with a head thrust maneuver. You perform a Dix-Hallpike maneuver, which elicits her symptoms and causes rotary nystagmus when she looks to the right. What is the most appropriate next step? A. Cardiac enzymes B. ECG C. Emergency CT scan D. Emergency MRI scan E. Epley maneuver

> The correct answer is E. This patient likely has benign paroxysmal positional vertigo (BPPV) caused by calcium carbonate debris in the semicircular canals. BPPV is the most common cause of vertigo. Her presentation is classic: episodic vertigo triggered by position changes lasting seconds or minutes. Patients may not always be able to identify triggers unless specifically questioned. Despite a negative head thrust, which would suggest a central lesion in the vestibular system, the Dix-Hallpike maneuver is diagnostic for BPPV. While always good to consider in your differential given the severity of a cerebrovascular event, head CT (C) or MRI (D) to evaluate for central lesions is not indicated in a young patient with no other risk factors. While dizziness may be related to cardiac causes, she clearly describes symptoms of vertigo. Hence, an ECG (B) and cardiac enzymes (A) are also unnecessary. Therefore, the Epley maneuver used to reposition the canalith in the vestibular canals would treat her BPPV.

Of the following, which are developmental milestones that you expect to see in a developmentally appropriate 9-month-old infant? Select all that apply. The best options are indicated below. Your selections are indicated by the shaded boxes. A. Waves bye-bye B. Has a well developed pincer grasp C. Sits without support D. Walks well E. Says 2 words plus "mama" and "dada"

> The correct answers are A and C.

Which of the following patients with vertigo would require neurologic imaging? Select all that apply. A. A 68-year-old woman with a history of hypertension and sudden acute onset constant vertigo. She has right nystagmus that changes direction with gaze and that does not disappear when she focuses. B. A 45-year-old man with recurrent episodes of brief intense vertigo every time he turns his head rapidly. He has no other neurologic signs or symptoms. He has a positive Dix-Hallpike maneuver. C. A 66-year-old man with recurrent episodes of vertigo associated with tinnitus and hearing loss. His head thrust test is positive with saccade back to center when his head is thrust to the right. D. A 28-year-old otherwise well woman with new onset constant vertigo with no other neurologic symptoms. On physical exam, she has unidirectional nystagmus that disappears when her gaze is fixed. E. A 40-year-old woman with history of migraine and new onset headache with constant, non-positional vertigo. Her head thrust test displays a normal response.

> The correct answers are A and E.

Which of the following are components of the metabolic syndrome? Select all that apply. The best options are indicated below. Your selections are indicated by the shaded boxes. A. Impaired fasting glucose B. Increased blood pressure C. Increased low-density lipoprotein (LDL) cholesterol D. Increased BMI SUBMIT

> The correct answers are A, B

Which of the following conditions are important causes of school failure? Select all that apply. A. Sensory impairment (e.g., hearing, vision) B. Sleep disorders C. Mood disorders (e.g., depression) D. Excessive sugar intake E. Learning disability F. Conduct disorders G. Hyperthyroidism

> The correct answers are A, B, C, E, F.

Which four non-pharmacologic interventions lower LDL cholesterol? Select all that apply. A. Reduction of saturated fat in diet B. Increased dietary fiber C. Alcohol cessation D. Increased physical activity E. Weight loss

> The correct answers are A, B, D, E

Which of the following are possible sequelae of obesity? Select all that apply. A. Obstructive sleep apnea B. Dyslipidemia C. Reactive airway disease D. Hypertension E. Slipped capital femoral epiphysis F. Type I diabetes mellitus G. Type II diabetes mellitus H. Steatohepatitis

> The correct answers are A, B, D, E, G, H.

Which of the following are appropriate therapeutic choices for a patient with peripheral vertigo? Select all that apply. The best options are indicated below. Your selections are indicated by the shaded boxes. A. Antimicrobial agents B. Diuretics C. Epley maneuvers D. Vestibular rehabilitation exercises E. Vestibular suppressant medications

> The correct answers are B, C, D and E. For information about the management of peripheral vertigo, see the Teaching Point below.

Which of the following are risk factors for childhood obesity? Select all that apply. The best options are indicated below. Your selections are indicated by the shaded boxes. A. Breastfeeding B. High birth weight C. Obese parent D. Delayed puberty E. Lower socioeconomic status F. Prader-Willi syndrome SUBMIT

> The correct answers are B, C, E, F.

Which of the following statements regarding stimulant medications are true? Select all that apply. The best options are indicated below. Your selections are indicated by the shaded boxes. A. Stimulant medications are addictive when used to treat children with ADHD. B. Prolonged use of stimulant medication is associated with later increased incidence of substance abuse. C. Stimulant medications may decrease appetite. D. Stimulant medications simply mask behavioral problems by sedating the child. E. Stimulant medications can cause tics. F. Patients may develop insomnia. G. Stimulant medication may cause decreased growth velocity. H. Stimulant medication leads to an increased risk of sudden cardiac death in otherwise healthy children.

> The correct answers are C, F, G.

Dr. Barnett then asks you to consider a differential diagnosis for Mrs. Saleh's dizziness based on your findings from her history and physical exam. From the following, select the top three diagnoses on your differential. Dizziness after URI A. Anemia B. Anxiety C. Benign paroxysmal positional vertigo (BPPV) D. Cardiac arrhythmia E. Cerebellar infarct F. Meniere's disease G. Orthostatic hypotension H. Labyrinthitis I. Transient ischemic attack (TIA) J. Vestibular migraine K. Vestibular neuritis

> The correct answers are C, H and K. You realize that when you moved in front of Mrs. Saleh with the ophthalmoscope, she lost her fixation and you noticed increased nystagmus, indicating that she has a peripheral lesion. This makes central problems such as cerebellar anxiety (B), infarct (E), TIA (I), and vestibular migraine (J) less likely. Orthostasis (G) has been ruled out in Mrs. Saleh's case by her normal orthostatic vital signs. Cardiac arrhythmia (D) typically causes dizziness and presyncope rather than the sensation of vertigo, which Mrs. Saleh has. Peripheral causes of vertigo include BPPV (C), Meniere's disease (F), Labyrinthitis (H), and vestibular neuritis (K). Of these, Meniere's disease is less likely because it causes recurrent episodes of vertigo associated with unilateral hearing loss and tinnitus.

Asia is an asymptomatic, thriving 9-month-old girl incidentally noted to have a RUQ mass and pallor on routine well-child care exam. She has no lymphadenopathy or jaundice. Which of the following are on your differential diagnosis? Select all that apply. A. Appendiceal abscess B. Congestive heart failure C. Constipation D. Fatty liver disease E. Hepatic abscess F. Hepatic neoplasm G. Hydronephrosis H. Neuroblastoma I. Teratoma J. Wilms' tumor

> The correct answers are F, G, H, I, J.

Dr. Medel asks you, "If we are to follow the 2017 ACC/AHA hypertension guidelines, what is the goal blood pressure we want to achieve with this patient?" Choose the single best answer. A. <130/80 mmHg B. <140/80 mmHg C. <150/90 mmHg

A In the ACC/AHA guidelines, for all patients, the goal blood pressure is 130/80 mmHg.

A 23-year-old female patient comes to your office complaining of bothersome symptoms the week before her period each month. She reports that she has significant breast tenderness, is very irritable, and eats significantly more than she does at any other time during the month. Her coworkers notice the difference in her mood and it is beginning to affect her interactions with them. The symptoms resolve after her period. She has no other medical problems or significant past medical history. Physical exam is normal. What is this patient's most likely diagnosis? A. Premenstrual syndrome B. Primary dysmenorrhea C. Premenstrual dysphoric disorder D. Secondary dysmenorrhea

A 23-year-old female patient comes to your office complaining of bothersome symptoms the week before her period each month. She reports that she has significant breast tenderness, is very irritable, and eats significantly more than she does at any other time during the month. Her coworkers notice the difference in her mood and it is beginning to affect her interactions with them. The symptoms resolve after her period. She has no other medical problems or significant past medical history. Physical exam is normal. What is this patient's most likely diagnosis? A. Premenstrual syndrome B. Primary dysmenorrhea C. Premenstrual dysphoric disorder D. Secondary dysmenorrhea

From the following list, select all the tests (and testing intervals) that the USPSTF recommends for colon cancer screening. Select all that apply. A. Fecal occult blood testing (FOBT), collected at home annually B. Colonoscopy every 10 years C. Barium enema every five years D. Computed tomography (CT) colonography every five years E. Fecal immunochemical (FIT) testing every 5 years

A, B, D

Which of the following medical conditions is associated with depression? Select all that apply. A. Hypothyroidism B. Parkinson disease C. Hypertension D. Dementia E. Asthma

A, B, D

Dr. Wilson asks you: Which of the following are common manifestations of end-organ damage caused by Type 2 diabetes? Select all that apply. A. Coronary heart disease B. Cerebrovascular disease C. Inflammatory bowel disease D. Hyperthyroidism E. Retinopathy F. Neuropathy G. Nephropathy H. Primary pulmonary hypertension

A, B, E, F

He asks, "Which of the following do you want to order for Mr. Reynolds?" Select only those that are strongly recommended by evidence-based guidelines. Select all that apply. A. Glucose B. Lipid panel C. TSH D. Ferritin E. Liver test panel F. Electrolyte panel: Na, K, Cl, CO2, BUN, Creatinine

A, B, F The correct answers are A, B, F. Mr. Reynolds is above age 40 and obese so needs a screen for diabetes (A) and cardiovascular disorders (B). Due to his history of hypertension, an electrolyte panel (F) to asses his kidney function is recommended. He does not need to be screened for thyroid disease (C), iron deficiency anemia (D) and liver disease (E).

What other tests could be done to find out the etiology of her hyperthyroidism? Select all that apply. A. Radioactive iodine uptake test and scan B. Thyroid ultrasound C. Anti-thyrotropin receptor antibodies D. MRI of the thyroid gland.

A, C

Which of the following are true statements about eye involvement in Graves disease? Select all that apply. A. The most common manifestations of Graves ophthalmopathy are eyelid retraction and exophthalmos. B. More than 50% of patients with Graves disease have clinically significant eye problems. C. The eye manifestations of Graves disease always happen when the level of thyroid hormones are high. D. Primary symptoms of the eye manifestations of Graves disease, when they occur, are related to corneal irritation from the eyelid retraction. E. While most of the time the eye signs and symptoms are bilateral, they can be unilateral. F. Treatment of the hyperthyroidism improves the eye manifestations of Graves disease.

A, D, E The most common manifestations of Graves ophthalmopathy (eye problems) are eyelid retraction and exophthalmos. Primary symptoms of the eye manifestations of Graves disease are related to corneal irritation from eyelid retraction. While most of the time the eye signs and symptoms are bilateral, they can be unilateral. While 50% of patients with Graves have some eye involvement by MRI, only about 20% to 30% of those are clinically relevant. In up to 10% the eye manifestations can happen when the patient is euthyroid or even hypothyroid. Treatment of hyperthyroidism does not affect the eye manifestations. In fact, some patients who get radioactive iodine will experience worsening symptomatology.

Which tests would you order to rule out other causes for symptoms of insomnia, fatigue, and a depressed mood? Select all that apply. A. Complete metabolic panel B. Urinalysis C. Brain CT scan D. TSH E. Chest x-ray F. CBC

A, D, F A complete metabolic panel (A) screens for electrolyte, renal, and hepatic problems A TSH (D) can detect hypothyroidism A CBC (F) will show anemia and vitamin deficiencies

Teenage patient has BMI of 26 You determine that this percentile places her in which of the following categories? Choose the single best answer. The best option is indicated below. Your selections are indicated by the shaded boxes. A. Obese B. Overweight C. At risk for overweight D. Normal weight SUBMIT

A: BMI Interpretation For children and teens, BMI age- and sex-specific percentiles are used to interpret the BMI for two reasons: The amount of body fat changes with age. The amount of body fat differs between girls and boys. For children, the American Academy of Pediatrics designates: BMI (kg/m2) Healthy 5th - 85th percentile Overweight 85th - 95th percentile Obese >95 percentile

Which of the following are known health risks of obesity. Select all that apply. A. coronary heart disease B. stroke C. osteoarthritis D. obstructive lung disease E. gallbladder disease F. obstructive sleep apnea G. endometrial cancer H. lung cancer I. breast cancer

All but D, F Obesity is associated with many poor health outcomes, including cardiovascular disease (including hypertension, dyslipidemia, stroke (B), and coronary heart disease(A)); osteoarthritis (C); obstructive sleep apnea (F); gallbladder disease (E); and several cancers (including breast, endometrial, pancreatic, renal, colon). Lung cancer (H) has not been associated with obesity. While obesity is a cause of restrictive lung disease, it is not associated with obstructive lung diseases (D).

Dr. Lee says, "Of course, these recommendations are based (in part) on the risk a patient will develop cervical cancer. So, now I have a question for you: What do you think are the risk factors for developing cervical cancer?" Select all that apply. A. Early onset of sexual intercourse. B. Multiple sexual partners. C. Cigarette smoking. D. Immunosuppressed patient.

All of the above

When you leave the room, Dr. Barnett asks you: What do you think are the advantages of group prenatal care? Select all that apply. The best options are indicated below. Your selections are indicated by the shaded boxes. A. Group prenatal visits offer an opportunity for group support and peer education. B. Group visits allow more time for education and anticipatory guidance with each patient. C. Particularly among African American women, preterm delivery is less likely if they participate in group prenatal care. D. Prenatal knowledge, labor preparedness, adequacy of prenatal care and patient satisfaction of women in group prenatal care appear to be better than those in routine care. E. Women enrolled in group prenatal visits who experience preterm deliveries have infants with greater birth weights than those enrolled in routine prenatal care.

All of the above

A 65-year-old female presents to your office for a routine visit. She is found to have a blood pressure of 146/96 mmHg. You repeat the blood pressure in her other arm and get 148/92 mmHg. Her pulse is 70 and regular. Her last BP reading was one year ago and was 120/76 mmHg. She has no other medical problems. Her BMI is 28. She states that she likes to walk 30 minutes every other day with her husband and has been doing that for years now. What is the most appropriate diagnosis at this time? A. White coat hypertension B. Elevated blood pressure reading C. Stage 1 hypertension D. Stage 2 hypertension E. Secondary hypertension

Answer Comment The correct answer is B. To diagnose hypertension, two separate readings greater than 130/80 mmHg each time - taken a week or more apart - are needed. Furthermore, ideally home blood pressure readings in the hypertensive range would be needed to confirm that she does not have white coat hypertension. Because this patient has had elevated blood pressure documented on only one occasion (today), the most appropriate current diagnosis is elevated blood pressure. If she has a second similarly elevated reading, Stage 2 hypertension may be diagnosed. Stage 1 hypertension refers to blood pressures between 130-139/80-89 mmHg. This patient has not yet been diagnosed with hypertension, so neither A, C, D nor E is appropriate.

A 55-year-old male with no significant past medical history presents for a routine physical exam. He last saw a doctor five years ago. Social history is remarkable for a 35-pack-year tobacco history since the age of 20. He indicates that his wife and children have urged him to quit smoking for the last few months. When you ask him if he has considered quitting, he replies, "I just don't see what the big deal is!" Which stage of change best describes this patient at this time? A. Action B. Contemplation C. Maintenance D. Precontemplation E. Preparation

Answer Comment The correct answer is D. Based on this man's response, it appears he has not actively considered quitting smoking despite his family's concern. All options refer to different stages in the Transtheoretical stages of change model. Given that he has not actively contemplated quitting, the best stage to describe this patient at this time would be the Precontemplation stage and not any of the other responses.

What antibodies seen in Graves v Hashimoto's

Anti-thyrotropin receptor antibodies (TRAb) are the pathologic mechanism for Graves disease and can be detected in the vast majority of patients with this condition. In patients with undiagnosed causes of hyperthyroidism, third-generation assays for TRAb are 97% sensitive and 99% specific for Graves. These antibodies are to be distinguished from anti-thyroid peroxidase (TPO) antibodies, which are elevated in 90% of patients with Hashimoto thyroiditis and 75% of patients with Graves.

A 48-year-old male with a past medical history that includes hypertension, chronic obstructive pulmonary disease (COPD), and hyperlipidemia presents to clinic as a new patient in October for a general physical exam. History reveals that he has been smoking a pack of cigarettes daily since age 20. He drinks two beers daily. He is intermittently nonadherent with his medications. Review of the state immunization database reveals that the only immunization he has received as an adult was a tetanus diphtheria shot administered 12 years ago. Which of the following vaccine combinations would be most appropriate for this patient? A. Influenza, meningococcal, and zoster B. Influenza, pneumococcal, and Tdap C. Influenza, zoster, and Tdap D. Meningococcal, pneumococcal, and Tdap E. Meningococcal, pneumococcal, and zoster

B Because this patient has a diagnosis of COPD and smokes cigarettes, both pneumococcal and annual influenza vaccination are indicated. Because his last tetanus immunization was over 10 years ago and because he has not had a booster pertussis shot as an adult, a one-time Tdap is recommended. At this time meningococcal vaccine is recommended for adolescents and young adults and not indicated for this patient. Zoster vaccine is recommended to all adults at age 50 or older.

How long should Mr. Smith remain anticoagulated after DVT? Choose the single best answer. A. Two months. B. Three months. C. Six months. D. Twelve months. E. Indefinitely.

B In Mr. Smith's case, he has a first episode of proximal idiopathic (unprovoked) thromboembolic disease with a low risk of bleeding. Recent guidelines suggest that he should be anticoagulated for three months (B). The option exists to extend this treatment for a total of six months, but data have not demonstrated this to be helpful. Given that six months of anticoagulation was considered the standard of care for many years, many students may see patients treated for this duration for an unprovoked DVT. Data clearly demonstrate that recurrence rates are high for treatment durations shorter than three months (A). Indefinite anticoagulation (E) is reserved for patients with chronic risk factors for VTE (e.g. Protein C deficiency).

You are seeing one of your regular patients, a 65-year-old female for a follow-up appointment for intractable knee pain from osteoarthritis. The knee pain has not responded to ibuprofen or acetaminophen. She has a past medical history that also includes obesity, diabetes, hypertension and depression. Her current medications include aspirin, HCTZ, metformin, and duloxetine. You are considering prescribing tramadol. She has never taken any kind of opioid medication in the past. Which of the following potential problems should you inquire about when adding tramadol? A. Hypotension B. Seizures C. Rash D. Recent hyperglycemia E. Hypercoagulability

B Tramadol is a centrally-acting synthetic opioid analgesic that works by binding to mu-opioid receptors and weakly inhibiting norepinephrine and serotonin reuptake. Significant side effects can include lowering the seizure threshold in patients with epilepsy, serotonin syndrome, respiratory depression, angioedema, bronchospasm and dependency. Other common side effects include constipation, nausea, dizziness, and pruritis. Hypotension, rashes, glycemic control, and hypercoagulability are not profoundly impacted by tramadol.

How long do postpartum blues typically last? Choose the single best answer. A. One week B. Two weeks C. Four weeks D. Eight weeks

B - 2 weeks Postpartum blues refers to mild, though often rapid, fluctuations in mood within the first two weeks postpartum, often peaking at about day five. Multiple factors likely contribute to postpartum blues. They usually resolve over time with support, reassurance, and rest. Women with postpartum blues should be counseled about seeking care if symptoms worsen or are not improved by two-weeks postpartum.

Which of the following are true about depression among minority populations in the U.S.? Select all that apply. A. Hispanics have higher rates of depression than non-Hispanic whites. B. Hispanic patients are less likely to be diagnosed than non-Hispanic whites. C. Asian, black and Hispanic patients with depression are less likely than whites to perceive a need for mental health treatment. D. Minority populations are more likely to suffer from psychotic depression. E. Racial, ethnic and economic minorities receive evidence-based treatments at the same rates as white Americans.

B, C

Which of the following statements are true regarding hyperosmolar hyperglycemic state (HHS)? Select all that apply. A. Metabolic acidosis is the primary disturbance. B. Dehydration is a common finding. C. Plasma glucose levels are commonly >600 mg/dL. D. Ketones are absent or mildly elevated. E. The mortality rate is extremely low.

B, C, D

Which of the following can be appropriate treatment options for an acute DVT? Select all that apply. A. Outpatient initiation of warfarin alone B. Outpatient initiation of rivaroxaban alone C. Inpatient admission with administration of unfrationated heparin overlapping with the initiation of warfarin D. Outpatient initiation of low molecular weight heparin (LMWH) overlapping with the initiation of warfarin E. Outpatient initiation of dabigatran alone

B, C, D The day-to-day risk of the development of a pulmonary empolism (PE) is high in patients with acute DVT, so immediate anticoagulation is necessary. This is to be distinguished from the day-to-day risk of a stroke in patients with newly diagnosed atrial fibrillation (which is much lower!). Most patients with DVT may be managed in the out-patient setting, though there are a few important exceptions (see below). Two oral factor Xa inhibitors (rivaroxaban and apixaban) have been demonstrated to be safe as monotherapy for DVT (option B). These agents have been shown to have similar efficacy to warfarin in preventing PE, but have been demonstrated to cause fewer bleeding episodes than warfarin. Advantages include that they do not require any laboratory monitoring, so they are much easier for patients to take. Disadvantages include their cost (compared to warfarin, which is very inexpensive) and the unavailability of immediate reversal agents in the case of dangerous bleeding. The American College of Chest Physicians recently recommended the choice of factor Xa inhibitors or the direct thrombin inhibitor dabigatran (collectively referred to as non-vitamin K antagonist oral anticoagulants or NOACs) over warfarin for the management of DVT or PE. The direct thrombin inhibitor dabigatran is another option for oral anticoagulation that has similar advantages to rivaroxaban and apixaban. It has not been studied as monotherapy however (option E), so it is recommended that patients be initiated on LMWH with five to 10 days of overlap. Before the development of the novel oral anticoagulants (NOACs), warfarin was the mainstay of the management of DVT. It remains an acceptable option and remains commonly used in many settings. Warfarin is a better option for patients who can't afford the cost of the NOACs and who are concerned about the lack of reversal agents. Warfarin takes several days to reach therapeutic efficacy, so simply starting it alone carries an unacceptable risk of PE (option A). Thus, patients must be started on either LMWH or unfractionated heparin while waiting for the patient's INR to come into the therapeutic range (2-3). LMWH is the preferred anticoagulant to pair with warfarin in most settings, and may be administered in the outpatient setting (option D) For many years, the standard of care for DVT was admission to the hospital and administration of unfractionated heparin overlapping with the initiation of warfarin (option C). Inpatient management remains the best option for patients who are hemodynamically unstable, who are at serious risk of acute bleeding with the initiation of anticoagulation (e.g. those with prior admission for gastrointestinal bleeding), or who have obstacles to outpatient management. Examples of this include the inability to afford NOACs and LMWH, or inability to get daily INRs checked during the initiation of warfarin therapy.

What are the United States Preventive Services Task Force Screening Recommendations for this patient? Select all that apply. (55, obese) A. Lung cancer screening B. High blood pressure screening C. Pancreatic cancer screening D. Testicular cancer screening E. Alcohol misuse screening and counseling F. Hepatitis C screening G. Depression screening

B, E, F, G Mr. Reynolds is 55 years old and obese, and as such meets USPSTF screening recommendations for hypertension (A) (all adults age 18 years and older); alcohol misuse screening (E) recommended for adults 18 years older; a one-time screening for HCV infection to adults born between 1945 and 1965 (F); and depression (G) in the general population.

Which of the following would be an appropriate response for a clinician who is not religious? Choose the single best answer. A. I would like to, but Dr. Wilson is waiting for me to join him. B. I don't think I should as my faith and beliefs may differ from yours. C. I will join you.

C

A 55-year-old male comes to the clinic for a visit. He has read about the dangers of being overweight and inquires about which category he fits into. He is 5' 10'' (1.78 m) and weighs 220 lbs (100 kg), BMI = 31.6. Which of the following categories most accurately describes the patient based on his BMI? The best option is indicated below. Your selections are indicated by the shaded boxes. A. Ideal B. Morbidly (very severely) obese C. Obese D. Overweight E. Underweight

C obese The categories of weight, according to BMI measurements, are as follows: Underweight: BMI < 18.5 Ideal: BMI 18.5 to 25 Overweight: BMI 25 to 30 Obese: BMI 30 to 40 Morbidly (very severely) Obese: BMI > 40.

Dr. Medel asks you, "Which of the following accurately describes the best way to measure blood pressure?" Select all that apply. The best options are indicated below. Your selections are indicated by the shaded boxes. A. The patient should be seated quietly for thirty minutes before a blood pressure measurement is taken. B. The patient should be seated on an examination table when a blood pressure is taken. C. The patient's arm should be supported at heart level. D. The length of the bladder of the cuff must be at least 80% of the arm circumference. E. An adult-sized cuff should be used for most adults.

C,D The patient should be seated quietly for at least five minutes in a chair with their back supported, rather than on an examination table. The arm should be supported at heart level. Blood pressure should be auscultated with a stethoscope using a properly calibrated and validated instrument. The most important point is that an appropriately sized cuff be used to ensure accuracy. The length of the bladder of the cuff should be at least 80% of the arm circumference. A cuff that is too small will give you an erroneously high blood pressure reading. Adults with obesity may require an extra large or "thigh-sized" cuff.

Which of the following is correct regarding breast self-examination? Choose the single best answer. A. Most women regularly perform breast self-examination. B. The practice of regular breast self-examination by trained female patients reduces mortality. C. Breast self-examination increases the number of biopsies performed. D. It's recommended to teach women breast self-examination

C. Breast self-examination increases the number of biopsies performed

What is the prevalence of obesity in adolescents in the United States? Choose the single best answer. The best option is indicated below. Your selections are indicated by the shaded boxes. A. 5% B. 12% C. 16% D. 20%

D

A 33-year-old G0P0 female presents to your clinic for her first prenatal visit. Her home pregnancy test was positive, she has been experiencing mild nausea for two weeks. Her last normal menstrual period was six weeks ago. What test should her initial prenatal visit include? A. Quad screen B. Abdominal ultrasound C. Complete metabolic panel D. Complete blood count E. Glucose challenge test

D An initial pregnancy evaluation should include CBC, RPR, HIV, Rubella, Blood type and Hepatitis B. A CBC is included to test for nutritional and congenital anemias as well as platelet disorders. The quad screen typically occurs later in the pregnancy (15-20 weeks) and transvaginal ultrasound (not abdominal ultrasound) is more reliable for dating a pregnancy at six weeks' gestation. A complete metabolic profile is not a routine part of an initial pregnancy evaluation, and should be ordered only when indicated. A glucose challenge test, in an average-risk prenatal patient like this one, is a screening test performed at 24 to 28 weeks gestational age, and not at the first prenatal visit.

A 53-year-old man has been experiencing three months of weight loss and palpitations. Work-up for hyperthyroidism, which is high on your differential, is pending. Which of the following symptoms is inconsistent with a diagnosis of hyperthyroidism? A. Tremor B. Insomnia C. Gynecomastia D. Constipation E. Fatigue

D The correct answer is (D) constipation. The other answer choices are typically associated with hyperthyroidism, while constipation is usually seen with hypothyroidism. Gynecomastia seen in 10% to 40% of patients with Graves disease and is thought to be because of sex hormone binding globulins being increased in Graves disease. Fatigue is common in both hyper- and hypothyroidism.

What is official USPSTF magazine for women <50

Decision to start regular, biennial screening mammography before age 50 should be an individual one and take patient context into account, including the patient's values regarding specific benefits and harms

As the incidence of childhood overweight and obesity has increased, there has also been an increase in which of the following health problems in children? Choose the single best answer. The best option is indicated below. Your selections are indicated by the shaded boxes. A. Diabetes Type II B. High cholesterol C. Hypertension D. B and C E. All of the above

E

When Dr. Lee returns from her discussion with Mr. Payne, you tell her you would like to order which of the following tests? Choose the single best answer. A. CBC B. UA C. X-ray D. MRI E. None of the above

E In the absence of red flags or findings suggestive of systemic disease, diagnostic testing, especially imaging, is not indicated until after four to six weeks of conservative treatment. Ordering tests too early is not only cost ineffective, but can also cause harm to the patient.

What is the grade of evidence of full skin examination by a primary care clinician for skin cancer screening in the adult general population by United States Preventive Service Task Force? Choose the single best answer. A. Grade A B. Grade B C. Grade C D. Grade E E. I statement

E In this clinical setting, the patient presents to the office with a suspicious skin condition, so a whole body skin examination by a physician may be warranted.

What do Non-modifiable risk factors include for Breast cancer

Family history of breast cancer in a first-degree relative (i.e., mother or sister) Prolonged exposure to estrogen, including menarche before age 12 or menopause after age 45 Genetic predisposition (BRCA 1 or 2 mutation) Advanced age (The incidence of breast cancer is significantly greater in postmenopausal women, and age is often the only known risk factor.) Female sex Increased breast density

What is first step to evaluate a breast lump

First, take a good history from the patient, including:

Osteoporosis screening recommendation

For women > 65 years old, screening with dual energy x-ray absorptiometry (DEXA) is recommended. For women < 65 years old, the USPSTF recommends using the World Health Organization's Fracture Risk Assessment Tool to risk-stratify. Screening with DEXA is recommended if the risk of fracture is greater than or equal to that of a 65-year-old white woman without additional risk factors (9.3 percent over 10 years). These recommendations are being reviewed by the USPSTF.

What is the most common cause of hyperthyroidism in adults and children? Choose the single best answer. A. Toxic nodular goiter (thyroid nodules) B. Toxic diffuse goiter (Graves disease) C. Thyroiditis D. Excessive iodine ingestion E. Drug induced hyperthyroidism

Grave's disease Toxic diffuse goiter (Graves disease) accounts for the majority (60-80%) of hyperthyroidism

What causes low estrogen states

Low estrogen states may be caused by early menopause (i.e., before age 45 years), prolonged premenopausal amenorrhea, and low weight and body mass index.

What is common hypothyroid cause of goiter

Hashimoto's disease

PHQ-2 Questions

Have you often been bothered by feeling down, depressed, or hopeless? (Yes/No) Have you often been bothered by little interest or pleasure in doing things? (Yes/No)

Dr. Hill poses a hypothetical question to you: Suppose Mr. Smith is on warfarin instead of dabigatran, and his INR comes back as 11.2 and he has no signs of active bleeding. Which of the following is the most appropriate approach to take next? Choose the single best answer. A. Discontinue warfarin, give Vitamin K 5 mg orally. B. Omit one dose of warfarin, if INR decreases in 24 hours begin warfarin at 1/2 original dose. C. Discontinue warfarin, repeat INR in 24 hours. D. Continue current dose of warfarin, recheck INR at 5 days. E. Discontinue warfarin, give 5 mg Vitamin K IV, and repeat until INR less than 4.0.

If the goal INR is substantially overshot, it increases the risk of bleeding complications significantly. Warfarin should be held, and an oral dose of Vitamin K should be given to reduce INR. Omitting a dose of warfarin is an insufficient response to a potentially dangerous situation. Probably the second best answer is to discontinue the warfarin and repeat the INR in 24 hours, and this would be appropriate if the INR was greater than five and less than nine. It is inappropriate to continue warfarin at the current dose because of risk of bleeding. Finally, discontinuing warfarin, giving 5 mg Vitamin K IV, and repeating until INR less than 4.0 is an overreaction to a supratherapeutic INR in a non-bleeding patient.

Why need to do B12 and TSH screening in diabetes

In addition to renal insufficiency, metformin can cause another side effect to take into account when deciding which labs to order. During clinical trials, up to 7% of patients receiving metformin developed asymptomatic subnormal serum vitamin B12 levels. In the setting of neuropathy, too, serum B12 levels would be a very reasonable diagnostic test to order. Consider screening patients with Type 1 diabetes for autoimmune thyroid disease and celiac disease soon after diagnosis.

What do Q waves that are greater than 25% of succeeding R wave and greater than 0.04 seconds suggest

Indicate infarction

What is on differential for lateral ankle injury after inversion

Lateral ankle sprain peroneal tendon tear, fibular fracture, talar dome fracture, subtalar dislocation

When use breast MRI

Not recommended for screening the general population of asymptomatic, average-risk women. May be indicated in the surveillance of women with more than a 20% lifetime risk of breast cancer (for example, individuals with genetic predisposition to breast cancer by either gene testing or family pedigree, or individuals with a history of mantle radiation for Hodgkin's disease). May be used as a diagnostic tool to identify more completely the extent of disease in patients with a recent breast cancer diagnosis. Contrast-enhanced breast MRI may be indicated in the evaluation of patients with breast augmentation in whom mammography is difficult.

What are side effects of TCAs

Older antidepressants such as TCAs can cause arrhythmias

What are physiologic reasons for nipple discharge

Physiologic: Pregnancy Excessive breast stimulation

Patient is a smoker at age 45, what vaccine to receive?

Pneumovax 23

What are specific q's for breast lump history

Precise location of the lump; How it was first noticed (accidentally, by breast self-examination, clinical breast examination, or mammogram); How long it has been present; Presence of nipple discharge; and Any change in size of the lump. (Especially ask whether the lump changes in size according to phase of the menstrual cycle.)

What factors are associated with decreased breast cancer

Pregnancy at an early age Late menarche Early menopause High parity Use of some medications, such as selective estrogen receptor modulators and, possibly, nonsteroidal anti-inflammatory agents and aspirin.

What are pathologic reasons for nipple discharge

Prolactinoma Breast cancer > Intraductal papilloma > Mammary duct ectasia > Paget disease of the breast > Ductal carcinoma in situ Hormone imbalance Injury or trauma to breast Breast abscess Use of medications (e.g., antidepressants, antipsychotics, some antihypertensives and opiates)

What are some interventions to help quit smoking

Quit rates are highest when patients are engaged in a group setting. Oral medications are somewhat effective at helping people stop smoking, with quit rates at 12 months 1.5 to 3 times the placebo quit rate. When combined with medication, a series of one-on-one counseling sessions (as in a physician's office) enhances quit rates. Providing practical problem-solving skills, assistance with social supports, and use of relaxation/breathing techniques can increase quit rates. Financial or material incentives such as those provided in the workplaces, clinics and hospitals appear to increase cessation rates while in place.

What are risk factors for osteoporosis due to

Risk factors for osteoporosis are mainly due to low estrogen states.

What mneumonic use for preventative visits

Risk factors: Identify risk factors for serious medical conditions during history and physical exam. Immunizations: Provide recommended immunizations/chemoprophylaxis. Screening tests: Order appropriate screening tests. Education: Educate patients on ways to live healthier while reducing risks for disease.

What are some smoking cessation strategies

Setting a quit date Using nicotine replacement Joining a support group Calling 1-800-QUIT-NOW Choosing an activity to substitute for smoking (e.g., taking a walk or chewing sugarless gum when the urge to smoke occurs) Making a list of the reasons why it is important to quit smoking and keeping it handy to refer to Keeping track of where, when, and why you smoke (helps identify smoking triggers to avoid) Throwing away all tobacco and smoking paraphernalia (i.e., ashtrays, lighters, anything else associated with the smoking habit) Taking medication

Ms. Marcos is a 65-year-old woman with a past medical history of Type 2 diabetes, hypertension, and hypercholesterolemia who presents with six months of insomnia despite self-medication with acetaminophen, diphenhydramine, and herbal remedies. She is 5' 2" and weighs 250 lbs. When considering a differential diagnosis, which one of the following is a common cause of insomnia in the elderly? A. Asymptomatic coronary artery disease B. Chronic sinusitis C. Hypoparathyroidism D. Pneumonia E. Sleep Apnea

Sleep apnea Sleep apnea occurs in 20% to 70% of elderly patients. Obstruction of breathing results in frequent arousal that the patient is typically not aware of; however, a bed partner or family member may report loud snoring or cessation of breathing during sleep.

Which of the following is generally considered to be helpful in distinguishing between COPD and asthma? Choose the single best answer. A. FEV1/FVC is greater than 60% in asthma, but less than 60% in COPD. B. FVC is increased in COPD, but unchanged in asthma. C. Air-flow obstruction in asthma is reversible, but in COPD it is not. D. Macrophages and T killer cells play a role in asthma, but not in COPD.

The correct answer is C.

"What do you think are the key findings?" Dr. Hill replies, "That's right. So, based on these key findings, what do you think is our diagnosis?" Choose the single best answer. A. Threatened abortion B. Inevitable abortion C. Incomplete abortion D. Missed abortion E. Septic abortion F. Complete abortion

Spontaneous abortion is the loss of a pregnancy without outside intervention before 20 weeks' gestation. Spontaneous abortions can be subdivided into: Threatened abortion - bleeding before 20 weeks gestation. Threatened abortion is simply a pregnancy complicated by bleeding before 20 weeks gestation, and is - in some ways - a "catch-all" descriptive diagnosis. Inevitable abortion - dilated cervical os. Incomplete abortion - some but not all of the intrauterine contents (or products of conception) have been expelled. Missed abortion - fetal demise without cervical dilitation and/or uterine activity (often found incidentally on ultrasound without a presentation of bleeding). Septic abortion - with intrauterine infection (abdominal tenderness and fever usually present). Complete abortion - the products of conception have been completely expelled from the uterus.

What does Horizontal ST segment depression or downsloping ST segment suggest

Suggests cardiac ischemia

how to dx compartment syndrome

The 6 P's (Signs of limb threatening Injury) Pain Pallor Pulselessness Paresthesia (A skin sensation, such as burning, prickling, itching, or tingling.​) Perishing cold (The inability to regulate one's body temperature.) Paralysis

What screening tool is used for alcoholism

The Alcohol Use Disorders Identification Test (AUDIT) is the most widely validated screening tool. It consists of 10 items and takes two to three minutes to complete, and longer to score. AUDIT-C is a brief version of the AUDIT comprising three questions scaled 0 to 12

Update for cervical cancer vaccine -2

The CDC notes that as of 2017, only the 9-valent will be available in the U.S. Another important change is that the HPV series is considered complete after two doses in patients who receive the first dose before age 15 and the second dose at least five months after the first (ideally six to 12 months apart)

What is USPSTF recommendation for prostate cancer screening

The U.S. Preventive Services Task Force (USPSTF) changed its recommendation for prostate-specific antigen (PSA) testing from a D (recommending against it) to a C (selective recommendation) for men ages 55 to 69. They found that there is a slight reduction in death from prostate cancer among men who have PSA testing. The number needed to screen is 781 men to prevent one death from prostate cancer. There are also significant rates of complications from both PSA screening and treatment for prostate cancer as well as evidence of significant rates of overdiagnosis. Thus they concluded that the decision to screen should be an individualized one between a doctor and an individual patient.

What are USPSTF recs for CRC screening

The USPSTF recommends screening for colorectal cancer beginning at age 50 years and continuing until age 75 years using: fecal occult blood testing sigmoidoscopy colonoscopy Fecal Immunochemical Testing (FIT Test) Fecal DNA Testing CT Colography

What are ovarian cancer screening recs

The USPSTF, the American College of Obstetricians and Gynecologists, and the American College of Physicians all recommend against routine screening for ovarian cancer in asymptomatic women.

Which of the following are benefits of menopausal HT? Select all that apply. The best options are indicated below. Your selections are indicated by the shaded boxes. A. Decrease in vasomotor symptoms. B. Decreased incidence of stroke. C. Improvement in atrophic vaginitis symptoms. D. Decreased risk of breast cancer. E. Prevention of osteoporosis.

The correct answer are A, C, E.

Which bedside instruments used for diagnosing delirium does evidence best support? Choose the single best answer. The best option is indicated below. Your selections are indicated by the shaded boxes. A. Confusion Assessment Method (CAM) B. Mini-Mental State Examination (MMSE) C. MiniCog SUBMIT

The correct answer is A.

Question Which of the following would be considered a Category I strip? Choose the single best answer. A. FHR of 130 with moderate variability, accelerations, and occasional variable decelerations. B. FHR of 144 with absent variability and late decelerations C. FHR of 155 with moderate variability and some early decelerations. D. FHR of 180 with minimal variability SUBMIT

The correct answer is C. Answer (C) is a category I strip because of the normal fetal heart rate, moderate variability. Early decelerations (a sign of head compression when the fetus is low in the pelvis) are permissible in category I.

Which of the following would be non-reassuring with respect to fetal status? Choose the single best answer. The best option is indicated below. Your selections are indicated by the shaded boxes. A. A baseline fetal heart rate of 170 beats per minute. B. Moderate variability with an amplitude of about 12 beats per minute. C. The patient reports active fetal movement throughout the day. D. Two fetal heart rate accelerations in a 20 minute period.

The correct answer is A. A baseline fetal heart rate of 170 beats per minute is defined as tachycardia, and would be cause for concern. Many problems could cause fetal tachycardia, including maternal fever or infection and fetal anemia or hypoxia. Minimal (5 or less beats per minute), absent, or marked (greater than 20 beats per minute) variability of the fetal heart rate.

A 61-year-old female has recently been diagnosed with Type 2 diabetes. Her fasting glucose was 240 mg/dL and her A1C was 8.9%. Her BP has been 148/90 and 146/86 at two separate office visits. Her home BP measurements have been in a similar range. Her creatinine is 0.9 and she has no known heart disease. She currently takes losartan 100 mg daily for a diagnosis of hypertension. Which of the following would be the most appropriate step in managing this patient's blood pressure? A. Make no changes to her medications as her blood pressure is at goal. B. Start amlodipine daily. C. Start furosemide daily. D. Start lisinopril daily. E. Start metoprolol daily.

The correct answer is (B). According to the 2017 AHA/ACC blood pressure guidelines, this patient's blood pressure goal should be 130/80 mmHg. She is clearly above that, and she should have a blood pressure medication added (or in a highly motivated patient, dramatic behavioral changes with close follow up). There is no preference for a first-line treatment for blood pressure in diabetic patients without albuminuria, although many providers start with an ACE inhibitor or ARB because diabetes is a risk factor for chronic kidney disease. Furosemide and metoprolol are not among the four major classes of medications for blood pressure management (ACEIs, ARBS, calcium channel blockers, and thiazides), so (C) and (E) are not acceptable choices. Lisinopril (D) is an acceptable first-line choice, but it should not be combined with an ARB. Since this patient is taking losartan (an ARB), adding an ACEI is contraindicated. Amlodipine is a good choice for this patient.

A 30-year-old male with PMH significant for one month of progressive hoarseness and fever presents with a painful neck mass. He reports increased sweating, racing heart, diarrhea, and fatigue. His vital signs are: Temperature: 39.5 Celsius Heart rate: 85 beats/minute Respiratory rate: 19 breaths/minute Blood pressure: 130/70 mmHg On physical exam, you palpate a hard, fixed, painless nodule on the left side of the thyroid gland. Bloodwork results show: TSH: decreased Free T4: increased Which of the following is correct about thyroid nodules? A. 25% of patients with hyperthyroidism are caused by thyroid nodules B. The majority of thyroid nodules symptomatic C. 4% to 5% of thyroid nodules are cancerous D. Older patients with thyroid nodules usually have solitary nodules E. Thyroid radioactive iodine uptake and scan is the best initial test to evaluate a new thyroid nodule

The correct answer is (C). Four percent to 5% of thyroid nodules are cancerous. Only 5% cases of hyperthyroidism are due to nodules (A). Most thyroid nodules are asymptomatic and are found by physical exam or imaging done for another purpose (B). Older patients are more likely to have multinodular disease, and younger patients solitary nodules (D). The best initial imaging for a thyroid nodule is thyroid ultrasound (E). The patient in this case could either have an ultrasound or fine needle biopsy as this case is consistent with thyroid cancer, probably papillary.

A 42-year-old female presents for a visit after recently being diagnosed with Type 2 diabetes. She has made a plan to work on diet and exercise. Her A1C is found to be 8.0%. What is the best medicine to start at this time? A. A sulfonylurea B. An SGLT2 inhibitor C. GLP-1 receptor agonist D. Insulin E. Metformin Basal

The correct answer is (E). Metformin is the best first medication to choose in this case. Sulfonylureas (A), GLP-1 receptor agonists (C), and SGLT2 inhibitors (B), DPP-4 inhibitors, thiazolidinediones, or insulin (D) may be used as second-line agents, depending on key patient factors including a) important comorbidities such as ASCVD, chronic kidney disease, and heart failure, b) hypoglycemia risk, c) effects on body weight, d) side effects, e) costs, and f) patient preferences.

A 45-year-old woman presents to the ED with two weeks of abdominal pain, progressive weakness and palpitations. She notes an unexplained 3-lb weight loss as well as black, sticky diarrhea. Her vital signs are: Temperature: 99.1 Fahrenheit Heart rate: 117 beats/minute Respiratory rate: 22 breaths/minute Blood pressure: 92/67 mmHg She appears pale and diaphoretic. Her neck is supple and non-tender. Lungs are clear to auscultation bilaterally. Cardiac exam reveals elevated heart rate and a diastolic murmur. Her abdominal exam is notable for diffuse epigastric pain and hyperactive bowel sounds. Which of the following additional lab values might you expect? The best option is indicated below. Your selections are indicated by the shaded boxes. A. PaO2 of 60 B. Elevated LDH C. Free T4 of 15 mcg/dL (nml range 4.5-11.2 mcg/dL) D. TSH of 0.3 uU/mL (nml range 0.5-5.0 uU/mL E. Hgb of 8.4 g/dL (nml range 12.0-16.0 g/dL) F. Platelet count of 530,000 /mm3 (nml range 150,000-400,000/mm3)

The correct answer is (E). This woman appears to be suffering from anemia due to GI losses, as evidenced by her abdominal pain, weakness, tachycardia, diastolic murmur, and tar-like stools. There is some evidence of exam of hyperthyroidism (C), like tachycardia and diarrhea, but the other aspects of her presentation point to loss of blood. PaO2 is not impacted in blood loss (A), nor is platelet count (F). LDH is elevated in hemolytic anemia.

Ms. Burton is a 45-year-old woman who has never been to a primary care provider. She presents today to establish care and get her health in order. Her concerns today are: fatigue, weakness, numbness, insomnia, feeling sad at times, anhedonia, increased appetite, weight gain, dry skin, and increasing hair loss within the past month. Her vital signs are: Heart rate: 78 beats/minute Respiratory rate: 18 breaths/minute Oxygen saturation: 95% Blood pressure: 152/84 mmHg Weight: 325 lbs Body Mass Index: 41 kg/m2 Today, her physical exam is significant for thinning hair, poor dentition, a systolic murmur heard at the left upper sternal border, an obese abdomen, and bilateral knee stiffness and pain on range of motion exam. Remainder of the physical exam is within normal limits. Which laboratory tests or studies can be done to rule out medical causes of insomnia, fatigue, and depression? A. CBC, CMP, and TSH B. Chest-X Ray C. CT head without contrast D. HgbA1c, lipid panel, urine microalbumin E. MRI brain with contrast

The correct answer is A. CMP can be used to detect electrolyte, renal and hepatic problems. TSH can be used to rule out hypo- or hyperthyroidism. CBC can be helpful to detect anemia and vitamin deficiencies. In addition, ESR can be used to test for rheumatologic disease. An ECG should be done if the patient is using drugs that might alter cardiac conductivity, such as TCAs.

Which of the following treatments would you choose for Mr. Fitzgerald's squamous cell skin cancer? Choose the single best answer. A. Wide excision under local anesthesia in the office B. Refer Mr. Fitzgerald for Mohs surgery. C. Treat the lesion with topical 5-fluorouracil (5-FU). D. Treat the lesion with cryotherapy. E. Refer Mr. Fitzgerald for radiation therapy. F. Observation for now, because it is still carcinoma in-situ.

The correct answer is A. Surgical excision (A) is the best option for Mr. Fitzgerald's lesion. Topical treatments and radiation destroy the malignant cells. However they do not offer the opportunity to examine the margins of the tissue to confirm complete eradication of malignant tissue. In contrast, Mohs microscopic surgery is more extensive than what Mr. Fitzgerald's lesion requires, as his lesion is relatively low risk in a cosmetically insignificant area, so there's no reason to be overly careful about sparing tissue in this region, and a wide excision should suffice.

Of these three choices, which is the best option for managing Mrs. Hernandez's condition right now? Choose the single best answer. A. Directly admit Mrs. Hernandez to a hospital bed with telemetry. B. Give Mrs. Hernandez IV furosemide in the office and oral furosemide at home. C. Send Mrs Hernandez by ambulance to the emergency department for stabilization and likely admission.

The correct answer is C. If your patient has new-onset CHF and you are unsure of the exact precipitating factors (particularly MI), it is usually safest to send the patient directly to the Emergency Department via an ambulance.

A 55-year-old male with no significant past medical history and generally healthy behaviors presents to clinic for a health care maintenance exam. He says, "I'd like to get tested for all types of cancer." He does not have any family history of cancer. Review of systems is negative for any symptoms of prostate cancer, such as urinary frequency, urgency, retention, hematuria, weight loss, or back pain. He is a lifelong non-smoker, and he doesn't drink alcohol or use recreational drugs. Which of the following screening tests is given either a USPSTF A or B recommendation in favor of its routine use for patients such as this one? A. Colon cancer screening B. ECG screening for coronary artery disease C. Lung cancer screening D. Pancreatic cancer screening E. Prostate specific antigen (PSA) testing

The correct answer is A. The USPSTF gives colon cancer screening an A recommendation for people age 50 to 75 years due to clear evidence of benefit. Lung cancer screening is given a B recommendation for 55-year-old men with a 30 pack-year tobacco history and who have smoked in the past 15 years. This patient is a non-smoker. Pancreatic cancer screening and ECG screening are both given D recommendations (against their use). PSA screening is given a C recommendation, indicating that doctors and patients should make individualized decisions about the use of this test.

Ms. Rogers is a 75-year-old woman who was found unresponsive in her house by her neighbor who had come over to help clean her house. An empty unlabeled pill container was found next to her on the bathroom floor. She was rushed to the ER, stabilized and is now in ICU on a mechanical ventilator. Which of the following are true regarding suicide in the elderly? A. Approximately 75% of the elderly who commit suicide had visited a primary care physician within the preceding month, but their symptoms went unrecognized. B. Elderly persons attempting suicide are more likely to be married and living with their spouse. C. Elderly persons attempting suicide usually report good sleeping habits. D. Firearms are the most common means of suicide in the elderly. E. Suicidal behaviors increase with age, but rates of completed suicides don't.

The correct answer is A. The USPSTF recommends screening all adults for depression, but especially patients with chronic diseases like diabetes, as they are at high risk for depression. The PHQ-2 inquires about the frequency of depressed mood and anhedonia over the past 2 weeks. The purpose of the PHQ -2 is not to establish a final diagnosis, but rather to screen for depression as a "first-step" approach. Patients who screen positive should be evaluated by the PHQ-9 to determine whether they meet the criteria for depression. Another screening tool which can be used is the Geriatric Depression Scale - Short Form (GDS-SF) which includes a series of 15 questions. Specifically related to suicide in the geriatric population: Elderly persons attempting suicide are more likely to be WIDOW(ER)S, AND LIVE ALONE; Elderly persons attempting suicide have REDUCED sleep quality; Suicidal behaviors DO NOT increase with age, but rates of completed suicides DO.

Mr. Giovanni is a 37-year-old male who drives a delivery truck. He presents to your clinic after acute onset of severe lower back pain, which began after lifting a large package while at work. When you enter the room, you find him standing, unable to sit comfortably. On physical exam, he has limited lumbar flexion, reduced to 45 degrees, positive straight leg test at 45 degrees on the left, normal gait, but difficulty with heel walk. He has 4/5 strength on the left with ankle plantar flexion. Strength is preserved on the right. Which of additional physical exam finding would be consistent with this man's level of disc herniation? A. Hypoactive ankle tendon reflex B. Decreased range of motion on lumbar extension C. 2/5 strength on hip flexion D. Decreased rectal tone E. Positive Stoop test

The correct answer is A. The clinical signs presented by this patient - difficulty with heel walk and the abnormal strength of ankle plantar flexion - is consistent with nerve root impingement at the level of L5-S1. Of the answers listed, a hypoactive ankle tendon reflex is also consistent with a nerve root impingement at this level. Pain with lumbar extension suggests degenerative disease or spinal stenosis, and spinal stenosis is similarly suggested by a positive stoop test. Diminished hip flexor strength suggests a lesion at the L2, L3, or L4 level and decreased rectal tone suggests a cauda equina lesion.

A 62-year-old female presents for follow-up of her hypertension and diabetes. In general, her chronic diseases are well controlled and she has suffered no target organ damage. She has worked hard to begin exercising, and is walking vigorously five times a week. She has also worked hard on dietary changes, and has been following the DASH eating plan very seriously. She quit smoking three months ago. Her blood pressure today is 148/88 mmHg, pulse is 72 and BMI is 32. She is taking metformin 500 mg twice daily, simvastatin 20 mg daily and hydrochlorothiazide (HCTZ) 25 mg daily, and she is adherent with her daily medications. Her labs today include an A1C of 6.6, an LDL of 88 and a basic metabolic panel within normal limits. Which of the following management steps today do you consider the most appropriate? The best option is indicated below. Your selections are indicated by the shaded boxes. A. Add amlodipine 5 mg daily B. Change her simvastatin from 10 mg to 20 mg C. Impress upon her the importance of making more lifestyle modifications D. Increase HCTZ to 50 mg daily E. Make no changes as she is at her treatment goals

The correct answer is A. The goal blood pressure for patients with hypertension is 130/80 mmHg, and this patient has not met this goal with HCTZ and major lifestyle changes. Increasing the dose of HCTZ from 25 to 50 does not improve blood pressure further, so adding a second medication would be more beneficial. While commending her on her lifestyle changes is important, counseling about intensifying them is not likely to be realistic nor helpful given all that she has already done. There is no need to change her statin, however calculating her ASCVD risk to determine whether she is on the appropriate dose would be helpful. The current cholesterol guidelines recommend a moderate intensity statin for patients with diabetes. For simvastatin, a dose of 10 mg represents a low-intensity dose. Increasing this to 20 mg would put her management more in line with these guidelines, though it would not address her elevated blood pressure.

Which of the following diagnoses do you consider most likely for Mr. Wright? Choose the single best answer. Patient is unwilling to cooperate, irritated, not happy A. Progression of stroke B. Stroke related dementia C. Post-stroke depression D. Major depression E. Dysthymia F. Personality disorder of aging G. Side effect of medication

The correct answer is C. Mr. Wright has post-stroke depression (C). His symptoms of depressed mood, decreased appetite, and irritability have been present for one week and do not appear to be severe; his lack of a prior history of depression is also supportive.

A 45-year-old female presents to your office complaining of left calf pain and swelling. She reports first noticing the pain after her yoga class earlier in the week. She denies having any chest pain or shortness of breath. She has no significant PMH. Her physical exam is significant for tenderness to palpation over the left calf. No edema or redness is present. The circumference of her left calf is 10 cm and the circumference of her right calf is 9 cm. What is the most appropriate next step in diagnosis? The best option is indicated below. Your selections are indicated by the shaded boxes. A. D-dimer B. MRI C. Lower extremity Doppler D. CT angiography

The correct answer is A. This is the case where the pretest probability of a DVT is low, so that a negative D-dimer would save further testing. If the D-dimer is positive then the next step would be the lower extremity Doppler. CT and MRI are expensive and do not add anything to diagnostic accuracy in the majority of cases.

A 54-year-old male with a history of chronic gout and GERD presents to your office for his health maintenance exam. Vital signs today are blood pressure 138/88 mmHg, pulse 65 beats/min, respirations 10/min, afebrile, BMI 29 kg/m2. He smokes 10 cigarettes per day, does not regularly exercise, and drinks one to two beers daily, four or five times a week. He has no current concerns, review of systems is negative, and his physical exam is unremarkable. You recommend lifestyle changes. Which of the following changes is least likely to improve his cardiovascular risk? A. Alcohol cessation B. DASH eating plan C. Increased exercise D. Smoking cessation E. Weight loss

The correct answer is A. While all of these options are reasonable lifestyle modifications to recommend for patients, this particular patient's blood pressure is least likely to be reduced by alcohol cessation. Moderate alcohol consumption actually improves blood pressure by 2 to 4 mmHg; therefore, stopping his moderate alcohol consumption could increase his blood pressure. However, it is not recommended to encourage alcohol use in patients who do not drink, because of the risk of encouraging problem drinking. Initiation of the NIH-sponsored Dietary Approaches to Stop Hypertension (DASH) eating plan has been shown to lower systolic pressure as have smoking cessation, weight loss, and increased exercise.

Which of the following would meet the diagnosis of preeclampsia with severe features? Choose the single best answer. A. Blood pressure of 180/125 mmHg B. 2+ pitting edema of the lower extremities C. Serum creatinine of 0.7 mg/dL D. 100 mg/dL of protein on a urine dipstick

The correct answer is A. 2+ edema of the lower extremities (B), while uncomfortable, is certainly not unusual in pregnancy and is not a sign of severe preeclampsia. While no longer considered criteria of preeclampsia, non-dependent edema (face and hands) is suggestive of preeclampsia. A serum creatinine of 0.7 mg/dL (C) is normal and not a sign of severe preeclampsia. 100 mg/dL of proteinuria on a dipstick (D) does not fall in the severe range, but would meet the criteria for preeclampsia. In fact, the amount of proteinuria is no longer used to define whether preeclampsia has severe features or not, as it is not consistently associated with more severe outcomes in comparison to the other criteria for preeclampsia with severe features

A 47-year-old female presents to the clinic complaining of an "itchy patch" on her skin. On further examination, you note a solid, elevated 1.5cm lesion on the extensor surface of the right forearm. How would you best describe the lesion? A. Plaque B. Papule C. Patch D. Nodule E. Macule

The correct answer is A. A plaque is a palpable flat lesion that is greater than 1cm in size. Papules are small raised palpable lesions that are less than 1cm in size. Nodules are solid raised palpable lesions which are larger in height and dimension than papules, occurring in the epidermis, subcutaneous tissue, or dermis. A macule is an area of skin discoloration that is 0.5 to 1cm in diameter, and a patch is a larger area of differently colored skin which is smooth to touch.

You are seeing a 62-year-old man with a history of osteoarthritis in his knees as well as well-controlled hypertension and chronic constipation. His arthritic pain has been disabling in recent months, making it very difficult for him to do his work as a plumber. He has tried full dose acetaminophen in combination with diclofenac, but still he reports 8/10 pain and stiffness. He would like to pursue other treatment options. His current medications include chlorthalidone, diclofenac, acetaminophen, and methylcellulose. On exam, he is in no acute distress and his vital signs are normal. His knees show no warmth, erythema or gross deformity. They are stable to varus and valgus stress. The Lochman test and McMurray test are both normal. There is moderate crepitus bilaterally. Which of the following would be the most appropriate next step in the management of his pain? A. Refer to physical therapy for strength and mobility training B. Prescribe amitriptyline nightly C. Prescribe a glucosamine sulfate and chondroitin sulfate combination pill D. Prescribe oxycodone after reviewing a pain management agreement and performing a urine drug screen E. Schedule for bilateral intra-articular hyaluronic acid (viscosupplementation) injections

The correct answer is A. There is clear evidence that exercise self-management programs, typically guided by physical therapists, help with osteoarthritis pain and stiffness. Tricyclic antidepressants can play a role in chronic pain management in appropriate patients, but they cause significant constipation. This would not be an appropriate choice for this patient, who takes a medication for chronic constipation. There is consistent evidence that glucosamine sulfate and chondroitin sulfate (alone or in combination) have no appreciable impact on symptoms from osteoarthritis and should be avoided. The same is true for hyaluronic acid injections. Chronic narcotics can play a role in the management of pain in selected patients, but they should only be considered after all safer options (e.g., physical therapy) have been tried first.

A 72-year-old female presents to the clinic one year after having suffered a stroke that left her with some residual left sided weakness/paralysis. On exam, the left shoulder demonstrates a severe decrease in both active and passive ROM and significant pain. The patient has a history of hypertension, hyperlipidemia, and diabetes. What is the most likely diagnosis? A. Adhesive capsulitis B. Osteoarthritis C. Rotator cuff tear D. Biceps tendonitis E. Subacromial bursitis

The correct answer is A. Adhesive capsulitis, also known as frozen shoulder, is characterized by pain and stiffness in the shoulder joint. Symptoms usually appear gradually and the majority will resolve in 1-3 years. The risk of developing adhesive capsulitis increases with conditions that limit the mobility of the arm, such as recovery from injury, stroke or mastectomy and also in diabetics. Adhesive capsulitis demonstrates a decrease in both active and passive ROM. Loss of active and passive ROM is more likely due to joint disease, whereas loss of only active ROM is more likely due to muscle tissue pathology. While osteoarthritis can present with decrease in both passive and active ROM, it is less common in the shoulder than in the hip and knee and the history is classic for adhesive capsulitis.

A 24-year-old male presents to clinic with history of upper abdominal pain, nausea and vomiting. He has previously had his gallbladder removed due to symptomatic gallstones. In reviewing his history, you want to screen for alcohol abuse due to the possibility of pancreatitis. Which one of the following is a sign of alcohol use disorder (AUD)? Choose the single best answer. A. Failure to fulfill work, school or social obligations due to the effects of drinking B. Denial of a drinking problem C. A score of 2 on the AUDIT-C test D. Consumption of 10 drinks per week E. Three drinks per social occasion SUBMIT

The correct answer is A. Alcohol Use Disorder is characterized by a maladaptive pattern of alcohol use in which patients report two or more signs of the negative effects of alcohol on their health and lives. Such negative effects include finding that drinking interferes with taking care of one's family and/or home; or has caused job troubles or school problems. Denying the presence of a drinking problem does not distinguish a patient with AUD from one without. The AUDIT-C test is a good screening test for AUD, though it is not one of the diagnostic criteria for AUD. For a male patient, a score of 4 or more is considered positive. The number of drinks a person consumes are not part of the DSM 5 definition of AUD.

A 15-year-old female with a history of allergic rhinitis presents to the clinic with a five day history of productive cough and wheezing that is worse at night. She denies any shortness of breath, chest pain, or fever. The patient states that she has had similar symptoms in the past, especially when seasons change. The only reported past medical history is atopic dermatitis which is well-controlled with an over-the-counter steroid cream. What is the best next step? A. Prescribe a short acting beta agonist inhaler with a short course of oral steroids B. Prescribe a long acting beta agonist inhaler C. Prescribe a daily corticosteroid inhaler D. Prescribe a daily corticosteroid and long acting beta agonist inhaler E. Prescribe antibiotic therapy

The correct answer is A. Because the patient is symptomatic and wheezing, she needs to be treated with a medication to control her symptoms and treat her exacerbation. A short acting beta-agonist and short course of oral steroids do just that. After controlling her symptoms and treating her exacerbation, a more in-depth interview needs to be performed around the asthma symptoms and other ancillary tests need to be ordered and interpreted including a chest x-ray and pulmonary function test. Then, a decision can be made regarding daily controller inhalers (inhaled corticosteroids, chromolyn, etc). Finally, antibiotic therapy is not indicated in the treatment of an asthma exacerbation.

Which of the following therapies are recommended as first-line therapy for secondary prevention of noncardioembolic TIA or noncardioembolic stroke? A. Aspirin B. Warfarin (Coumadin) C. Pravastatin (Pravachol) D. Recombinant tissue plasminogen activator (t-PA) E. Ticlopidine (Ticlid)

The correct answer is A. For noncardioembolic (or atherothrombotic, lacunar) strokes, possible first-line therapies for secondary prevention include aspirin, aspirin in combination with extended-release dipyridamole (aggrenox), and clopidogrel (Plavix). For cardioembolic strokes, aspirin may potentially be desirable in patients with a contraindication for anticoagulation, but most people will need to be on coumadin for secondary prevention of stroke. High-intensity statins are recommended to help treat high cholesterol in patients who have had a stroke or have risk factors for stroke. Pravastatin is a low-intensity statin, which would not be recommended in a patient with prior atherosclerotic disease such as this. T-PA is used in the acute management of a stroke and not as secondary prevention of noncardioembolic stroke. Ticlopidine is an antiplatelet agent that is not as effective as aspirin or clopidogrel. It is, in fact, no longer available in the U.S.

Ms. Kovacs is a first time mother you are seeing on postpartum day two in the hospital, with her newborn son Christopher. Her pregnancy was uncomplicated and Christopher was born full-term via spontaneous vaginal delivery with no complications and a birth weight of 7 lbs. (3178 grams), 0 oz. Ms. Kovacs is getting ready to be discharged home today but is concerned because the baby now weighs 6.8 lbs (3087 grams) and she hasn't been able to produce milk every time Christopher wants to feed. Upon receiving your recommendations, she has been attempting to breastfeed with good technique every two to three hours with strong latching and vigorous suckling. Mom is still concerned her baby isn't getting enough to eat. Which of the following would be the most appropriate advice for mom? A. Continue to attempt breastfeeding B. Stop attempting to breastfeed and begin formula C. Switch from breast milk to formula for a few days to allow moms body to produce more milk, and then attempt breastfeeding again D. Remain in the hospital until Christopher is having full and consistent feeds with breast milk E. Request a lactation consultation to ensure the baby gains weight SUBMIT

The correct answer is A. Given the history, the correct approach is for the mother to continue breastfeeding as it can take up to 72 hours after delivery for significant milk production to begin. It would be inappropriate to supplement with or switch to formula. There is no rationale for continued hospitalization as the baby is suckling well on a regular basis that will stimulate appropriate milk production in the mother. A lactation consultation is unnecessary as the baby's weight loss is normal (down only 3% since birth) and as the mother has appropriate technique and the baby is latching and suckling well. However, a visit from a lactation consultant prior to discharge can be a very helpful resource for new moms and can provide additional support should parents encounter breastfeeding problems once they are at home.

Ms. Wicket is a 30-year-old female establishing with you as a new patient. Her past medical history is significant for hypothyroidism for which she has been without treatment for five years. She currently takes no medication and has no surgical or family history. She is single, with no children, drinks alcohol socially, and does not use tobacco or other recreational drugs. The only records she has are from blood work that she had done last month significant for a very elevated thyroid stimulating hormone (TSH). What constellation of symptoms would you expect to find on your review of systems? A. Dry skin, sensitivity to cold, constipation, and fatigue B. Palpitations, sweating,lightheadedness, and fatigue C. Depression, frequent bowel movements, sweating and increased appetite D. Tremor, hair changes, difficulty sleeping and heavy menses SUBMIT

The correct answer is A. Hypothyroidism classically causes fatigue, drying of the skin and hair, constipation and sensitivity to the cold (A), though patients rarely report all of these at the same time. Goiter and weight gain are also common symptoms which may suggest hypothyroidism. Palpitations, sweating, lightheadedness and fatigue (B) are all symptoms of hyperthyroidism. It is worth noting that fatigue is actually a symptom of both hyper and hypothyroidism. Options C and D list mixtures of symptoms of both hyper and hypothyroidism. Depression and heavy menses are typically thought of as associated with hypothyroidism. Increased stool frequency, sweating, tremor, increased appetite and difficulty sleeping are more likely with hyperthyroidism.

Ms. Jones is a 35-year-old female with a significant past medical history of SLE who had been on NSAID therapy for the previous three months presented four weeks ago with heartburn. At that time, she reported episodic, mealtime epigastric burning radiating to the throat for the past few months. She has had no surgeries. Serologic testing for H. pylori IgG was reported to be positive a few days after her visit and she was begun on triple therapy. She now returns to the office for follow up. Today she denies any epigastric burning or tenderness. Physical exam is not significant. Which of the following is an accepted indication for performing repeat testing at this visit for H. pylori eradication? Choose the single best answer. A. Restarting of chronic NSAID therapy for SLE B. Documentation is required for all patients with confirmed H. pylori infection C. Treatment with triple rather than quadruple therapy D. Positive serologic test prior to therapy E. Age below 40 years

The correct answer is A. Indications for testing for proof of H. pylori eradication include: patients with an H. pylori-associated ulcer, persistent symptoms despite appropriate therapy for H. pylori, patients with H. pylori-associated MALT lymphoma, history of resection for early gastric cancer, and patients planning to resume chronic NSAID therapy. Documentation of eradication is NOT required for all patients who have tested positive.

Ms. Jones is a first time mother who brings in her 5-day-old son Tommy for his first scheduled newborn visit. Upon reviewing Tommy's hospital records you note that mom's pregnancy was uncomplicated and that Tommy was born full term via spontaneous vaginal delivery and his birth weight was average for gestational age at 3000 grams. Per your discussion with mom, she has been breastfeeding successfully with good technique every two to three hours on demand since delivery. Tommy's physical exam is normal and largely unchanged from the hospital; he is at the 70th percentile for height and head circumference, but his weight has decreased to 2750 grams. Mom is very concerned by this weight loss and wants to know what she should do. You should recommend: A. Continuing breast feeding with same frequency and return to clinic in 5-7 days B. Hospitalizing Tommy until he shows adequate weight gain for his age C. Increasing the frequency of breastfeeding until Tommy starts gaining weight D. Supplementing breast milk with 1 to 2 ounces of baby formula until Tommy starts gaining weight E. Refer Ms. Jones and Tommy to a lactation consultant

The correct answer is A. It is normal for newborns to lose up to 10% of their birth weight in the first week. Their weight should be back to their birth weight by two weeks of age. Tommy has only lost 8.3% of his birth weight, and breastfeeding has been well established. There is no need to increase the frequency of breastfeeding or to supplement with formula if breastfeeding is successfully established, as it is here. Referral to a lactation consultant is frequently a helpful intervention for parents who are encountering challenges with breastfeeding, but this would not be indicated here

Mr. and Mrs. Thomas are first-time parents who have brought their 5-day-old newborn son into your office for his first visit. Mom's pregnancy, baby birth, and today's physical exam are all normal. Both parents are sharing the duties of feeding, cleaning, holding, and watching their son without any problems. When Mrs. Thomas steps out of the room to use the restroom, Mr. Thomas asks to speak with you privately. He wants to know if you can prescribe some antidepressant medication for his wife because since giving birth she hasn't been the same. She has been having mood swings multiple times a day, he has found her crying in bed at night after putting their son in his crib, and she gets very irritated and yells at her parents any time they try to help with the baby. What would be the most appropriate advice to give Mr. Thomas? A. This is known as the "postpartum blues"; is quite common for new moms; and is very likely to self-resolve. B. This behavior is very common and a mild antidepressant can help most women get through it. C. The behavior is uncommon at five days postpartum and mental health counseling should be considered. D. This behavior sounds dangerous and hospitalization is in order. E. You should challenge Mr. Thomas on whether he is helping enough with child care.

The correct answer is A. Mood swings, crying spells, and irritability are common (~50%) in the first week after giving birth. Symptoms that persist after two weeks are more serious and may represent postpartum depression. There is no role for therapy (C), medications (B), or hospitalization (D) in the management of postpartum blues. Patient understanding, support and reassurance are appropriate. Questioning Mr. Thomas' support for his wife (E) could weaken your relationship with him, as the postpartum blues are not caused by a lack of support for the new mother. He has expressed concern about his wife, and he may feel unfairly singled out if you question his behavior.

Ms. Michaels is an 80-year-old female with a past medical history of shingles. She comes to your office accompanied by her daughter Jennifer who reports that her mother is forgetting things. Jennifer explains that her mother will ask the same question several times throughout the day. Ms. Michaels also gets confused easily and is more passive than usual. Her memory problem was noticed two years ago after she forgot to pay her bills on multiple occasions. Jennifer now pays her mother's bills and cleans and cooks for her. Ms. Michaels' vital signs are temperature of 99.2 Fahrenheit, blood pressure of 118/70 mmHg, heart rate of 80 beats/minute, and respiratory rate of 12 breaths/minute. Her physical exam is significant for bilateral osteoarthritis hand deformities. She does not have a tremor, nor jerky uncontrolled movements. She is not on any chronic medications. She has no smoking history and does not drink alcohol. CT head shows mild atrophy of the hippocampus. Her MMSE is 20. The patient's diagnosis is most likely associated with? A. Alzheimer disease B. Huntington disease C. Lewy bodies D. Prion protein E. Vascular disease SUBMIT

The correct answer is A. Ms. Michaels' symptoms and CT results are consistent with Alzheimer disease (A). The hippocampus is a critical area for memory retention. The presenting symptoms of Huntington disease (B) are choreiform movements (random, jerky and uncontrollable movements), which are absent in Ms. Michaels, so this option is less likely. Lewy bodies (C) are abnormal aggregates of protein that develop inside nerve cells in Parkinson disease and often presents with Parkisonian symptoms, fluctuations in alertness and attention (delirium), and visual hallucinations. Ms. Michaels does not have a tremor so it is less likely that she has dementia caused by dementia with Lewy bodies. Prion proteins (D) are seen in transmissible spongiform encephalopathies, such as Creutzfeldt-Jakob disease, and are exceedingly rare, so it is less likely in this case. Patients with vascular dementia (E) usually have cardiovascular risks such as a hypertension, hyperlipidemia, or tobacco use. Ms. Michaels does not have any of these risk factors, so it is less likely.

A 67-year-old man comes to the clinic for a health maintenance visit. His past medical history is significant for chronic allergic rhinitis, severe chronic obstructive pulmonary disease (COPD), osteoporosis, psoriasis, atrial fibrillation, and benign prostatic hypertrophy. Vital signs show his temperature is 36.8C (98.2F), pulse is 76 beats/minute, respiratory rate is 12 breaths/minute, and blood pressure is 118/70 mmHg. His weight is 129.2 kg (285 lbs) and his body mass index (BMI) is 41. Which of his co-morbidities is most likely to be associated with his BMI? A. Atrial fibrillation B. Benign prostatic hypertrophy C. Chronic allergic rhinitis D. Osteoporosis E. Psoriasis

The correct answer is A. Obese patients are at a significantly increased risk for developing atrial fibrillation. This is thought to be related to increased left-atrial volume. Weight loss may reduce the burden of atrial fibrillation in obese patients. Also, obesity has been associated with COPD and asthma, although the nature of the association has not been fully elucidated. Additional related health risks include atherosclerotic cardiovascular disease (including stroke, coronary artery disease, and peripheral vascular disease) and heart failure. Psoriasis, chronic allergic rhinitis, and benign prostatic hypertrophy are less likely to be associated with obesity. Osteoporosis is associated with low BMI, not obesity.

A 52-year-old woman comes to the clinic to discuss weight loss. Her medical history is significant for obesity; her body mass index (BMI) is 41; hypertension; hyperlipidemia; and obstructive sleep apnea. She knows that losing weight will help her hypertension and hyperlipidemia, but she doesn't feel like these things bother her. Her only other concern is fatigue; she doesn't use her continuous positive airway pressure (CPAP) machine, because she doesn't like the mask. What additional information can you provide her to help motivate her weight loss? A. Her obstructive sleep apnea may improve with weight loss. B. Her risk of cardiovascular disease is similar to that of a woman with a normal BMI. C. Obesity is mainly a cosmetic issue. D. Surgery should be considered before diet and exercise.

The correct answer is A. Obesity is associated with a number of medical co-morbidities affecting multiple organ systems, including the cardiovascular (atherosclerotic cardiovascular disease, atrial fibrillation, heart failure, venous thromboembolism), pulmonary (obstructive sleep apnea, obesity hypoventilation syndrome), gastrointestinal (gastroesophageal reflux disease, cholelithiasis, hepatic steatosis), endocrine (diabetes), and renal (chronic kidney disease). In patients with medical co-morbidities related to obesity, weight loss is a cornerstone of therapy. In this woman, improvement in hypertension, hyperlipidemia, and sleep apnea can be expected with weight loss. Therefore, counseling regarding lifestyle interventions, weight loss medications, and possibly surgery if non-surgical interventions are unsuccessful, is warranted to manage obesity-related co-morbidities.

A 24-year-old female presents to the clinic complaining of nausea and headache for the last week. She denies any fever, changes in bowel movements, or sinus symptoms. She is sexually active with one partner, and admits to not being consistent with her birth control pills. She does not remember the date of her last menstrual period, and reports a history of irregular menstrual cycles. She has not taken a home pregnancy test. Her vital signs reveal a blood pressure of 124/76 mmHg, a pulse of 78/min, respirations of 20/min, temperature of 98.1o F (taken orally), and an oxygen saturation of 98% on room air. The first most appropriate step in management is to: A. Obtain a urine hCG. B. Admit the patient to the hospital for monitoring. C. Obtain a serial serum hCG. D. Perform a pelvic exam to test for sexually transmitted infections. E. Ask the patient to return to the clinic after she has taken a home pregnancy test, and prescribe Zofran 8mg to control her nausea.

The correct answer is A. Obtaining a urine hCG is the most appropriate first step in the case of a female of childbearing age, whose history includes the patient unable to remember her LMP date and is inconsistent with her contraception method. Admission to the hospital is unnecessary, as her vital signs are normal and her symptoms not life-threatening. Serial serum hCG tests are used when monitoring fetal viability, not to make the diagnosis of pregnancy. The patient does not complain of symptoms that would indicate a sexually transmitted infection, and if pregnancy can be confirmed in the office, it would make no sense to send her home to take a home pregnancy test.

A 35-year-old female with a history of headaches presents to your office with a headache that is not responsive to 400 mg Ibuprofen TID or 500 mg Tylenol TID. Upon gathering further history you find that in the past her headaches were controlled with these medications but recently have been worse with this same treatment. She complains of 4-5 headaches weekly that are often present on awakening. Which of the following is the best treatment for her disorder? A. Discontinue use of Ibuprofen and Tylenol B. Prescribe a calcium-channel blocker for headache prevention C. Prescribe a different NSAID to take for acute headaches D. Prescribe an opioid medication to take for acute headaches E. Prescribe sumatriptan to take as needed

The correct answer is A. The patient in this case is likely experiencing rebound headaches, due to a dependence on the ibuprofen and Tylenol. The appropriate treatment for this type of headache is discontinuation of the agents (A). Counseling must be provided that headaches may worsen before resolving over time. A calcium channel blocker (B) does not help overcome rebound headaches, but may be used for migraine prophylaxis. Prescribing a different NSAID (C) will continue her pattern of rebound headaches. Adding an opioid (D) would not resolve symptoms and might also lead to dependence. Sumatriptan (E) is a treatment for migraine headaches, not rebound headaches as this patient has.

A 23-year-old female comes to the clinic to discuss infertility. She states she was treated for an STI two years ago, and remembers receiving a ceftriaxone shot and taking a course of doxycycline. She admits to being hospitalized last year due to a severe infection with gonorrhea. She states that she and her fiance have attempted to have a baby for the past year, without success. She reports no dysmennorhea and has regular menses, but has pain during intercourse. Her last menstrual period was 4 weeks ago. Urine hCG is negative. Vital signs stable. Physical exam is unremarkable. What is the most likely cause of the patient's inability to conceive? A. Pelvic inflammatory disease B. Uterine leiomyoma C. Endometriosis D. Candidiasis E. Ovarian cyst

The correct answer is A. The patient most likely was treated for pelvic inflammatory disease twice in the past. The most common cause of PID is Neisseria gonorrhoeae and/or Chlamydia trachomatis. It is treated with ceftriaxone and doxycycline to cover both bacteria. Pelvic inflammatory disease can cause infertility due to scarring of the fallopian tubes/ tubal adhesions. Uterine leiomyomas are benign neoplasms of the female genital tract, and are usually asymptomatic. They are not treated with antibiotics. They are unlikely to cause infertility, unless they are submucosal and distort the endometrial cavity of the uterus in a way that makes it an undesirable environment for pregnancy. Endometriosis involves functional endometrial tissue outside of the uterus, and often causes cyclical pelvic pain. Less commonly it may cause painful intercourse. Treatment may include surgery or a course of oral contraceptive pills to stop ovulation, or NSAIDs to treat symptoms. It can cause infertility, however is not the most likely cause of infertility in this patient due to her history. Candidiasis is caused by an overgrowth of yeast that is found naturally in a female's genital tract. The treatment is fluconazole. Candidiasis does not fit this patient's history, and is not a common cause of infertility. Ovarian cysts present with lower abdominal and pelvic pain. It may be severe pain if the cyst has ruptured. They are not treated with antibiotics making this less likely to fit the patient's history or be the cause of this patient's infertility.

A 63-year-old male with a past medical history significant for hypertension, COPD, and long-term tobacco use is accompanied by his wife to a hospital follow-up clinic appointment. She is very concerned about her husband's recent hospitalization for a COPD exacerbation and asks what can be done to improve her husband's health. Which of the following holds the greatest long-term health benefit for this patient? A. Cessation of tobacco products B. Immunization against pneumococcus C. Prednisone taken daily D. Pulmonary rehabilitation program E. Tiotropium (Spiriva) inhaled daily SUBMIT

The correct answer is A. This is a key intervention in all patients with COPD who continue to smoke and can reduce the rate of FEV1 decline. The pneumococcal vaccine is recommended for COPD patients ≥ 65 years old or < 65 years old and for all smokers or patients with chronic lung disease. While important to prevent complications from COPD (pneumonia), it is not as fundamental as smoking cessation. Long-term monotherapy with oral corticosteroids is not recommended. Pulmonary rehabilitation may improve dyspnea, walking distance, and quality of life but does not have as much supporting evidence as tobacco cessation. Tiotropium is a medication for COPD, which may be used to decrease symptoms and/or complications, but no medication for COPD has been shown to modify long-term decline in lung function.

A 20-year-old female G1P0 at 37 weeks gestation presents to the emergency department with the concern of a headache that has not gone away, despite taking acetaminophen 3 hours ago. She reports no visual changes, but admits to feeling nauseous and having stomach pain that she attributed to having a "bad case of heartburn." She states she went to her primary care physician that morning, and her blood pressure was elevated. She states she was asked to do a 24-hour urine collection when she left the clinic, but has not been able to use the restroom. Her vitals reveal a blood pressure of 182/106, a pulse of 92, a temperature of 98.2o F, a respiration rate of 20, and a pulse oximetry of 97%. What is the diagnosis and the best course of management? A. Severe gestational hypertension - admitting the patient to the hospital, with administration of magnesium-sulfate, management of hypertension, workup for preeclampsia, followed by possible induction or cesarean section B. Pharmacological management with 1 tablet metoprolol 50mg bid C. Admitting the patient to the hospital, and giving IV fluids and IV labetalol 20 mg, followed by close monitoring until a urinalysis is obtained D. Pharmacological management with Macrobid 100mg q12 hours x7 days E. Admitting the patient to the hospital, followed by pharmacologic management of Lopressor HCT 50/20 bid, and obtaining a urinalysis SUBMIT

The correct answer is A. This patient is exhibiting symptoms that are associated with severe gestational hypertension and possible preeclampsia. Admission to the hospital for monitoring and evaluation of the fetus, work up for preeclampsia (i.e. CBC, CMP, 24-hour urine collection), and administration of magnesium-sulfate (prophylactic therapy for preeclampsia) are appropriate. A diagnosis of preeclampsia involves a systolic blood pressure of > 140 mmHg, and/or a diastolic blood pressure of > 90 mmHg, and proteinuria of 0.3 g or more in a 24-hour urine collection. When the blood pressure is >160/100, typically IV medications such as labetalol or hydralazine are needed to mitigate the risk of CV complications such as stroke. If necessary, the patient with severe preeclampsia may undergo induction for delivery or cesarean section due to the possible risks involved to both the mother and fetus.

A 16-year-old male patient presents with dull, aching pain in his left scrotum. He says that the dull aching pain is more prominent upon standing. Which of the following the most likely diagnosis? A. Varicocele B. Epididymitis C. Hydrocele D. Testicular torsion E. Anatomical variant

The correct answer is A. Varicocele is a collection of dilated and tortuous veins in the pampiniform plexus surrounding the spermatic cord in the scrotum. Patients may be asymptomatic in some cases. As much as 15% of cases are seen in adolescents.

Which of the following is a comorbid condition that needs to be controlled in order to improve asthma symptoms? A. High BMI B. Insomnia C. Atopic dermatitis D. Anxiety E. Anemia

The correct answer is A. When a patient experiences difficulty with asthma control, the physician must consider and address comorbid conditions known to effect asthma control. These comorbid conditions include: gastroesophageal reflux (GERD), chronic sinusitis/uncontrolled allergic rhinitis, stress/depression, obstructive sleep apnea, and being overweight or obese. Successful treatment of these conditions often results in improved control of the patient's asthma symptoms.

After you leave the room, Dr. Medel takes a moment to ask you a question. "If Ms. Rios had had an elevated fasting or one-hour glucose measurement, we would want to follow up with a three-hour glucose tolerance test (3hr GTT). This test includes a fasting glucose, a 100g glucose load, then a glucose at one, two, and three hours post-prandial. What determines when a 3hr GTT is abnormal, allowing you to diagnose Gestational Diabetes?" Choose the single best answer. A. One of the four glucose measurements exceed the cutoffs. B. Two or more of the four glucose measurements exceed the cutoffs. C. Three or more of the four glucose measurements exceed the cutoffs. D. All four of the four glucose measurements exceed the cutoffs. SUBMIT

The correct answer is B.

How useful are Tinel's and Phalen's when testing for carpal tunnel syndrome? Choose the single best answer. A. Very useful given their high sensitivities B. Somewhat useful with lower sensitivities and higher specificities C. Virtually useless given their low sensitivities and specificities D. Somewhat useful with lower specificities and higher sensitivities SUBMIT

The correct answer is B.

She then asks you, "Which lifestyle modification will decrease blood pressure the most?" Choose the single best answer. A. Weight loss of 10 kg or less B. DASH eating plan C. Dietary sodium reduction D. Physical activity E. Moderation of alcohol consumption F. Dietary potassium

The correct answer is B. Answer B has been selected. However, if a person loses more than about 11 kg of weight, weight loss may result in a greater decrease in blood pressure.

Which of the following are within the role of a health care provider in the care of a victim of domestic violence? Select all that apply. A. Acknowledge the abuse and its impact on victim's health. B. Stop the abuse. C. Support your patient's decisions. D. Solve the victim's problems. E. Get the victim to leave the perpetrator. F. Provide lengthy counseling. G. Develop a safety plan. H. Help protect safety of victims. I. Facilitate access to resources. J. Consider the safety of other persons in the home.

The correct answers are A, C, H, I, J.

Ms. Anderson is a 60-year-old woman who comes in to clinic as a walk in appointment. She is tearful and is carrying a box of tissues in her hand. She says she doesn't know why but she has been very sad of late. She reports trouble falling asleep and staying asleep. She used to be the head of her Bridge club, but quit two weeks ago and doesn't feel like going out anymore. She also says she has lost interest in walking her dog, and now just allows him to use the doggie door to let himself out. She also says she feels weak and fatigued and no longer has the energy to do her gardening or shopping. She spends most of her day on the sofa crying while watching TV. She also reports a greatly diminished appetite. She denies suicidal or homicidal ideation, but she does have a history of a previous suicide attempt following her divorce seven years ago for which she was hospitalized. A recent CBC, CMP, CXR, TSH, U/A and CT of the head were all within normal limits. How long do the above symptoms need to be present in order to make the diagnosis of Major Depressive Disorder? A. One week B. Two weeks C. Four weeks D. Five weeks E. Eight weeks

The correct answer is B. Depressed mood or anhedonia and at least five of the following eight criteria must have been present for two weeks or longer. (Mneumonic = SIG E CAPS)

A 60-year-old male with a past medical history of chronic gout, depression, and Stage 1 hypertension presents to your office for a follow-up visit. He has been attempting to reduce his blood pressure with behavioral changes, but has had difficulty maintaining the changes. Today, his vital signs are blood pressure 144/90 mmHg, pulse 78 beats/min, respirations 12/min, temperature 98.7 F. His recent basic metabolic panel was completely normal. As you consider starting a medication for his hypertension, which of the following medications is most likely to cause an adverse event in this patient? A. Amlodipine B. Hydrochlorothiazide C. Lisinopril D. Losartan E. Metoprolol

The correct answer is B. Hydrochlorothiazide (HCTZ) (B) can cause hyperuricemia and therefore should be used with caution in patients with gout. Metoprolol (E) is not a first-line choice for the management of blood pressure, but there is no particular reason to expect this patient to experience an adverse drug event due to a beta-blocker. While all of the other medications listed are appropriate first-line anti-hypertensives, many clinicians would select one of the other options over HCTZ for this patient given his history of gout.

Which of the following statements is incorrect? Choose the single best answer. A. A positive anterior drawer sign is indicative of an anterior cruciate ligament injury. B. Lachman test assesses the stability of the posterior cruciate ligament. C. The valgus stress test assesses the medial collateral ligament. D. An abnormal varus stress test could indicate rupture of the lateral collateral ligament. E. The McMurray test assesses the medial and lateral menisci.

The correct answer is B. Lachman test Assesses the stability of the anterior cruciate ligament (and not the posterior cruciate ligament) This test is performed with the patient lying supine with the injured knee raised and slightly flexed to 30 degrees. The distal femur is stabilized by the physician with one hand, while the proximal tibia is held by the other hand. Force is applied to move the tibia anteriorly. The test is considered positive if there is excessive motion of the tibia.

A 42-year-old woman presents with nausea, vomiting and RUQ pain radiating to her back for one day. She reports a history of similar episodes but none have ever lasted for this long of a time period. Her vital signs are HR of 108, BP of 145/90, RR of 20, O2 saturation of 98% and temperature of 100.8F. Her labs reveal the following: WBC = 14.0 x 103/mL AST = 55 U/L ALT = 60 U/L Amylase = 70 U/L Lipase = 7 U/L What is the most likely diagnosis? A. Biliary colic B. Acute cholecystitis C. Acute pancreatitis D. Duodenal ulcer E. Hepatitis

The correct answer is B. Acute cholecystitis has similar pathophysiology to biliary colic and often results from a bile stone that is lodged in the biliary tree. However, there are additional pathophysiologic reasons for acute cholecystitis. The symptoms are similar to biliary colic but typically last longer than 4-6 hours and may be more severe. Symptoms may include fever and elevated WBC. Biliary colic typically lasts 4-6 hours or less, radiates under right shoulder blade, often is accompanied by nausea, vomiting and can often follow a heavy, fatty meal. The hallmark of biliary colic is the stone is still mobile and gallbladder function resumes with relief of symptoms. Acute pancreatitis is often difficult to distinguish from biliary colic but will have an increased lipase and amylase. A patient with duodenal ulcer typically has epigastric pain that is relieved by food or antacids. Hepatitis is usually distinguished by malaise, anorexia, itching and icterus or jaundice. Signs of hepatitis would include hepatomegaly and elevated transaminases.

A 27-year-old G0P0 female presents to your office with abdominal pain, fever and vaginal discharge. The discharge has an abnormal odor. She reports a new, male, sexual partner. On pelvic exam, there is cervical motion tenderness and adnexal pain during the bimanual exam. A cervical culture is positive for Chlamydia trachomatis. What risk factor for chlamyida infection is present in this patient? A. Sexual orientation B. New sexual partner C. Age D. Nulliparity E. Race

The correct answer is B. Chlamydia is the most common sexually transmitted bacterial infection in the U.S. Chlamydial infection is often asymptomatic and can result in serious complications-particularly for females. Risk factors for chlamydial infection include: age (women and men less than 24 years old and younger are at greatest risk), history of chlamydial or other sexually transmitted infection, new or multiple sex partners, inconsistent condom use, exchanging sex for drugs or money. Race is not a risk factor for the development of a sexually transmitted disease

Your 28-year-old male patient who presented with severe testicular pain has an exam consistent with a testicular torsion. Which of the following imaging studies is best for confirmation? A. X-ray B. Color Doppler US C. CT scan without contrast D. MRI E. Angiography

The correct answer is B. Color Doppler is the best imaging test among the choices above to evaluate for a possible testicular torsion. If a testicular torsion is present, intratesticular flow is either decreased or absent in comparison to a non-torsed testicle. X-ray, CT scan, MRI, and angiography are inappropriate studies to evaluate testicular torsions and the decrease in flow associated with it.

Ms. H is a 68-year-old woman with a medical history significant for obesity, type II diabetes, hypothyroidism, hypertension, and recently diagnosed hyperlipidemia. Her most-recent lipoprotein (LDL), three months ago, was 197 mg/dL. At that time, atorvastatin was initiated. Other medications include metformin, insulin glargine, amlodipine, hydrochlorthiazide, and levothyroxine. Which of the following may be contributing to her elevated LDL? A. Amlodipine B. Hydrochlorthiazide C. Insulin glargine D. Levothyroxine E. Metformin

The correct answer is B. Dyslipidemia is typically familial, although there are secondary causes of hyperlipidemia that clinicians should be aware of. These include type II diabetes, cholestatic or obstrutive liver disease, nephrotic syndrome, acute hepatitis, alcohol, and medications including hydrochlorthiazide, beta blockers, oral contraceptives, and protease inhibitors. In addition to a possible familial cause of hyperlipidemia, this woman's type II diabetes, hypothyroidsm, and hydrochlorthiazide all may be contributing to dyslipidemia.

A 29-year-old female presents to your office complaining of very heavy periods. These started about six months ago. She reports that her periods were always normal until six months ago when she started passing a significant number of clots each day. Her periods occur in a regular pattern but usually last more than eight days. She reports having to change a super absorbent pad every two to three hours. On pelvic exam, the uterus is small, non-tender, and has uniform, smooth contour. On physical exam, you note a moderately enlarged thyroid and dry skin. What is her most likely diagnosis? A. Metrorrhagia B. Menorrhagia C. Uterine leiomyoma D. Cervical polyp

The correct answer is B. Menorrhagia is a descriptive diagnosis that has at the core of its definition, an increased length (usually ≥ 7 days) and quantify of menses. Quantity of menses is very difficult to define precisely and is only one of the terms that can mean abnormal uterine bleeding. The absolute criterion for menorrhagia is blood loss of more than 80 milliliters. Some providers try to use pad or tampon count. However there is variability in the absorption of different pads and how much blood a woman has on the pad prior to changing. Asking about clots may help, but again not easy to quantify. In fact, many women either over- or under-estimate the blood loss. A uterine leiomyoma can present as menorrhagia, but when uterine fibraoids do present with menorrhagia, they are much more likely present with an enlarged uterus. Cervical polyps more commonly present with irregular bleeding, often after intercourse. Metrorrhagia is irregular bleeding.

You are seeing a 92-year-old male in the hospital. He was admitted after a fall in which he broke his hip. It is now post-op day two from surgery to repair the fracture. His children report that he has been very confused this morning, with varying levels of alertness. Yesterday, he had been recovering well and even participated in physical therapy. Today, he does now know where he is and is at times combative with his care team. His medical history is positive only for hypertension. His medications include amlodipine 5 mg daily and morphine 4 mg IV every 4 hours as needed for pain. His vital signs are all normal today, and his general exam is unremarkable except for expected post-operative changes to his hip. He is somnolent during the examination. Today's labs include a normal CBC and basic metabolic panel. A urinalysis reveals no leukocyte esterase and is nitrite negative. What is the most likely cause of his current mental status? A. Acute stroke B. Morphine C. Residual effects of the anesthetic medications from his surgery D. Pneumonia E. Urinary tract infection

The correct answer is B. Morphine (B) and other sedating medications (such as other opioids, benzodiazepines, anticholinergics, etc.) commonly cause delirium in older patients, particularly after the cumulative effects of repeated doses. Discontinuing the pain medication may not be an option, due to his need for pain control, though reducing the dose and careful attention to the dosing frequency may help minimize his symptoms. Acute stroke (A), while sometimes the cause of acute mental status change, is unlikely in a patient with no new physical findings to suggest stroke, such as new motor weakness. Anesthetic medications (C) can cause delirium, however they would be expected to do so initially after they are given. The fact that he did not manifest delirium on post-op day one makes this an unlikely explanation. Pneumonia (D) is a common cause of delirium in hospitalized patients, but it would be expected to present with fever, elevated WBC count, lung findings, and possibly increasing oxygen requirements. This patient has none of these findings. A urinary tract infection (E), while common in post-operative patients, is less likely in a patient with a normal urinalysis.

A 51-year-old woman comes to you with acute pain and swelling of the knee. Joint fluid analysis confirms the diagnosis of acute gout. She has a past medical history of atrial fibrillation, hypothyroidism, hypertension, and prior treatment for H. pylori infection. Her current medications include losartan, warfarin, levothyroxine, and omeprazole. She is allergic to penicillin medications. Recent laboratory studies revealed normal hemoglobin and hematocrit, blood urea nitrogen and creatinine levels. Which of the following information from her history would dissuade you from initiating NSAID therapy? A. Her age B. Currently on warfarin C. Previous H. pylori infection D. Penicillin allergy E. Hypothyroidism

The correct answer is B. NSAIDs can increase the risk of gastrointestinal bleeding with prolonged use, particularly when used in combination with warfarin or other anticoagulants. While they are not absolutely contraindicated, they should be used together only with caution. The risks of NSAIDs are increased among geriatric patients, but this patient is only 51. Prior H. pylori infection and hypothyroidism have no impact on the use of NSAIDs. NSAIDs do not preferentially cause allergic reactions in patients with allergies to penicillin antibiotics.

A 13-year-old female patient comes to your office for a physical. Her mother is concerned because she complains of menstrual cramps during her period each month. You determine that menarche was earlier that year and her periods have been mostly regular since that time. The pain is in her lower abdomen and is relieved with Ibuprofen and a heat pack. She has no other medical problems and her physical exam is normal. What best describes this patient's condition? A. Premenstrual dysphoric disorder B. Primary dysmenorrhea C. Premenstrual syndrome D. Secondary dysmenorrhea

The correct answer is B. Primary dysmenorrhea is defined as the onset of painful menses without pelvic pathology, and is the most likely diagnosis in this case, given the normal history and physical. Secondary dysmenorrhea implies painful menses secondary to some pelvic pathology, by definition. Finally, this patient's symptoms are during menses, as opposed to during the other phases of the menstrual cycle

A 52-year-old female presents for her third visit this year for productive cough. She has a 34-pack year history but has weaned down to only five cigarettes per day since she began to notice a cough. With her smoking history, you are concerned about the possibility of COPD. Which of the following criteria is included in the GOLD classification for diagnosis of COPD? A. Brain natriuretic peptide >500 B. Post-bronchodilator FEV1/FVC ratio of < 70% of predicted C. Flattened diaphragm on lateral chest film D. Left ventricular function <40% E. Oxygen saturation level of < 89%

The correct answer is B. Spirometry (pulmonary function tests) is the gold standard for diagnosing COPD. If the FEV1 to FVC ratio is less than 70% of predicted (or less than the 5th percentile), then the patient has COPD. Brain natriuretic peptide levels >500 are suggestive of congestive heart failure. Flattened diaphragm on a lateral chest film may be suggestive of advanced COPD but is not diagnostic. Left ventricular function <40% is seen with systolic heart failure. Oxygen saturation levels <89% may be seen in those with COPD but is very nonspecific.

You are seeing a 52-year-old male for a full physical at the local free clinic for patients who are uninsured. He has no concerns other than intermittent headaches, which are well controlled with ibuprofen. His family history is positive only for dementia in his mother and stroke in his father. He does not smoke tobacco, drink alcohol, or use illicit drugs. His general physical exam is normal aside from his being overweight. You mention that he is due for colon cancer screening, and he responds that his older brother has been pressuring him to get a colonoscopy. Unfortunately, he is unable to afford this test. There is a local program to provide free colonoscopies to patients at high risk for colon cancer, but your patient would not qualify on the basis of his lack of risk factors. What is the most appropriate next step with this patient regarding colon cancer prevention? A. Recommend yearly abdominal x-rays, which are available at the free clinic B. Recommend annual fecal occult blood testing C. Recommend that he start saving up money for a colonoscopy, as that is the screening test with the best evidence D. Recommend yearly sigmoidoscopy E. Recommend that given his lack of symptoms or risk factors for colon cancer, he can wait until he is aged 60 to start colon cancer screening

The correct answer is B. The USPSTF recommends starting colon cancer screening at age 50 in low-risk individuals and suggests a variety of options for how to screen. Annual fecal occult blood testing (FOBT) (B) is one of the recommended screening options, and there is randomized controlled trial evidence in its support. This is a more affordable, while still effective, method of screening for some patients. Abdominal x-rays (A) are not effective in screening for colon cancer. Many clinicians have preferentially recommended colonoscopy (C) as a means of screening, and there are some potential advantages to it, including the ability to remove a polyp during the screening procedure and the 10-year screening interval. However, insisting that this is the only acceptable method presents an unnecessary barrier to many patients actually receiving screening (including this one). Flexible sigmoidoscopy (D) is a good option for colon cancer screening. There are multiple randomized trials that demonstrate its effectiveness. However, the recommended screening interval is every five years, not every year. Waiting until age 60 for starting screening (E) would be inappropriate, as starting at age 50 is the recommendation for low-risk patients, like this one.

A 52-year-old female with no past medical history presents to your office with amenorrhea. The patient states that her menstrual cycles previously occurred approximately every 28 to 34 days. However, she has not had a menstrual cycle for the last 10 months. She also endorses insomnia and intermittent dysuria. She denies any headaches, abdominal pain, constipation or diarrhea, changes in hair distribution, or easy bruising. She has lost 15 pounds since her last visit eight months ago, which she attributes to improving her diet and beginning regular exercise. Which of the following tests or pairs of tests is used to confirm menopause? A. Thyroid Stimulating Hormone (TSH) and Free T4 B. Luteinizing Hormone (LH) and Follicle Stimulating Hormone (FSH) C. Prothrombin Time (PT) and International Normalized Ratio (INR) D. Anti-Mullerian hormone E. Testosterone and Dehydroepiandrosterone sulfate (DHEA-S) SUBMIT

The correct answer is B. The definition of menopause is no period after 12 months so she is not there yet. Elevated FSH and LH levels are sometimes used to confirm menopause. During menopause, ovarian granulosa cells produce less inhibin, thereby affecting the negative feedback loop of FSH and LH secretion from the pituitary gland. TSH and free T4 are used in testing thyroid function. One might be concerned about thyroid function if there was unexplained weight loss along with other positive symptoms in the ROS, but those are not present in this case. Anti-Mullerian hormone is a test that assesses ovarian function. Testosterone and DHEA-S do not influence menopause.

What is the most appropriate treatment at this time? Choose the single best answer. Find clue cells on vaginal wet mount with WBC A. Ciprofloxacin 500 mg twice a day for 1 week B. Clotrimazole cream, 1 vaginal applicator full per vagina twice per day for 1 week C. Metronidazole, 500 mg twice per day for 1 week D. Bactrim DS, 1 tablet twice per day for 1 week

The correct answer is C metronidazole Clotrimazole would be appropriate if yeast had been detected. She does not have evidence of another bacterial infection, such as a urinary tract infection, so the other antibiotic choices are not correct. Ciprofloxacin (and any quinolone antibiotic) is also contraindicated in pregnancy due to its potential effects on the fetal bone growth plates.

Mr. York is a 44-year-old man presenting for evaluation of an eyelid lesion. He noticed the lesion about one year ago. There is no associated itching, discharge, or other bothersome symptoms. Which of the following is the next-best step in the management of the eyelid lesion? Eyelid lesion - xanthelasma A. Low potency topical corticosteroid B. Measurement of serum cholesterol levels C. Measurement of serum uric acid levels D. No further management E. Skin biopsy

The correct answer is B. The eyelid lesion is most likely a xanthelasma associated with hyperlipidemia. Xanthelasma are cholesterol-filled, soft, yellow plaques which may appear on the medial aspect of the eyelid or on extensor surfaces. They are benign findings, and removal is typically only pursued for cosmetic reasons. Despite the benign nature of the lesion itself, measurement of serum cholesterol levels should be pursued to identify hyperlipidemia in patients with xanthelasma.

The correct answers are A, B. Recommended therapies: Relative rest (A) If Mr. Chen does not rest from some of his repetitive overhead activity, he runs the risk of worsening impingement, tendinopathy and ultimately a tear of the rotator cuff. Pain medication as needed in topical and/or oral form (B) Mr. Chen reports occasional acetaminophen use-you could trial him on a maximal dose of acetaminophen (noting no known liver disease) of 1000mg four times daily as needed and tolerated; or, alternatively trial him on a NSAID such as ibuprofen.

The correct answer is B. The muscles that make up the rotator cuff are the supraspinatus, infraspinatus, teres minor and subscapularis muscles. The teres major, deltoid and rhomboid minor are not part of the rotator cuff.

A 68-year-old male with a past medical history significant for hypertension and diabetes presents to your office with a three month history of headaches. He describes a pain that has occasionally awoken him from sleep and is often worse in the morning. He denies any weakness or changes in vision. His exam shows 4+ reflexes in the right upper and lower extremities but is otherwise normal. What is the best next step in management for this patient? A. Initiate treatment with sumatriptan B. Neuroimaging C. Reassurance D. Referral to counselor for relaxation exercises E. Sleep study

The correct answer is B. This patient exhibits several red flags that are of concern for intracranial pathology: age over 50, headaches awakening him from sleep and abnormal reflexes. Because of these concerning features, it is important to perform neuroimaging without delay (B). Thus, reassurance (C) is inappropriate. The symptoms are not typical of migraine, so initiating sumatriptan (A) would not be appropriate. Referral to counseling (D) may be appropriate for tension-type headaches, but would be inappropriate until you had ruled out more serious pathology in this case. Symptoms described do not raise concerns for sleep apnea, so a sleep study (E) would not be indicated.

A 61-year-old male with a history significant for COPD presents to the emergency department for shortness of breath. Upon exam you see a thin male with perspiration on his forehead. He is having a difficult time answering questions because "he just can't catch his breath." You order an arterial blood gas on the patient. The results are pH 7.22 (7.34-7.44) PaCO2 81 mmHg (35-45 mmHg) PaO2 55 mmHg (75-100 mmHg) . What is the next best step in diagnosis or management? A. Administer Rocephin B. Begin noninvasive mechanical ventilation C. Immunize against influenza D. Provide nicotine replacement patches E. Repeat the test in two hours

The correct answer is B. This patient is in respiratory distress as evidenced by his dyspnea, physical exam, and ABG. He has a respiratory acidosis as his pH is low and he is retaining CO2. His PaO2 is also low. Initially you would want to improve his respiratory status with some type of mechanical ventilation such as nasal cannula, facemask, bipap, or even intubation if indicated by worsening of respiratory status such as decreasing oxygen saturation, confusion and drowsiness. After stabilizing the patient, you might consider giving an antibiotic such as Rocephin if he was diagnosed with pneumonia or possibly a COPD exacerbation. Immunizing against the flu and providing tobacco cessation counseling are always good steps in patients with COPD, but would not be the initial step in this scenario. You might want to repeat the ABG after the patient is placed on mechanical ventilation to ensure improvement in his oxygenation status.

A 71-year-old woman presents to the emergency department with acute headache and numbness of the left arm for the past six hours. Past history includes hypertension and diabetes. She takes amlodipine, chlorthalidone, metformin, and rosuvastatin, but ran out of all her medications recently. Her vitals are: Heart rate: 80 beats/minute Blood pressure: 205/110 mmHg Respirations: 16 per minute Body Mass Index: 30 kg/m2 An EKG reveals normal sinus rhythm with a rate of 82 and no ischemic changes. Which of the following is the most likely diagnosis A. Transient ischemic attack B. Hypertensive emergency C. Embolic stroke D. Brain tumor E. Hypoglycemia

The correct answer is B. This patient is presenting with an acute hypertensive emergency. While there is no exact threshold at which hypertensive emergency is universally defined, typically the patient has to experience severely elevated blood pressures (eg. >185/110 mmHg) and an acute elevation above the patient's baseline. Both features are true in this case. Additionally the patient must be experiencing acute end-organ damage such as neurologic symptoms, cardiac injury or acute kidney injury. A TIA is possible in this patient, though the context of her running out of her medications and experiencing headache makes hypertensive emergency more likely. An embolic stroke is not as likely a diagnosis given that she is not in atrial fibrillation. It is still a possibility however, as the patient could have paroxysmal atrial fibrillation and only be in sinus rhythm at the time of presentation. Given her markedly elevated blood pressure, a hypertensive emergency is a more likely explanation for her current presentation. Hypoglycemia is unlikely in a diabetic who has run out of her medications. A brain tumor is a possible but not a likely explanation in this patient.

This 28-year-old patient has not been seen by a physician for several years. You feel like she really needs a complete exam, but she is only scheduled for an acute visit. You contemplate the best action. Choose the single best answer. The best option is indicated below. Your selections are indicated by the shaded boxes. A. Talk to the nurse and reschedule the next couple of patients to accommodate the need for Ms. Bell's complete physical exam. B. Tell Ms. Bell that her problem is much more complex than anticipated and ask her to reschedule and return for a complete physical exam in the near future. C. Address the most urgent issue raised during the visit today with a focused exam and screening, and schedule a follow up in the near future, including a plan for a complete exam. D. Order all the recommended preventive screening tests today and schedule a follow up visit in the near future to review all the results and completion of the physical exam. E. A complete exam at age 28 is not indicated. Address the concerns raised by the patient for today and schedule follow ups as needed.

The correct answer is C.

Ms. Brady, a 78-year-old female prevents to your office after six months for follow-up. Her interval history is significant for a TIA three months ago. Today her Mini-Mental State Examination (MMSE) is 19. You note that six months ago her MMSE was 22, and nine months ago it was 26. Physical exam shows temperature of 98.8 Fahrenheit, blood pressure of 167/95 mmHg, heart rate of 76 beats/minute, respiratory rate of 14 breaths/minute, and BMI of 23. Chest: regular rate and rhythm, no murmurs; lungs: clear to auscultation; neuro: weakness in the right upper extremity; abdomen: soft, non-tender. She takes atorvastatin and aspirin. Lab studies show Hgb A1c: 5, TSH: 3, B12: 500 pg/mL. Which of the following is the most important recommendation to prevent further disability in this patient? A. Order a mammogram B. Start hydrochlorothiazide C. Start metformin D. Start vegetarian diet E. Weight loss

The correct answer is B. To prevent additional TIAs and stroke, her blood pressure needs to be controlled. Hydrochlorothiazide (B) is a reasonable first-line medication. A 78-year-old patient with cerebrovascular disease and evidence of moderate dementia (MMSE of 19) is likely to have a high risk of mortality within the next five years. In this setting, the benefit of screening for breast cancer (A) would be expected to be very low. Her HgbA1C is in the normal range so metformin (C) would not be an appropriate choice. A vegetarian diet (D) would have little impact on limiting dementia. Her BMI is 23, so weight loss (E) is not necessary at this point, and would likely not help with memory issues.

You are seeing a 55-year-old female patient for follow-up who you met two weeks ago at her physical exam. Her review of systems and physical exam were only significant for fatigue. Her blood work shows iron deficiency anemia. She is post-menopausal x 2 years with no significant past medical history or family history but a 10 pack-year smoking history. Prior to establishing with you she hadn't seen a doctor since she was 18 years old. Which of the following would be the most appropriate next step in diagnosis? A. Prescribe oral iron supplementation and pursue no further workup if her fatigue improves B. Refer her for a colonoscopy C. Order a hemoglobin electrophoresis D. Order a pelvic ultrasound E. Recommend fecal occult blood testing now and every year moving forwards

The correct answer is B. Treating the patient with iron (A) is appropriate to address her iron deficiency, however a further workup must be conducted to find an explanation for this diagnosis. Gastrointestinal blood losses are the most common cause of iron deficiency in non-menstruating patients. Thus, referral for a colonoscopy (B), and likely also an upper endoscopy, is the appropriate next step in the management of this patient. Hemoglobin electrophoresis (C) would be indicated when hemoglobinopathies are suspected and not for iron deficiency which suggests blood loss. A pelvic ultrasound (D) in absence GYN symptoms in a post-menopausal woman would not be indicated. Annual fecal occult blood testing (E) is an appropriate approach to screening for colon cancer. However, this patient is not a candidate for routine screening as she actually has symptoms that could be explained by colon cancer. In her case, we have suspicion that she has gastrointestinal bleeding, and she should undergo a diagnostic colonoscopy.

A 32-year-old female presents at your office for a preconception health visit. She is a G2P2, both deliveries were vaginal. Her first child was born with a neural tube defect. According to the USPSTF, what dosage of folate should this patient take daily before she gets pregnant? A. 1 mg B. 4 mg C. 8 mg D. 400 mcg E. 800 mcg

The correct answer is B. US Preventative Services Task Force (USPSTF) recommends that all women planning or capable of becoming pregnant take a 400 to 800 mcg daily supplement of folate to prevent neural tube defects. The recommendation increases to 1 mg daily in patients with diabetes and epilepsy. In patients who have previously had a child with a neural tube defect, the recommendation increases to 4 mg daily. The 8 mg option is not included in the guidelines.

A 53-year-old female with a past medical history of diet-controlled hypertension presents to the office with a two-month history of worsening hot flashes. Her menstrual cycles are regular, occurring every 30 to 32 days, but they have gradually lessened in duration, now lasting four to five days instead of the previous six to seven days. Vital signs and physical exam are normal. Which one of the following treatments is most likely to improve the patient's symptoms? A. Black cohosh B. Oral estrogen C. Venlafaxine D. Gabapentin E. Yoga

The correct answer is B. While all of the listed items have been touted for treatment of hot flashes, estrogen is clearly the most effective, but should be used with progesterone if a woman still has a uterus. There is evidence that venlafaxine and gabapentin help some. There is no convincing evidence that black cohosh or other herbal medications help, and there is no evidence that yoga, mindfulness or exercise help.

A 23-year-old epileptic male presents to the emergency department after a generalized tonic-clonic seizure. You notice that the patient is holding his right shoulder and that his arm is adducted and internally rotated. What is the best imaging modality for this type of injury? A. MRI B. X-ray C. Ultrasound D. CT scan E. PET scan

The correct answer is B. X-ray is the indicated imaging in the setting of acute injury and suspected fracture or dislocation. MRI is used to evaluate possible rotator cuff tears and other soft tissue etiology. Ultrasound can be used to evaluate soft tissue structures. It's desirable due to its low-cost and because it exposes the patient to no radiation. CT can be used in the setting of complicated fracture, suspected tumor, or when MRI is contraindicated. PET scans indicate functional activity of organs such as the brain, heart and lung and are mostly used in the oncologic setting.

A 21-year-old female with no significant past medical history experienced an inversion-type injury to her right ankle while playing soccer a day prior to presentation. She remembers immediate pain and swelling but was able to weight bear and limp off the field. She has noticed some significant swelling which is mostly still present. She has been icing the ankle since the injury as her coach recommended. Pain is still present near the lateral malleolus. Physical exam reveals that the lateral right ankle is edematous with purplish bruising, bilateral pulses are good and the patient had good sensation and motor function in both feet about equally. Palpation of the posterior edge of the lateral malleolus elicits significant pain from the patient. What is the next best step in the management of this patient? A. Reassurance B. Immobilize with cast or splint C. X-ray imaging of right ankle D. Emergent fasciotomy E. Rest, ice, Ibuprofen, compression and elevation and re-evaluate in one week

The correct answer is C. Based on the Ottawa Ankle Rules, tenderness of the lower 6 cm of the posterior lateral malleolus may predict fracture and justifies X-ray imaging of the ankle. Tenderness of the lower anterior lateral malleolus, on the other hand, is very common in ligamentous injury such that x-rays are not warranted. Because of the possibility that a fracture exists, the other treatment options are not correct in this scenario.

A 56-year-old male presents to your office for a six-month follow-up visit for DM Type 2 diagnosed at his previous visit with a HbA1C of 7.0%. At that visit, you discussed dietary modification including eliminating carbohydrates like rice from his diet. You also emphasized the importance of exercise and recommended he join a gym. On repeat testing of HbA1C today, you find that his HbA1C has increased to 7.5%. What might you do differently when counseling the patient at this visit? A. Suggest he get a personal trainer. B. Recommend he follow a more strict dietary protocol including elimination of meat from his diet. C. Find out from the patient whether barriers exist to his adherence to your previous suggestions. D. Tell him you are disappointed that he was not able to adhere to your previous recommendations.

The correct answer is C. Before counseling a patient it is critical to assess barriers to adhering to the plan developed together. It is important to ask patients for understanding of the plan and whether they feel confident that they can follow the plan. Suggesting a personal trainer may be unrealistic for financial or time reasons. Eliminating meat may not appeal to him and telling him about your disappointment could make him less likely to tell you what is going on, or even making a follow-up visit.

A 65-year-old truck driver presents to your office complaining of right calf redness and swelling. He reports that he has had these symptoms off and on for some time but noticed that the redness and swelling on the right has worsened over the past few days and he also noticed fevers and chills. His PMH is significant for DM Type 2, COPD, and heart failure. He has a 25-pack-year smoking history. His physical exam shows a temperature of 101.1, 2+ pitting edema of his calves bilaterally, and dry, flaking skin over both calves. His right foot shows interdigital maceration and he has redness, warmth, and tenderness just inferior to the knee. When measuring the circumference of his right leg, you note that it is 12 cm compared to the left, which has a circumference of 10 cm. What is the most appropriate next step in this patient's care? The best option is indicated below. Your selections are indicated by the shaded boxes. A. Initiate treatment with Low molecular weight heparin. B. Order a lower extremity Doppler. C. Initiate treatment with antibiotics. D. Check a D-dimer.

The correct answer is C. In light of the fever with redness and tenderness of the exam, the extra swelling is most likely due to cellulitis. There is no added benefit of treating with heparin and no particular reason to rule out a DVT with the Doppler or D-dimer

A 55-year-old man presents to your office complaining of pain and stiffness in his knees. He used to play basketball three times a week with his friends but stopped about seven months ago because he was finding it difficult to keep up. His PMH is significant for hypertension, hyperlipidemia, and obesity. His BMI is 32. On exam, he is afebrile. There is no erythema or effusion of his knee joints but there is crepitus on movement of each joint. His range of motion of the knees is full. His x-rays show narrowed joint spaces and subchondral calcification. What is the most appropriate initial counseling for this patient? A. Instruct him to see an orthopedist for steroid injections. B. Refer him for knee replacement surgery. C. Develop a weight loss plan with the patient. D. Prescribe opioids for pain relief.

The correct answer is C. Obesity is an important cause of mortality and morbidity. Knee pain is common in obese patients and losing weight can be helpful. Steroid injections generally help in osteoarthritis when there is joint inflammation, and this patient does not have signs of an effusion. There is no indication at this point for knee replacement surgery or opioids. Opioid pain medications are generally only indicated for severe acute pain and in this case his pain is likely to be chronic.

According to current immunization guidelines, what immunizations should you offer to Mr. Dennison? Select all that apply. A. Influenza vaccine B. Tetanus, diphtheria, acellular pertussis vaccine (Tdap) C. Haemophilus influenzae type b conjugate vaccine (Hib) D. Pneumococcal polysaccharide (valent-23) vaccine (PPSV) E. Pneumococcal conjugate (valent-13) vaccine (PCV13)

The correct answers are A, B, D. Appropriately Updating Vaccinations According to the CDC Immunization guidelines, Mr. Dennison should receive the influenza vaccine (A) and tetanus, diphtheria, acellular pertussis vaccine (B) if due. Due to his asthma (chronic lung disease), he should also be offered the pneumococcal polysaccharide (PPSV) vaccine (D).

A 41-year-old male with no significant past medical history is brought to the Emergency Department after falling to the ground in the middle of a pick-up basketball game with friends. He did not lose consciousness nor hit his head when he fell. As he landed on the ball of his foot after having taken a shot, he recalls hearing a popping sound followed by immediate pain in the posterior right ankle. On physical exam, the posterior right ankle is edematous and palpation is tender. He is unable to plantarflex his right foot. What is the most likely diagnosis of his current condition? A. Ankle ligament sprain B. Calcaneal fracture C. Achilles tendon rupture D. Ankle tendonitis E. Ankle arthritis

The correct answer is C. This is a classical description of an acute rupture of the achilles tendon. Middle-aged males are more commonly affected than other groups. The mechanism does not describe inversion injury making an ankle ligament sprain less likely and there is no direct trauma making fracture less likely. There is no history of overuse or chronicity making arthritis and tendonitis less likely.

A 55-year-old male with a family history of melanoma presents to the clinic for evaluation of a skin lesion on his back which appeared three months ago. His wife first alerted him to it, hasn't noticed it change and he has not noticed any symptoms associated with it. Physical examination reveals a 7 mm uniformly black macule that is symmetrically round with sharply demarcated borders on his upper back near the right shoulder. Which of the following characteristics would most justify it being biopsied today? A. Borders B. Color C. Diameter D. Location E. Symmetry

The correct answer is C. Using the ABCDE mnemonic, this nevus is not Asymmetrical, does not have irregular Borders, does not display Color variation and he does not describe any Evolution or change or symptoms. The only positive is that its Diameter is > 6 mm, which is considered a red flag supporting biopsy. Location is not considered a predictive factor for melanoma.

A 65-year-old truck driver presents to your office complaining of right calf pain and swelling. He has recently returned from a four-day cross-country trip after which he had onset of his current symptoms. The patient reports that the week prior to his four-day trip, he was mostly in bed recovering from a bout of the flu. His PMH is significant for DM type 2 and COPD. He has a 25-pack-year smoking history. He denies having any chest pain or SOB. His physical exam shows 2+ pitting edema of his right leg. When measuring the circumference of his right leg, you note that it is 14 cm compared to the left, which has a circumference of 10 cm. What is the most appropriate next step in diagnosis? A. D-dimer B. MRI C. Lower extremity Doppler D. CT angiography

The correct answer is C. Venous Doppler has the best sensitivity and specificity for diagnosing a DVT. D-dimer is best used when there is a low pre-test probability of a DVT. If it is negative then it is fairly certain that there is not a DVT. A positive test does not diagnose a DVT (poor specificity) but indicates the need for further testing. MRI and CT angiography are very expensive, carry risks with contrast, and do not add to the care unless an invasive intervention is being contemplated.

Althea has no known drug allergies and her weight is 44 lbs (20 kg). What is the most appropriate first-line choice of antibiotics for management of her strep pharyngitis? Choose the single best answer. A. Erythromycin Ethylsuccinate 300 mg po twice daily for ten days (30 to 50 mg/kg/day day in 2 to 4 divided doses) B. Amoxicillin Suspension 25 mg/kg po twice daily for 10 days C. Penicillin V 250 mg PO three times daily for 10 days D. Penicillin G benzathine (Bicillin L-A) 600,000 units IM single dose (1.2 mil units IM for adults or children greater than 27 kg). E. Cephalexin 20 mg/kg/dose twice daily for 10 days SUBMIT

The correct answer is C. (A) is incorrect as macrolides are reserved for use in penicillin allergy. (B) is incorrect as amoxicillin's broader spectrum increases the risk of contributing toward antibiotic resistance. (D) is incorrect as Penicillin G should be reserved for patients in whom there is concern they will not complete the course of treatment. (E) is incorrect as cephalosporin's broad spectrum contributes toward antibiotic resistance.

A 36-year-old female, G3P2 at 21 weeks gestation, returns to clinic for an ultrasound. She currently smokes a half pack of cigarettes per day. Her last two children were delivered by cesarean section. She reports no vaginal bleeding, no urinary symptoms, and no fluid leaking. She states her baby moves "all the time." Her vitals reveal a blood pressure of 130/74 mmHg, a temperature of 98.3o F, a pulse of 82, a respiration rate of 18, and a pulse oximetry of 98%. The ultrasound reveals no fetal abnormalities, but the location of the placenta is partially covering the cervical os. The diagnosis and treatment plan is: A. Placenta previa with immediate cesaerean section B. Placenta previa with admittance to the hospital for fetal and maternal monitoring C. Placenta previa with subsequent ultrasound surveillance to monitor for any progression or resolution D. Placenta previa with no additional education or counseling given to mom on risk of bleeding during pregnancy

The correct answer is C. Answer 'C' is the best answer, given the patient's history, symptoms, and ultrasound findings. Placenta previa is a condition in which the placenta is attached at a position that gives it the potential to cover the cervical os. A pregnant woman with placenta previa will usually present after 24 weeks gestation with painless vaginal bleeding. If detected early in gestation, the condition has a higher chance of resolving without intervention. Without resolution, there is a risk for excessive bleeding at or just before delivery, and cesarean section is usually warranted. Prior to term, if a woman is diagnosed with placenta previa and is asymptomatic, she can return home with instructions to seek immediate medical care if she experiences any bleeding. When found on routine ultrasound screening, these women may require more frequent ultrasound surveillance to see of the previa resolves on its own or not which will affect delivery planning as the due date approaches.

What is the best way to begin a discussion with Marissa and her mother about her weight? Choose the single best answer. A. "Marissa, you are obese according to your BMI and that puts you at risk for developing diabetes and high cholesterol. We need to help you diet so you can lose weight." B. "Marissa, I want to talk to you about your weight. Would you say you are overweight, underweight or the correct weight?" C. "Marissa, I was looking over your chart and I am concerned that you are at an unhealthy weight and I would like to discuss it further. Is that okay with you?" D. "Mrs. Payne, I see that you are overweight, have you thought about how your bad habits affect your daughter?"

The correct answer is C. Answer C asks permission to discuss with Marissa and gives information. It isn't open ended, but allows Marissa to let you know if she is willing to talk. Answer A is very blunt and to the point, however many patients will be put off by that approach. Answer B asks a very specific question. It is helpful to know how patients consider themselves. There are some people who are normal weight that would consider themselves either over or under weight. A person's cultural background can also influence the answer to this question. This would likely be a good follow up question, but might seem out of the blue with the current conversation. Answer D directs the blame for Marissa on her mom and also directs the question away from Marissa.

An otherwise healthy 57-year-old G2P2 female presents to your office with vaginal bleeding that began one week ago. She began her menses at age 13 and had regular menstrual cycles until the age of 49. She denies any tobacco or alcohol use. Further review of systems is negative. You perform a physical exam, including a vaginal exam, pap smear, and bimanual exam. Which of the following is the most appropriate next step? A. Reassurance B. Trial of oral contraceptives C. Transvaginal ultrasound D. Endometrial ablation E. Referral for hysterectomy

The correct answer is C. Any postmenopausal bleeding needs to be investigated; reassurance alone is inappropriate in this case. A transvaginal ultrasound is the most cost-effective initial test in women with abnormal uterine bleeding at low-risk for endometrial cancer. It is highly sensitive (96%) for the detection of endometrial cancer. Endometrial biopsy is considered the gold standard for evaluation of post-menopausal bleeding and is up to 99% sensitive. Oral contraceptives, endometrial ablation, and hysterectomy are treatment modalities that would not be appropriate until an actual diagnosis is made.

You are seeing a 72-year-old female with a recent diagnosis of diastolic heart failure (HFpEF) with an ejection fraction of 60% on a recent echo. She was given a diuretic in the ER last week and told to follow up with you. She complains of mild new dyspnea on exertion, orthopnea and lower extremity edema. On exam, her vitals include a blood pressure of 142/86 mmHg, pulse of 84 beats/minute, respirations of 16/minute and oxygen saturation of 98% on room air. Pulmonary exam reveals mild bibasilar crackles, cardiac exam reveals a regular rate and rhythm with no murmurs, and her extremities have 1+ edema to the lower shins. Of the following, which would be the most appropriate choice of medication for this patient? A. Amlodipine B. Digoxin C. Metoprolol succinate D. Hydrochlorothiazide E. Spironolactone

The correct answer is C. Beta blockers are part of the first-line treatment for diastolic heart failure. Treatment guidelines for diastolic CHF include the use of beta-blockers and ace inhibitors, similar to systolic CHF. There is no role for digoxin or potassium-sparing diuretics in diastolic failure. HCTZ may help with BP control (important in diastolic CHF) but does not improve the ventricular filling as the chronotropic effects of beta-blockers do.

Which of the following are in your differential for vaginal bleeding for this patient? Select all that apply. Endometrial bleeding in post-menopausal woman A. Cervical polyp B. Endometrial hyperplasia C. Hormone-producing ovarian tumors D. Normal bleeding with hormone replacement and no further work up warranted E. Endometrial cancer F. Proliferative endometrium

The correct answers are A, B, C, E, F.

What factors increase a patient's risk for completed suicide? Select all that apply. A. Having served in the military. B. Male sex. C. Older age. D. Living in poverty. E. Having a previous suicide attempt.

The correct answers are A, B, C, E.

A 40-year-old male presents to the clinic with dry cough and wheezing for the past two days. He states that his symptoms began two days ago with a headache for which he took aspirin. He denies fever, but does report some continued shortness of breath. He is a smoker but only smokes 1-2 cigarettes a day for about six months. Physical exam is negative except for bilateral wheezing and erythema on the face. What is the most likely diagnosis? A. COPD B. Foreign body aspiration C. Pneumonia D. Asthma E. Pulmonary embolism

The correct answer is D. 21% of adults who have asthma have aspirin-induced asthma and should avoid aspirin and NSAIDS. Even though the patient is a smoker, he is less likely to have a COPD exacerbation because he has only smoked for six months. The patient's duration of symptoms (two days) and reporting no fever lead away from the diagnosis of pneumonia. Also, the clinical history gives you very little reason to suspect foreign body aspiration or pulmonary embolism. The diagnosis of asthma could be confirmed with pulmonary function testing.

A 35-year-old man presents to clinic with acute onset of abdominal pain. On abdominal exam, you flex the patient's right hip to 90 degrees and take his right ankle in your right hand and with your left hand externally then internally rotate his hip by moving the knee back and forth. The patient denies any abdominal pain with this movement. What physical exam sign did you just perform and what disease is it ruling out? A. Psoas sign to rule out appendicitis B. Psoas sign to rule out cholecystitis C. Obturator sign to rule out appendicitis D. Murphy's sign to rule out appendicitis E. Obturator sign to rule out cholecystitis

The correct answer is C. During an abdominal exam, it is important to rule out other causes of abdominal pain. Appendicitis is another cause of acute abdominal pain and can be ruled out with a number of signs. One of these signs is the Obturator sign and the technique is discussed in the question stem. Another sign to rule out appendicitis is the psoas sign. This sign includes passive extension of patient's thigh as they lie on their side with their knees extend, or asking the patient to actively flex their thigh and hip. Pain with movement is often indicative of appendicitis due to inflammation and irritation of the psoas muscle. Murphy's sign is to identify cholecystitis. The technique involves having the patent breathe out slowly and completely, then gently placing your hands under the right costal margin. The patient is then instructed to take a deep breath in while you palpate for a hardened mass and determine if you elicit any significant tenderness.

A 24-year-old G1P0 female at 38 weeks gestation presents to ED complaining of strong lower abdominal contractions that are 10 minutes apart for the last hour. Subsequent cervical examination demonstrates that she is 2cm dilated. FHT are 140 and NST is non-reactive with early decelerations. What is the most appropriate management of this patient? A. Augment labor with oxytocin (Pitocin) B. C-section C. Expectant management D. Immediate vacuum delivery E. Rupture membranes to increase labor

The correct answer is C. Expectant management is the most appropriate management at this time. ** Patient is currently not in active labor. Active labor is defined as cervical dilation > 6 cm in the presence of contractions. Contractions are normally 3-5 minutes apart. Fetus is not in distress at this time. Early decelerations are indicative of head compression and are not especially concerning. It is not generally advised to induce patients prior to 39 weeks if there are no complications and fetus is not in distress. C-section would not be indicated in this case because both mother and baby are stable and not in distress. Immediate vacuum delivery is not indicated in this case. Vacuum delivery is indicated only during the second stage of labor, which is the beginning of pushing to the delivery of the baby.

A 32-year-old female at 33 weeks and 5 days gestation (G2P1) presents to the clinic with headache and RUQ abdominal pain. Blood pressure is 172/121 mmHg on examination while seated. No visual changes noted. Edema is present in the hands, bilaterally. Urine dipstick demonstrated 4+ protein. FHT are 117. Which of the following is the most appropriate next step in the management of this patient? A. Twice-weekly non-stress testing B. Daily aspirin C. Expedited delivery of the premature fetus D. Lisinopril E. Strict bed rest until 37 weeks

The correct answer is C. Expedited delivery of the fetus is the best treatment for severe preeclampsia. (Note: severe preeclampsia is not necessarily an indication for a C-section, however.) Twice-weekly non-stress testing combined with assessment of amniotic fluid volume (or twice-weekly biophysical profiles) is part of the management of preeclampsia without severe features. This would not be appropriate in this case. Aspirin is used in the second and third trimesters among patients at risk of developing preeclampsia and those with pregestational hypertension. This patient has severe preeclampsia and needs expedited delivery, not preventive therapy. Lisinopril is contraindicated in pregnancy. In the third trimester, it can cause oligohydramnios. Strict bed rest is not encouraged in the management of preeclampsia.

Ms. Martinez, a 74-year-old female, is brought to your clinic by her husband. He is concerned because his wife, who used to take pride in keeping the house in good shape, has stopped cleaning and caring for her appearance. She recently went shopping and left the stove on. She frequently misplaces her car keys and checkbook. Ms. Martinez reports trouble concentrating and does not have much energy. She has a poor appetite and no longer enjoys knitting. Her Mini-Mental State Examination (MMSE) is 28/30. Physical exam: General appearance: well-nourished, appears stated age; HEENT: pupils equal, round and reactive to light, moist mucus membranes; chest: normal S1 and S2, no murmurs; lungs: clear to auscultation bilaterally; abdomen: soft, non-tender, non-distended; extremities: no cyanosis, no clubbing; neurologic: no focal deficits; psychiatric: flat affect. Labs: TSH: 2; WBC: 6.0; HgbA1c: 4.5. Which of the following is the most likely diagnosis? A. Delirium B. Dementia C. Depression D. Hypoglycemia E. Normal bereavement

The correct answer is C. Her exam findings are normal with the exception of her affect. This suggests a mood disorder such as depression (C). As there is nothing in the history to suggest a recent loss, bereavement (E) is less likely Ms. Martinez has a normal MMSE, so (B) is incorrect. Hypoglycemia (D) is less likely as Ms. Martinez does not have diabetes and is not on glucose-lowering medications. History is not suggestive of delirium (A), which would present with more fluctuations in symptoms and alteration in cognition.

A 27-year old female, G2P1, returns to the clinic for her second prenatal visit. Her labs reveal that her blood type is A Rh-. She states she has done research online, and is concerned that this pregnancy will result in her baby dying if it has a different blood type than her own. To reassure the patient, you explain that her team of health care providers will: A. perform a cesarean section to prevent fetal demise B. treat the baby with Rhogam within the first 72 hours after delivery to prevent hemolytic anemia of the newborn C. treat the mother with Rhogam when she is at 28-weeks gestation to prevent development of antibodies against Rh+ antigens, and if it is determined the neonate is Rh+, the mother will receive a second dose of Rhogam postpartum. D. treat the mother with penicillin during labor to prevent transmission of gram-positive bacteria E. treat the mother with Rhogam when she is at 28-weeks gestation to prevent thalassemia in the newborn, and if it is determined the neonate is Rh+ the mother will receive a second dose of Rhogam postpartum

The correct answer is C. If a pregnant woman is Rh-, she is given Rhogam (anti-Rh antibodies) in order to prevent sensitization of her immune system to the fetus' Rh+ antigens. Rhogam is given at 28 weeks gestation, within 72 hours post delivery, or with any episode of vaginal or intrauterine bleeding. If untreated, the antibody containing blood of a sensitized Rh- mother may cross the placenta and cause hemolytic anemia in her Rh+ fetus. Answer C gives the correct time frame for administering Rhogam, as well as the reason for giving Rhogam.

Samantha and Joe bring in their newly adopted 5-year-old son, Matthew, to your office with the chief concern of sore throat, without cough, that began two days ago. He was born in the United States and was neglected by his biological parents. Samantha and Joe believe Matthew lived in an old building and ate mostly non-nutritious meals. They are unsure if his immunizations are up to date. He is behind on routine immunizations. Upon physical exam, he is playful and interactive. His vital signs are: Temperature: 99.0 F; heart rate: 80; respiratory rate: 19, blood pressure: 95/63 mmHg. He has tender anterior cervical lymphadenopathy, tonsillar exudates and palatal petechiae. Which pairing contains the most appropriate next steps in the management of this patient? A. Empiric antibiotic treatment and recommend immunizations today B. Empiric antibiotic treatment and do NOT recommend immunizations today C. Rapid strep test and recommend immunizations today D. Rapid strep test and do NOT recommend immunizations today E. Reassurance and follow-up in one week

The correct answer is C. Immunizations (C) that are due (or overdue) should be given unless there is a contraindication. Having a minor acute illness with or without a fever is not a contraindication. (A) is incorrect as empiric antibiotics are not appropriate for strep throat as many viral illnesses present similarly. (B) is incorrect as this child with a minor illness should be offered immunizations that are due and empiric antibiotics are not appropriate. (D) is incorrect as this child with a minor illness should be offered immunizations that are due. (E) is incorrect as this child with a minor illness should be offered immunizations that are due and his Centor score warrants further evaluation for strep throat.

A 35-year-old man presents to clinic with acute onset of constant right upper quadrant abdominal pain. Additionally, he complains of ongoing nausea and vomiting. He denies any past abdominal surgeries or chronic diseases and his only medication is a multivitamin. His AUDIT-10 was positive. What lab(s) would you most likely expect to be abnormal to confirm your diagnosis of acute alcoholic hepatitis? A. GFR and INR B. CRP and alkaline phosphatase C. AST, ALT and total bilirubin D. AST, ALT, INR and WBC E. Lipase and amylase

The correct answer is C. It is expected that someone with acute alcoholic hepatitis would have an elevated AST, ALT and total bilirubin. GFR (glomerular filtration rate) is a marker of kidney function and is often not abnormal in early hepatitis. INR is a marker of synthetic functioning of the liver and would be elevated in end-stage liver disease. WBC is often elevated in cholecystitis and infectious processes. Elevation of lipase and amylase is associated with pancreatitis.

Which of the following statements is most accurate about placenta previa? Choose the single best answer. A. Bleeding from a placenta previa usually occurs early in pregnancy. B. Placenta previa is more common in nulliparous than multiparous women. C. Placenta previas found at this point in pregnancy (approximately 21 weeks at the time of the ultrasound) usually resolve on their own. D. Partial or marginal placenta previa are less likely to resolve than complete placenta previa. E. Women with placenta previa must be hospitalized due to their risk of bleeding.

The correct answer is C. Placenta previas found at this point in pregnancy usually resolve on their own (C).

A 13-year-old female patient comes to your office for a physical. Her mother is concerned because she complains of menstrual cramps during her period each month. You determine that menarche was earlier that year and her periods have been mostly regular since that time. The pain is in her lower abdomen and is relieved with Ibuprofen and a heat pack. She has no other medical problems and her physical exam is normal. What is the most appropriate treatment for this patient? A. Start OCPs B. Start an SSRI C. Continue Ibuprofen and heat packs, return if worsening D. Start Danazol

The correct answer is C. Primary dysmenorrhea is associated with increasing amounts of prostaglandins. Therefore, NSAIDs are often the first line of treatment, rather than the other therapies mentioned, of which OCPs would be the most common first/second-line treatment in the appropriate candidate. SSRIs may help with comorbidities, but is not a primary treatment for primary dysmenorrhea. Danazol is an androgenic medication with progesterone effects. It lowers estrogen and inhibits ovulation. It is primarily used for conditions like endometriosis or premenstrual syndrome, which this patient doesn't have, and its multiple androgenic side effects, including weight gain, suppressing high-density lipids, and hirsutism, limit its desirability among patients.

A 25-year-old female presents with vaginal bleeding and cramping. Her last normal menstrual period was six weeks ago. The patient's vital signs are stable. On speculum exam of the vagina, there is no bleeding from the cervix. A quantitative beta-human chorionic gonadotropin (beta-hCG) level is 1492 mIU/ml. The patient is sent home and told to return to the office in 48 hours. Her beta-hCG on the return visit is 3000 mIU/ml. What is the appropriate next step in the management of this patient? A. Surgery for ectopic pregnancy B. Methotrexate for ectopic pregnancy C. Ultrasound to confirm intrauterine pregnancy D. Dilation and curettage for non-viable pregnancy E. Progesterone level to confirm pregnancy

The correct answer is C. Quantitative serum beta-hCG levels rise at a predictable rate, making it a useful tool to help distinguish between an intrauterine pregnancy and an ectopic pregnancy. During the first 6-7 weeks of gestation, beta-hCG levels double every 48 hours. With a beta-hCG level of 1500-1800 mIU/ml, a transvaginal ultrasound will be able to detect an intrauterine pregnancy. A transabdominal ultrasound will be able to detect pregnancy when beta-hCG levels > 5000 mIU/ml. An ectopic pregnancy will have beta-hCG levels lower than normal and levels aren't expected to double during early gestation. In the above case, the beta-hCG levels are doubling every 48 hours and a transvaginal ultrasound would be used to confirm intrauterine pregnancy. Progesterone levels, while still useful at some points in the workup of first trimester vaginal bleeding, are not diagnostically useful in the setting of an hCG being available, and increasing.

A 9-year-old male who moved to the United States from Canada one year ago presents to your clinic with a three day history of fever, sore throat, and muscle aches. He lives at home with his parents who practice strict vegetarianism, and he does not eat meat of any kind. He is up to date on all vaccinations, and review of his medical records indicates that he has had several episodes of otitis media in the last five years. He was hospitalized twice in the past, once for a splenic rupture sustained in a MVA, and the second time was two months ago when he was diagnosed with Kawasaki disease for which he was put on daily aspirin. You suspect a viral infection and order the appropriate tests. Which of the following is the greatest risk factor for complications of your suspected diagnosis in this patient? A. Recent immigrant B. Splenectomy C. Long-term aspirin therapy D. Age < 10 E. Decreased Vitamin B-12

The correct answer is C. Risk factors for complications of Influenza are children < 5 years, increased in children less than 2 years of age, COPD, conditions that affect the ability to handle respiratory secretions/increased risk of aspiration, CHD, metabolic conditions, Chronic Renal Disease, Immunosuppression, and Long term aspirin therapy. A splenectomy increases his risk of infection by encapsulated bacteria (strep, h. flu etc.), but in this case he probably has influenza so it is not a risk to him now.

A 23-year-old female patient comes to your office complaining of bothersome symptoms the week before her period each month. She reports that she has significant breast tenderness, is very irritable, and eats significantly more than she does at any other time during the month. Her coworkers notice the difference in her mood and it is beginning to affect her interactions with them. The symptoms resolve after her period. She has no other medical problems or significant past medical history. Physical exam is normal. What is the most effective treatment for this patient's condition? A. Danazol B. OCPs C. Continuous SSRI treatment D. Regular exercise

The correct answer is C. Selective serotonin reuptake inhibitors during menses are an effective treatment of premenstrual syndrome, especially if severe or mood symptoms predominate. There are three effective regimens for SSRI use. One regimen is continuous daily treatment. Another is intermittent treatment, which is just as effective as daily treatment for decreasing both psychologic and physical symptoms. There are two types of intermittent treatment. One method is to start therapy 14 days prior to menses (luteal phase of cycle) and continue until menses starts. The second method is to start on the first day a woman has symptoms and continue until the start of menses or three days later. Many randomized trials have used fluoxetine and sertraline. Venlafaxine can be used as well. Lower doses are effective. If one medication does not work, another in the same class should be tried prior to considering the treatment a failure. Follow-up should occur after two to four cycles. Intermittent treatment is associated with fewer side effects and lower cost. Danazol is an androgenic medication with progesterone effects. It lowers estrogen and inhibits ovulation. However, its multiple androgenic side effects, including weight gain, suppressing high density lipids, and hirsutism, limit its desirability among patients. GnRH agonists, such as leuprolide, are effective at treating premenstrual syndrome through ovulation inhibition. However, their anti-estrogen effects, including hot flashes and vaginal dryness, make these less popular. Oral contraceptives are effective treatment for dysmenorrhea, anovulation, and in some cases menorrhagia. While not always effective for premenstrual syndrome, they are a good place to start. It would be appropriate to try this in a woman also needing birth control. The most favorable pill is the formulation containing ethinyl estradiol and drospirenone. One study demonstrates potential improved effectiveness by decreasing the placebo pills to four days from seven.

What is the best test you should do at this point? Choose the single best answer. A. CT scan of the chest B. Chest x-ray C. Spirometry, both without and with bronchodilation D. Methacholine challenge test E. Ventilation/perfusion scan

The correct answer is C. Spirometry (C) findings will help us rule in or rule out several of the diagnoses on the differential; the other tests are not indicated at this time.

Which of the following choices correctly identifies where an infant's weight should be at two weeks of age? Choose the single best answer. A. 10% above birth weight B. 5% above birth weight C. At birth weight D. 2% to 3% below birth weight E. Depends entirely on whether the baby is bottle or breast fed

The correct answer is C. Term newborns typically lose weight in the first several days after birth. Weight loss of up to 10% of the birth weight is considered normal. Most newborns regain their birth weight in the first 10 to 14 days, whether fed by breast or bottle. Once gaining weight, neonates typically gain approximately 30 gm (1 oz) per day until three months of age.

A 41-year-old male presents to clinic with left shoulder pain after a fall where he caught himself with his left arm. To assess the injury the patient's arms were placed in 30 degrees of horizontal forward flexion and his shoulders were abducted to 90 degrees. The patient attempted to resist downward pressure and was unable. This test assesses the integrity of the: A. infraspinatus muscle B. Teres minor muscle C. Supraspinatus muscle D. Biceps tendon E. Deltoid muscle

The correct answer is C. The Empty Can Test assesses for pathology of the supraspinatus muscle. For the Empty Can Test, the patient's arms are placed in approximately 30-degrees of horizontal adduction with the shoulders abducted to 90-degrees. The patient attempts to resist downward overpressure applied by the person performing the test. If the patient cannot resist the pressure, that is a positive result and can indicate pathology of the supraspinatus muscle.

You are seeing a 42-year-old established female patient in the office for her yearly physical. She has a past medical history of hypertension, and she has no significant family history. She is a teacher at a local high school, and she has three school-aged children. She does not drink alcohol or use illicit drugs, but she has smoked a pack and a half of tobacco for the past 20 years. She would like you to order a colonoscopy because a friend of hers was just diagnosed with colon cancer and has passed away from it. What combination of types of cancer screening all receive an A or B recommendation from the USPSTF for this patient? A. Cervical cancer screening, lung cancer screening, and breast cancer screening B. Cervical cancer screening and breast cancer screening C. Cervical cancer screening only D. Cervical cancer screening, breast cancer screening, and colon cancer screening E. Breast cancer screening and lung cancer screening

The correct answer is C. The USPSTF gives an A recommendation to cervical cancer screening for all women with a cervix from age 21 to age 65. This is the only cancer screening that receives an A or B recommendation for a 42-year-old female. A, B, D, and E all include types of cancer screening that are recommended in some patients, but not in a patient at this person's age. The USPSTF gives breast cancer screening a C recommendation for women between ages 40 and 49, meaning physicians should engage in shared decision making about mammography with patients in this age group. The USPSTF gives a B recommendation to lung cancer screening (with low dose CT) among patients with at least a 30 pack-year tobacco history (like this patient), but only for ages 55 through 80. This 42-year-old is too young to commence screening, though you should certainly recommend tobacco cessation. The USPSTF gives an A recommendation to colon cancer screening for low-risk patients between ages 50 and 75. Having a friend die of colon cancer understandably increases this patient's anxiety about this disease, but does not increase her risk.

A 26-year-old male professional football player comes to the clinic with the concern of hair loss. On examination, the scalp is scaly and erythematous, and certain regions are purulent. There are several circular spots where the hair follicles are no longer present. KOH of skin shows hyphae. What is the most appropriate next step in the management of this patient? A. Punch biopsy of lesion B. Topical griseofulvin C. Oral griseofulvin D. Oral prednisone E. Topical prednisone

The correct answer is C. The description given in this scenario is suggestive of tinea capitis. Tinea capitis requires oral antifungals, rather than topical treatments. Topical therapies do not penetrate the infected hair shaft. Prednisone is not the treatment of choice for this condition. Biopsy is not necessary due to the typical appearance of this condition.

Mr. Rodriguez is a 32-year-old male who presents with three months of post-prandial epigastric burning. He reports no relief with antacid therapy. He has no allergies and no significant past medical history. He denies any hematemesis, odynophagia, dysphagia, hoarseness, or sore throat. Physical exam is unremarkable other than epigastric tenderness. H. pylori IgG serology is positive. Which of the following treatment regimens will most likely be successful in treating Mr. Rodriquez? Choose the single best answer. A. Omeprazole daily for 8 weeks B. Omeprazole twice daily for 4 weeks C. Omeprazole twice daily, clarithromycin 500mg twice daily, and amoxicillin 1g twice daily D. Omeprazole daily, clarithromycin 500mg twice daily, and tetracycline 500mg three times daily E. Omeprazole twice daily, tetracycline 500mg three times daily, and metronidazole 500mg twice daily SUBMIT

The correct answer is C. The eradication of H. pylori required triple or quadruple therapy. Options include: twice daily dosing of a proton pump inhibitor (PPI) plus amoxicillin plus clarithromycin; a PPI once or twice daily (OR ranitidine twice daily) plus metronidazole, tetracycline and bismuth salicylate four times daily; or, for penicillin-allergic patients, twice daily dosing of a PPI plus clarithromycin plus metronidazole.

A 22-year-old male comes to the office for a physical before he graduates from college. He wants to know which cancer is most common in males in his age range. A. Pancreatic cancer B. Skin cancer C. Testicular cancer D. Gastric cancer E. Follicle cancer

The correct answer is C. The median age of diagnosis is 33 years of age. Pancreatic cancer occurs more commonly in adults. Skin cancer can occur in any age group especially those with extensive sun exposure. Gastric cancer occurs most commonly in men over 40.

14-year-old Jennifer is brought to clinic by her mother who is concerned about ADHD. She states Jennifer lacks motivation, regularly does not clean her room, and has to be reminded to do chores around the house. She also is concerned Jennifer does not listen to her and avoids doing homework. Which of the following would most strongly support additional evaluation to assess for ADHD? A. A diagnosis of major depression B. A report card with good grades C. A teacher's report of inattentive behavior D. Fidgety behavior in the car when driving long distances E. Symptom onset with entry to high school

The correct answer is C. The presence of inattentive and/or hyperactive behavior in at least two different settings warrants additional evaluation to assess for ADHD. (A) is incorrect as the diagnosis of major depression or mood disorder with lack of motivation could adequately explain her symptoms. (B) is incorrect as good grades do not suggest impairment in the school setting. (D) is incorrect as fidgety behavior in the car without other negative impact does not constitute dysfunction in two or more settings. and (E) is incorrect as symptoms should be present prior to the age of 12 to consider the diagnosis of ADHD.

A 12-year-old male is brought to the clinic with a pruritic, red, scaly rash in the creases of his elbows. He denies new topical exposures and generally feels well. He has a history of seasonal allergies. Which of the following is the most appropriate course of action? A. Biopsy the skin lesion B. Ask the patient to return to the clinic after three days C. Prescribe topical corticosteroids D. Prescribe oral antibiotics E. Prescribe oral corticosteroids SUBMIT

The correct answer is C. This child appears to have eczema (atopic dermatitis). Persons affected by eczema tend to have pruritic lesions which can appear in patches of dry, erythematous and at times excoriated skin most commonly in a flexural distribution. Chronic inflammation and scratching can result in lichenification and cracking of the skin. Acute treatment of choice is topical corticosteroids. Potency should be adjusted based on severity of presentation and symptoms. Ointments will be more effective than creams and lotions. Patients should be counseled to keep skin moist by using emollients twice a day and after bathing and washing and treated for allergies with antihistamines and environmental control as possible. In addition, addressing stress can reduce recurrences.

A 67-year-old female with an 80 pack year smoking history presents to an urgent care for worsening shortness of breath. She also reports her chronic cough is more frequent and is producing greenish mucous throughout the day. She is using her inhaler every two hours and has been taking an antibiotic left over from a previous sinus infection. On exam there is a whistling noise when she exhales. What other treatment should you consider at this time? A. Digoxin B. Furosemide C. Prednisone D. Propranolol E. Theophylline

The correct answer is C. This patient meets the criteria for a COPD exacerbation as she has increased dyspnea, increased sputum volume and increased sputum purulence. Inhaled bronchodilators (albuterol) and oral glucocorticosteroids such as prednisone are effective treatments with or without an antibiotic. Digoxin is commonly used for patients with congestive heart failure or atrial fibrillation. Furosemide is a diuretic used for a number of things including volume overload. Propranolol is a beta-blocker used for hypertension and coronary artery disease. Theophylline is an oral methylxanthine that antagonizes adenosine receptors and increases cAMP. It can be used in asthma and COPD but would not be the initial choice of drug in this case.

You are seeing a 32-year-old female in the family medicine ambulatory practice who presents with concern of fatigue. She finds herself simply exhausted at various points of her day, particularly when she is sitting and having to focus on something. When she is busy and physically active, she notices the fatigue less. She reports no recent weight change, skin changes, temperature intolerance, tremulousness, or unusual bleeding. She uses a levonorgestrel IUD and does not have menses on a monthly basis. Her past medical history is positive for a history of seasonal allergies that are well-controlled with loratadine. She does not smoke, drink or use illicit substances. She works as an emergency department nurse at a local hospital, alternating between evening and over-night shifts. On exam, she is well-appearing and has normal vital signs and a BMI of 24 kg/m2. She has no conjunctival pallor. Her thyroid is non-palpable. Her cardiopulmonary exam, abdominal exam, and neurologic exam are essentially normal. Her affect displays concern about her fatigue, but shows a normal range and she is able to smile and laugh at times. Of the following, which is most likely to be the cause of her fatigue? A. Side effects of her medication. B. Iron deficiency anemia. C. Sleep dysfunction due to alteration of her circadian rhythm. D. Hypothyroidism E. Chronic Fatigue Syndrome

The correct answer is C. This patient most likely is experiencing sleepiness due to her alternating shifts between evening and overnight hours, combined with caring for three children who can be assumed to be awake during daytime hours. Shift work disorder is a well-recognized cause of chronic sleepiness that is most pronounced during periods of less activity and stimulation. Iron deficiency anemia (B) is less likely in this patient who does not have monthly menses. Loratadine (A) is a non-sedating antihistamine, and it rarely causes significant fatigue. While hypothyroidism (D) is possible in this patient, it is less likely given that she shows no other signs of this disease (e.g. skin or weight changes, temperature intolerance). Chronic Fatigue Syndrome (E) typically presents with fatigue that is made significantly worse by physical activity.

A 32-year-old female comes to the ED complaining of abdominal pain. She states the pain has been bothering her for the last week, and has progressively become worse. She denies nausea, vomiting, any episodes of diarrhea, or chest pain. She states she was diagnosed with high blood pressure six months ago, and is currently taking medication for it. She is sexually active with her boyfriend, and admits to having some pain during sex. She states she does not feel comfortable telling her boyfriend that she does not want to have sex. Her last menstrual period was two weeks ago. She denies a history of sexually transmitted infections, but says that she has not questioned her significant other because "he would be upset." Vital signs reveal a temperature of 98.7, blood pressure of 142/90 mmHg, heart rate of 82, and respirations of 18. The patient is visibly uncomfortable during the physical exam, which shows a few bruises on her abdomen and lower back, which are at different stages of healing. Which of the following is an appropriate technique for screening for domestic violence? A. Request an appointment with the patient's significant other, and ask him about abusing the patient B. Ask if she had ever been sexually abused as a child C. Ask the patient, "All couples disagree at some point in time. What happens when you and your partner argue or disagree?" D. Use family and friends when an interpreter is needed E. Avoid direct questions about guns in the house or about drug/alcohol use by them or their partner SUBMIT

The correct answer is C. To screen for domestic violence in a patient should involve asking indirect questions that include the opportunity to directly assess the safety of the the patient and their environment. Asking the patient's significant other to be present when screening for domestic violence could alter the answers of the victim, as well as damage the clinician-patient relationship. Sexual abuse in the past, whether present or not, doesn't preclude the immediate issue of potential partner abuse/violence. When in need of an interpreter, it is essential to avoid using family or friends for this purpose. In the case of screening for domestic violence, impartial interpreters are needed to enable the patient to feel comfortable revealing what could be viewed as family secrets as well as to ensure the patient's safety in making such a disclosure. Sitting down at the level of the patient allows for the intimidation to be less of a factor. It is appropriate to ask about drug and alcohol use, as well as the presence of guns in the victim's home environment in order to assess safety.

You are seeing a 68-year-old woman who has presented to the emergency department with left sided facial droop, aphasia, and left sided weakness in her arm and leg. These symptoms began while she was having lunch two hours earlier. On exam, she is hypertensive and afebrile. Her neurologic exam reveals left-sided hemiparesis with expressive aphasia and left-sided hyperreflexia. When evaluating a person with a possible stroke, when might it be acceptable to administer t-PA? A. If onset of neurologic symptoms was < 8 hrs prior to presentation B. If emergent CT head shows a hemorrhagic stroke C. If onset of neurologic symptoms was < 4.5 hours prior to presentation D. If emergent CT head shows multilobar infarction E. If the patient awoke from an 8-hour sleep with stroke symptoms of uncertain duration

The correct answer is C. When assessing a patient who presents acutely with symptoms suggestive of a stroke, one possible treatment option is using recombinant tissue plasminogen activator. When used in the appropriate patient, this medication can help minimize the damage done related to suffering an acute ischemic injury to the brain. However, it is only acceptable to administer t-PA in select situations. These include when onset of neurologic symptoms has been within 4.5 hours of the onset of symptoms, emergent CT head does not show an intracranial bleed, an early acute infarct, or a brain mass. Patients with an unknown duration of stroke symptoms due to sleep should not be given t-PA.

A 24-year-old female presents to the clinic complaining of a headache and arm pain. She is accompanied by her boyfriend, who insists on staying in the room with her during the visit. When asked to describe when she noticed the pain and how it occurred, the boyfriend states that the patient hit her head on the bathroom cabinet two days ago, and fell on her side afterwards, hitting her arm. The patient nods in agreement, while looking down at the floor. The next best step in obtaining an accurate history and physical exam is to: A. Direct all questions to the boyfriend, since it appears the patient does not want to discuss her situation with you B. Accuse the boyfriend of domestic abuse and call the police to report him C. Ask the boyfriend to leave the room so you can perform the physical exam, and explain that you always do that part of the exam with just the patient, and that he may join you after that part is completed D. Give the patient referrals to neurology and orthopedics to evaluate the patient's headache and arm pain E. Prescribe acetaminophen 650mg po BID to manage the pain, and tell the patient to return in a week if symptoms have not disappeared

The correct answer is C. When domestic violence is suspected and the partner is present for the visit, the best step in obtaining an accurate history and physical from the patient is to politely but firmly ask the partner to leave the room. To aid you in asking the partner to leave, it is suitable to tell them that the next part of the exam is done with the patient independently, and the partner may rejoin you and the patient after the exam is performed. ** If the partner still insists on staying, use other tactics, such as a test performed outside of the exam room or by taking the patient to get a urine sample. Directing questions to the boyfriend instead of the patient is inappropriate. You should always direct all questions to the patient. Accusing the boyfriend of domestic abuse and calling the police could precipitate the boyfriend acting out in the office towards you or the patient. It could also damage the physician-patient relationship, since it is not proven that the boyfriend is engaging in domestic violence towards the patient. Referring the patient to neurology and orthopedics is inappropriate. Initial workup for headache and arm pain can be done in the clinic. Referrals would be appropriate after the exam and workup are done, and not as an initial step to obtain a history and physical. Prescribing acetaminophen would not be a step towards gathering a history and physical for the patient, and would be done at the end of the office visit.

A 13-year-old female comes to your clinic stating she has been having fever and chills for three days, and aching muscles for the last two days. She states she has also had a mild cough, but is not having any difficulty with breathing. She is up to date on vaccines and her only other medical history is having her tonsils and adenoids removed last year. On physical exam, you find her temperature to be 102.6 degrees F, pulse 96, and her BP to be 108/62 mmHg. She has clear rhinorrhea and her oropharynx is mildly erythematous. The rest of her physical exam is normal, and a rapid strep test in the office is negative. What is the next best step in management? A. Zanamivir B. Aspirin C. Ibuprofen D. Amantadine E. Albuterol

The correct answer is C. While this patient's presentation is strongly suspicious for Influenza infection, antivirals such as zanamivir, oseltamivir, amantadine, and rimantadine only decrease the duration of the infection by 24 hours, and are generally not efficacious outside of 48 hours from the beginning of symptoms. This patient also does not appear to require hospitalization at this time, which would be another reason to consider initiating antiviral therapy. Since this patient is presenting outside of the 48-hour window, she should be treated supportively with rest, hydration, and ibuprofen or Tylenol for pain/fever. Albuterol would be unhelpful as there is no history difficulty breathing and no wheezing on exam.

Dr. Nayar asks you, "I know Andrew is a boy, but let's consider which of the following should NOT be screened in a sexually active 17 yo adolescent female who initiated sexual activity one week ago?" Choose the single best answer. A. Chlamydia B. Gonorrhea C. Trichomonas D. Pap test for cervical cancer E. HIV

The correct answer is D.

If his epigastric pain does not resolve after salvage therapy, what is the next most appropriate course of action? Choose the single best answer. The best option is indicated below. Your selections are indicated by the shaded boxes. A. Continuation of a PPI indefinitely for symptomatic control B. Obtain a 24-hour pH probe to determine if he has GERD C. Obtain a urea breath test to verify H. pylori eradication D. Refer to a gastroenterologist for upper endoscopy/EGD with mucosal biopsy E. Prescribe quadruple therapy

The correct answer is D.

Which of the following is the LEAST appropriate step to take next regarding Ms. Rios's abnormal quad screen? Choose the single best answer. A. Offer referral for genetic counseling and amniocentesis B. Offer follow-up as needed for Ms. Rios to talk about any concerns that come up after her visit today C. Schedule ultrasound for evaluation of fetal anatomy D. Reassure her that the serum screen is not very accurate E. Double-check that the correct gestational age was recorded by the laboratory

The correct answer is D.

Which of the following is the best next step in diagnosis? Choose the single best answer. The best option is indicated below. Your selections are indicated by the shaded boxes. A. Serum creatinine B. Pulmonary angiogram C. Stress echocardiogram D. Pulmonary function testing E. Chest CT F. Chest radiography

The correct answer is D.

Which of the following patients is an appropriate candidate for a exercise stress test? A. A 58-year-old male who presents to the emergency room with constant substernal chest pressure, diaphoresis and shortness of breath B. A 44-year-old female with a BMI of 40 kg/m2 and history of asthma C. A 48-year-old female with a history of intermittent anginal episodes that have been controlled on medications. D. A 52-year-old female with new atypical chest pain with a history of elevated cholesterol, smoking and family history of coronary artery disease

The correct answer is D. An exercise stress test is useful if the pretest probability of the disease is high as in D with 3 significant risk factors. False positives increase if the pretest probability is low so the usefulness of a stress test is low in B. Likewise if the diagnosis is certain as in A then there is not as much use in getting a stress, especially in an unstable patient. Since the purpose of invasive treatment is symptom control, the patient in C has controlled symptoms and hence no need for stress testing.

According to the National Asthma Education and Program guidelines, there is more than one option for maintenance medication to treat the severity of Mr. Dennison's asthma. From the list below, which are recommended options to use as maintenance medication for Mr. Dennison's moderate persistent asthma? Select all that apply. A. Combination of a low-dose inhaled corticosteroid and a long-acting beta2 agonist inhaler B. Medium-dose inhaled corticosteroids C. High-dose inhaled corticosteroids D. Oral corticosteroids E. Leukotriene receptor antagonist F. Theophylline

The correct answers are A, B.

A 60-year-old female presents to the office complaining of increased frequency of urination and fatigue for the past several months. She denies fevers, dysuria, back pain, diarrhea and abdominal pain. She has noted some weight loss without working on diet or exercise. Her past medical history is significant for hyperlipidemia and hypertension, for which she takes simvastatin and lisinopril. She is a non-smoker and consumes one to two glasses of wine per week. Her vitals are: Heart rate: 70 beats/minute Blood pressure: 130/70 mmHg Body Mass Index: 30 kg/m2 Physical examination reveals increased pigmentation in her axilla bilaterally. Her labs are as follows: Random plasma blood glucose: 205 mg/dL Creatinine: 0.8 mg/dL TSH: 2.1 U/L. What test is needed to diagnose diabetes mellitus? A. An oral glucose tolerance test B. Fasting blood glucose C. HgbA1C D. The random blood glucose is sufficient E. Urine microalbumin

The correct answer is D. Diabetes can be diagnosed with either an HbA1C > 6.5%, a fasting plasma glucose ≥ 126 mg/dl (7.0 mmol/l), a plasma glucose ≥ 200 mg/dL (11/1 mmol/l) two hours after a 75 g glucose load, or symptoms (such as polyuria, polydipsia, unexplained weight loss) and a random plasma glucose ≥ 200 mg/dL (11.1 mmol/l). Answers (A) (B), (C) and (E) are incorrect, as the diagnosis of diabetes can be made based on random blood glucose with symptoms.

Which of the following statements is the most correct? Choose the single best answer. A. Nonsteroidal anti-inflammatory drugs (NSAIDs) are first line treatment for mild to moderate pain related to osteoarthritis because of their tolerability and low side effect profile. B. People who experience osteoarthritis pain with exercise should be discouraged from physical activity. C. Intra-articular steroid injections should be considered when there is no knee effusion, as long as there is pain related to osteoarthritis. D. Paracetamol, or acetaminophen, is effective and safe, and should be the first choice for short-term, mild to moderate pain related to osteoarthritis. E. Glucosamine has been shown to significantly decrease pain and slow progression of joint space narrowing related to osteoarthritis.

The correct answer is D. NSAIDs are 2nd line

Which of the following symptoms are most likely to be due to acute coronary syndrome? A. 23-year-old male with acute onset of difficulty breathing and hyperresonance upon lung auscultation B. 42-year-old woman with a pulsating pain in the center of her chest at night C. 35-year-old man with chest pain radiating down his left arm after falling off a ladder at work one week ago D. 59-year-old woman with palpitations that increase with exercise and are associated with nausea and vomiting E. 55-year-old woman with diffuse central chest pain that is worse when lying down

The correct answer is D. Palpitations, nausea, and vomiting are seen as prodromal symptoms of ACS in women more than men. Young men with difficulty breathing and hyperresonance are likely to have a pneumothorax. Pulsating pain is one of the types of pain, along with pleuritic and positional pain, that is less likely to be related to heart disease. Chest pain in a young man associated with trauma is less likely to be ACS.

Ms. Vasquez is a 38-year-old female with a past medical history of sarcoidosis and recently completed a six-month steroid taper. She presents to her primary care physician after two weeks of lower lumbar back pain. She does not recall any trauma but began to feel a sharp pain after bending over to pick up laundry. The pain radiates bilaterally into her anterior abdomen. She has found no relief with over the counter NSAIDs. On physical exam, she has point tenderness along her vertebrae in the L1-L2 region. There are no neurologic deficits and reflexes are intact. Which of the following is the most appropriate next step in management? A. Reassess in four weeks B. Refer to spine specialist C. Order complete blood count D. Order a plain x-ray E. Recommend conservative management

The correct answer is D. The "red flag" in this history is that of chronic steroid use, and the concomitant risk of osteoporotic vertebral fracture. A vertebral fracture is best diagnosed with a plain x-ray. A CBC will not help with the diagnosis, and referral to a spine specialist is unnecessary. Conservative management and/or reassessment in four weeks demonstrate a failure to recognize the "red flag."

A 65-year-old male with Type 2 diabetes mellitus and no other chronic health issues presents to the emergency department with altered mental status. The patient experienced no known head trauma. His vitals are: Temperature: 38.1 Celsius Heart rate: 102 beats/minute Respiratory rate: 16 breaths/minute Blood pressure: 90/74 mmHg He responds when you say his name, and he appears well nourished. His mucous membranes appear very dry. Neurological exam reveals no focal deficits. His plasma glucose is found to be 700 mg/dL. Urinalysis reveals no ketone bodies. What is the most likely diagnosis? A. Cardiac arrhythmia B. Cerebrovascular accident C. Diabetic ketoacidosis (DKA) D. Hyperosmolar hyperglycemic state (HHS) E. Thiamine deficiency

The correct answer is D. The correct answer is (D). HHS is seen typically in patients with Type 2 diabetes. It includes very high sugars > 600; ph > 6.4; dehydration; and lack of ketones in the urine and blood. Diabetic ketoacidosis is more common in Type 1 diabetes, and the patient will have ketone bodies in the urine. Thiamine deficiency can cause Korsakoff syndrome, and is typically seen in alcoholics with alcohol use disorder with severe malnutrition, however, this patient is not a known alcoholic and doesn't appear malnourished. Despite the confusion in this patient, stroke is an unlikely diagnosis in this case given the lack of focal deficits on exam. Cardiac arrhythmia can cause dizziness, but is less likely to cause prolonged altered mental status.

A 35-year-old woman has been diagnosed with Graves disease and has decided to go for treatment with radioactive iodine (RAI). She has had good relief from taking propranolol for her palpitations. What is the best way to counsel this patient about what to do after her treatment? A. Tell her to stop her propranolol a week after she gets the RAI. B. Check her TSH level two weeks after her treatment with RAI. C. Describe the symptoms of hypothyroidism to her and tell her to expect to become symptomatic within one month. D. Check her TSH levels two to three months after her treatment and treat as necessary.

The correct answer is D. The correct answer is (D). The RAI takes several months to destroy the overactive thyroid cells. There is no point in checking the TSH after only two weeks and since she may still be symptomatic from her hyperthyroidism for several months there is no reason to stop her propranolol until she becomes euthyroid. It generally takes a number of months to notice symptoms of hypothyroidism, so while it is important to let her know what to expect in the future you do not want her to have unrealistic expectations.

A 24-year-old previously healthy female has been exhibiting some new concerning symptoms over the past four months. From her history, you gather that she's been having increased diarrhea, tremors, palpitations, and fatigue over this time. Your physical exam reveals a fine tremor in the bilateral upper extremities, sweaty palms, and a smooth, mildly enlarged thyroid gland. You determine that serum testing is warranted. Of the following lab results, which would best support the diagnosis of Graves disease? A. Low TSH, normal free T4, high free T3 B. Low TSH, normal free T4, normal free T3 C. Low TSH, low free T4, low radioactive iodine uptake, low thyroglobulin D. Low TSH, high free T4; high, diffuse radioactive iodine uptake E. Low TSH, high free T4; high, nodular radioactive iodine uptake F. High TSH, high free T4

The correct answer is D. The correct answer is (D). The diagnostic algorithm of the AAFP for hyperthyroidism can be found by clicking on the following weblink: http://www.aafp.org/afp/2005/0815/p623.html. As can be intuited from the answer choices, the first step is to check if serum TSH is low or high and if serum T4 is low or high. If TSH is low and T4 is high, primary hyperthyroidism is diagnosed, and a radioactive iodine uptake scan is next.

Mr. Jones is an 82-year-old man who presents to the office for his six-month chronic disease visit. His diabetes and hypertension are controlled on his usual home medications. He reports that his wife died four weeks ago, and he is now experiencing insomnia most days of the week and fatigue and loss of energy nearly every day; reports decreased enjoyment of his activities, such as playing chess with his neighbor; and is also experiencing loss of appetite but no weight loss. He denies any suicidal ideation and has no previous suicide attempts. Mr. Jones says he often hears his wife's voice while going to bed. He says he goes to church to pray. You are trying to determine if your patient's symptoms are normal grief or if you should diagnose and treat him for Major Depressive Disorder (MDD). Which feature of Mr. Jones' case would suggest MDD rather than a normal grief reaction? A. Change in appetite B. Fatigue C. Hearing wife's voice D. Inability to experience any joy E. Insomnia

The correct answer is D. The loss of a loved one can be a traumatic event and it is normal to experience a period of grief. DSM-5 states that MDD can be diagnosed during a period of grief as long as the criteria are met. Grief can be difficult to distinguish from major depression with symptoms of sadness, fatigue, changes in appetite, sleep disruption, and decreased concentration. Since your patient is also exhibiting diminished pleasure with normally enjoyable activities, this may indicate the patient has MDD, as pervasive unhappiness and misery are rarely a part of the normal grieving process. Other features that differentiate MDD from grief include: Guilt about things other than actions taken or not taken at the time of death Thoughts of death other than feeling that he or she would be better off dead or should have died with the deceased person Morbid preoccupation with worthlessness Marked psychomotor retardation Prolonged and marked functional impairment Hallucinatory experiences other than hearing the voice of, or transiently seeing the image of, the deceased person

A 35 year old overweight woman in good health comes to the clinic for a routine physical. Which of the following screenings are recommended by the U.S. Preventative Services Task Force (USPSTF)? A. Complete blood count B. Thyroid stimulating hormone levels C. Cholesterol panel D. Blood pressure screening E. HbA1C

The correct answer is D. The only one of these that the USPSTF recommends in this age group is blood pressure screening. It is recommended to screen women with a cholesterol if there are risk factors for CHD.

A 20-year-old female who is a long-standing patient at this clinic with no significant past medical history presents with first-time onset of dysuria accompanied by frequency and urgency for the past day. She thinks that there is a strange odor to her urine but denies any hematuria. In addition, she feels mild lower abdominal discomfort but denies fevers, chills, nausea, vomiting, constipation, diarrhea, or costo-vertebral angle (CVA) pain. She reports no known allergies She has never been sexually active and has no vaginal discharge nor irritation. LMP was one week ago and was typical. She wonders if using a perfumed bubble bath for the first time a few days ago might have triggered her symptoms. On exam, her vital signs are stable; she has no CVA tenderness and mild suprapubic discomfort. Urinalysis of a mid-stream catch is within normal limits, and a pregnancy test on the same sample, performed despite her history, is negative. Which of the following reflects best management in this situation? A. Insist that a pelvic exam and cervical DNA probe be performed today. B. Await urine culture results before any treatment. C. Reassure that this is not a urinary tract infection. D. Trimethoprim/sulfamethoxazole (one tab twice daily for three days). E. Urine DNA probe

The correct answer is D. There is very good evidence that common typical symptoms of urinary tract infection (UTI) (e.g. dysuria and frequency) with the absence of vaginal symptoms are highly predictive of UTI in young women who have no systemic symptoms. The constellation of typical symptoms outweighs a normal urinalysis. Therefore, based on the symptoms presented, the most reasonable treatment option is to treat empirically with a standard antibiotic for a short course. It is not unreasonable to send urine for culture but management does not need to wait till that result is available. While it is always prudent to consider the possibility of sexually transmitted infection in this age group, based on the information presented about an established patient, it would be inappropriate to insist on a pelvic exam or await results of a urine DNA probe in the face of such strong UTI symptoms.

A 22-year-old female with no significant past medical history experienced an inversion-type injury to her right ankle while playing volleyball. The ankle quickly became edematous, but she used ice and was able bear weight on the foot. When she comes to clinic two days later, there is minimal edema, she has good motor function, and has normal sensation. She has tenderness at the anterior lower lateral malleolus but not inferiorly nor posteriorly. X-rays are not indicated. You recommend continued relative rest and also tell the patient to keep it elevated and ice it several times during the day to help with the pain and swelling. You inform the patient that immobilization and compression is good for the conservative management of her condition. What is the best compression device to use in this situation? A. Tape B. Compression stockings C. Elastic wrap D. Semi-rigid ankle support E. Solid cast QUESTION 5

The correct answer is D. This patient apparently has a ligamentous injury to her anterior talo-fibular ligament. In recommending the RICE mnemonic, a semi-rigid ankle support (like an Air Stirrup) provides protection from repeat inversion injury while allowing the patient to actively dorsi- and plantar-flex her foot, which aids recovery. Compression stockings and elastic wrap do not provide adequate support. A solid cast completely immobilizes the ankle and delays recovery, while the evidence supporting taping of the ankle is lacking

You are seeing a 55-year-old man who presents to the family medicine practice with a two-week history of daily episodes of sudden lightheadedness and palpitations. These seem to occur without provocation and last for approximately 5 minutes each time. He has not passed out during these episodes, but he has felt the need to sit down when they come. Sitting calmly and waiting seems to make them go away. He denies chest pain, diaphoresis, jaw pain, or arm pain with these episodes. He has a past medical history of gout and hypertension, for which he takes daily allopurinol and losartan. He does not smoke or drink. He is not under any recent stresses at home or work. On exam, he is in no acute distress. His vital signs are completely normal as is his cardiopulmonary exam. A recent complete blood count and TSH were normal. Which of the following is the most appropriate next step? A. Transfer him to the emergency department for admission to the hospital. B. Order an exercise stress test. C. Reassure him that his palpitations are likely benign and that he should focus on a healthy lifestyle. D. Order a 48-hour Holter monitor. E. Order an echocardiogram.

The correct answer is D. This patient is experiencing frequent palpitations that are symptomatic enough to make him lightheaded and need to sit down. Reassuring him that these episodes are benign could risk missing an important diagnosis. Given his description of his symptoms and his normal exam, episodic arrhythmia is the most likely cause of his symptoms. As such, a 48-hour Holter monitor would be the most appropriate test. He is having daily symptoms, so a longer-term loop monitor is not needed in this situation. The patient is perfectly stable and is not having symptoms or findings at the time of the visit, so referring him to the ED is not appropriate. Cardiac stress testing and echocardiography would be appropriate tests to evaluate for coronary artery disease or congestive heart failure. These are less likely explanations of his current symptoms than an arrhythmia.

What is the sensitivity of a "quad screen" in detecting Down syndrome? Choose the single best answer. A. 50% B. 60% C. 70% D. 80%

The correct answer is D. When three chemical markers (alpha fetoprotein [AFP], estriol, and hCG) are used, the test is often referred to as a "triple screen"; and when a fourth is added (inhibin-A), the test is referred to as a "quad" screen. The triple screen detects Down syndrome in about 69% of cases, and the quad screen detects Down syndrome in about 81% of cases with a false-positive rate of 5%.

A 22-year-old female comes to the clinic complaining of frequent urination. She states she has noticed some increased frequency for the past week. She denies itching or pain in the vaginal area, but reports a burning sensation during urination that began a week ago. When asked if she has noticed blood in her urine, she admits to not paying attention to its color. She states that she has some abdominal pain in her pelvic area. She admits to being inconsistent with her birth control pills, and states she is sexually active with both men and women. She does not recall the date of her last menstrual period. Vital signs show a blood pressure of 126/78 mmHg, temperature of 98.2, respirations of 18, heart rate of 82, and oxygen saturation of 97%. Physical exam reveals suprapubic tenderness. She exhibits no costovertebral angle tenderness. Urine hCG test is negative. What is the most likely diagnosis? A. Pyelonephritis B. Urosepsis C. Pregnancy D. Uncomplicated urinary tract infection E. Vaginitis

The correct answer is D. A urinary tract infection presents with increased urinary frequency, dysuria, urinary urgency, and suprapubic pain. It can be associated with burning upon urination. Risk factors for UTIs include sexual intercourse, female gender, pregnancy, the presence of an indwelling urinary catheter, structural abnormalities (BPH, neurogenic bladder,etc.), immunocompromised states, diabetes, spinal cord injuries, and a history of recurrent UTIs. ** Pyelonephritis would present with fever, chills, and flank pain. It often is associated with urinary symptoms, such as increased frequency and dysuria. Nausea and vomiting may be present. Physical exam for a patient with pyelonephritis would reveal tachycardia, fever, and positive costovertebral angle tenderness. They may exhibit abdominal tenderness. The patient does not have fever, CVA tenderness, and is not complaining of symptoms that are indicative of pyelonephritis. Urosepsis would present with acute onset of fever, chills, tachycardia, tachypnea, and altered mental status. This patient does not have any of these presenting signs. Pregnancy does involve increased urinary frequency, and it a risk factor for UTIs. However, it is not a likely diagnosis in this case, as the patient's urine hCG is negative. Vaginitis would present with vaginal itching or irritation, as well as vaginal discharge. Examination of the discharge is necessary to discover the etiology. This patient is not complaining of any vaginal discharge or itching, making this a less likely diagnosis.

A 17-year-old male football player presents to the emergency department with acute onset of rightshoulder pain. The pain started approximately two hours ago when he was tackled during a game while carrying the football. Shoulder swelling and pain were present after injury. He is holding ice to a swollen shoulder, and on exam you notice severe bruising, pain on palpation over the lateral edge of the shoulder and raised bump over that same area. There is also severe tenderness over the coracoclavicular ligaments. What is most likely diagnosis? A. Rotator cuff tear B. Subacromial impingement C. Subacromial bursitis D. AC joint sprain E. Rotator cuff tendonitis

The correct answer is D. AC joint sprain presents with swelling, bruising and point tenderness of the AC joint after a fall directly on the acromion with arm adducted. AC joint injuries are classified Type I- Type VI depending on the number of ligaments involved. Type I injuries involve injury of AC ligaments with no injury to the coracoclavicular (CC) ligaments. This causes a tender AC joint that often has mild swelling. This type usually heals within a few weeks.Type II injuries involve a complete tear of the AC ligaments and a sprain or partial tear of the coracoclavicular (CC) ligaments. This causes a tender AC joint, often with significant swelling.Type III injuries involve a complete tear of both the AC and CC ligaments. Significant swelling and tenderness of CC ligaments is present along with visible abnormality of the AC joint. Type III injuries can take several weeks to months to heal.Type IV, V, VI injuries are the most severe. Treatment often requires surgery. With rotator cuff tears, there is limited ROM with significant pain. In a complete tear, the patient will not be able to raise their arm above the head. In patients with subacromial impingement or subacromial bursitis, there will be pain with overhead ROM and patients will have positive Neer and Hawkins-Kennedy tests. Patients with Rotator cuff tendonitis, when mild, will have preserved ROM and minimal pain but can have a positive Apley's scratch tests.

A 57-year-old man presents for follow-up after suffering an ischemic stroke with residual weakness on the left. His past medical history includes type II diabetes. He smokes 1/2 pack per day of tobacco and he does not drink alcohol. He reports no medication allergies. He has at times found the costs of his medications and test strips to be a barrier to adherence. What pairing of an anti-platelet agent and its rational is the best choice for this patient? A. Aspirin 81 mg; more effective at preventing subsequent strokes than higher doses B. Aspirin 325 mg; more effective at preventing subsequent strokes than lower doses C. Aspirin 325 mg; fewer bleeding complications D. Aspirin 81 mg; fewer bleeding complications E. Clopidogrel 75 mg; optimal for patients with diabetes

The correct answer is D. Although in the past it was believed that patients on a 325 mg dose of aspirin compared to those on an 81 mg dose where conferred more protection from ischemic stroke, studies now show that increasing the dose of aspirin, while increasing the risk of hemorrhage, does not confer more benefit than the 81 mg dose. For that reason, the 81 mg dose of aspirin, is sufficient to help prevent a stroke in a patient with a TIA or other risk factors for stroke. Clopidogrel is an option for patients who have experienced stroke or TIA. It is considerably more expensive than aspirin, and thus is likely to be a barrier for this patient. Clopidogrel has not been found to be clinically superior to aspirin in general patients or those with diabetes.

Fussy baby who can drink milk What is the most likely diagnosis for Amelia at this point? Select the single best choice. The best option is indicated below. Your selections are indicated by the shaded boxes. A. Pyloric stenosis B. Intussusception C. Milk allergy D. Colic E. Gastroesophageal reflux F. Infection G. Failure to thrive

The correct answer is D. Amelia most likely is experiencing colic, as she is gaining weight normally, has normal vital signs, and a completely normal physical exam. Infants with pyloric stenosis experience projectile vomiting and may have a palpable pyloric mass on abdominal exam. Intussusception (B) is characterized by sudden recurrent periods of severe pain leading to a progressively ill-appearing child. Cow's milk allergy (C) can take a variety of forms, including anphylactoid reactions, eczema, to proctitis leading to bloody stools. There is mixed evidence that colic in bottle-fed infants can improve with a change to soy-based or hydrolized cow's milk formula, but milk protein allergy is not felt to be the predominant etiology for colic. Gastrointestinal reflux (E) typically leads to regurgitation of food, but not with the increase in fussiness seen in Amelia's case. Both infection (F) and failure to thrive (G) would present with an ill-appearing child who is not gaining weight as expected.

A 56-year-old male presents for care at the ED complaining of dry cough for the past three days. He notes that this problem started a few days after his family's annual fish fry and barbecue and has been worsening since. He has no known past medical history but mentions that he has not seen a doctor in years. He notes that the cough is worse at night often waking him from sleep. He is unable to lie flat on his back and has started using three to four pillows to sleep comfortably. He also reports increased swelling in his legs that worsens throughout the day. He denies having any chest pain or palpitations and also does not believe he has had any sick contacts. He does not know his family history since he was adopted as a child. He has not had any fevers, sweats, or chills. On exam, you observe a tachypneic, obese man in mild distress. On chest auscultation, he has an S3, bilateral crackles at the lung bases, and 2+ pitting edema in the lower legs bilaterally. What diagnostic test would you perform first? A. Exercise stress test B. Pharmacologic stress test C. Echocardiogram D. Electrocardiogram E. Cardiac catheterization SUBMIT

The correct answer is D. An ECG can quickly help determine whether the patient is in sinus rhythm, whether there is ischemia or infarction, or whether there is left ventricular hypertrophy. These findings may help you determine the etiology of this patient's congestive heart failure. Other tests (especially an echocardiogram) may be indicated at some point during the patient's work-up, but may not be appropriate first tests to order. Specifically, cardiac stress testing and cardiac catheterization are more useful in ascertaining the diagnosis and prognosis of coronary artery disease than those of congestive heart failure, although they are often a concurrent part of an extensive workup.

An 18-year-old non-smoking male comes to the clinic with cough, chest tightness, and difficulty breathing. His past medical history is positive only for allergic rhinitis as well as an undiagnosed chronic cough primarily at night since he was 4 years old. He has no past surgical history and no recent travel. On physical exam, you notice the patient appears in mild distress, has hunched shoulders, is using accessory muscles during respiration, and is only able to talk in two to three word sentences. His vitals are: Pulse 125/min, O2 sat 88%, BP 100/70 mmHg. On auscultation, you hear wheezing on inspiration and expiration throughout both lungs. You also notice a prolonged expiratory phase. What is the most likely diagnosis? A. Cystic Fibrosis B. CHF exacerbation C. Foreign body aspiration D. Severe asthma exacerbation E. Pulmonary embolism

The correct answer is D. Based on the clinical presentation including the patient's age and uncomplicated past medical history, the most likely diagnosis is an asthma exacerbation. The presence of hypoxemia places this in the "severe" category. While pulmonary embolism and foreign body aspiration are still on your differential, they are both much further down again based on the patient's age and reported past medical history. Neither cystic fibrosis nor heart failure are likely given the past medical history.

You are seeing a 56-year-old male who presents with daily headaches for the past two weeks. He describes them as an intense pressure behind both eyes that is throbbing in nature. He reports that lights and loud noises bother him some, but he has been able to tolerate his usual activities as a short-haul truck driver. The headaches tend to last all day. He has not had nausea, visual changes, fever, or chills. These symptoms have occurred in the context of him quitting tobacco, which he has found difficult. He read online about cluster headaches, and he thinks that is what he is experiencing. He would like you to prescribe oxygen treatment to help with the headaches. Which of the following characteristics is most typical of cluster headaches? A. Patients with them often complain of a pulsating quality of pain. B. The pain is typically periorbital and bilateral. C. They are associated with photophobia and phonophobia. D. They are associated with rhinorrhea, lacrimation, miosis, and ptosis. E. They typically last less than a week.

The correct answer is D. Cluster headaches are generally described as severe, stabbing, and unilateral episodes of periorbital or temporal pain associated with rhinorrhea, lacrimation, miosis and ptosis on the affected side (D). There is a strong male predominance among those who experience them, though they are still relatively rare even among men. Pulsating or throbbing pain are words typically used to describe migraines (A), though there is some variability in how patients experience their headaches. Bilateral, periorbital pain (B) is typical of tension-type headaches and/or migraines. Cluster headaches are exclusively unilateral.

A 28-year-old, G2P1 female delivers a 6 lb., 7oz. baby boy at 39 weeks gestation. At one minute, the baby has blue extremities and a pink body; his arms and legs are flexed and he is moving them vigorously with prompt response to stimulation; HR is 118 bpm and he is coughing and crying vigorously as well. What would his APGAR score be at one minute? A. 6 B. 7 C. 8 D. 9 E. 10

The correct answer is D. His APGAR is calculated as follows: One point for "Appearance" (color); two points each for "Activity," "Pulse," "Grimace" and "Respiration."

Your 28-year-old male patient with testicular pain has been experiencing pain at a level of 8/10 x 2 hours. Within how many hours should surgery be performed on a patient with a diagnosed testicular torsion? A. 24 hours B. 12 hours C. 8 hours D. 6 hours E. 4 hours

The correct answer is D. In order to increase the chances of saving the testis in a testicular torsion, one must seek medical attention and have the testicle detorsed within 6 hours of onset of pain. If the individual waits longer than 6 hours, he increases his chances of losing the testicle and/or impaired fertility.

Ms. Tsvetanova is a 42-year-old woman with no significant past medical history presenting to establish care with her primary care physician. On review, she notes a weight gain of 14 kg (30 lbs) over the last three years. She attributes this mostly to her sedentary lifestyle, snacking, and difficulty with portion control. She works as a receptionist for a local physician's office and spends most of her day sitting. She denies constipation, low energy, cold intolerance, muscle weakness, depressed mood, easy bruisability, or other skin changes. On physical exam, vital signs reveal temperature is 36.8C (98.2F), pulse is 82 beats/minute, respiratory rate is 12 breaths/minute, blood pressure is 130/82 mmHg, weight is 81.6 kg (180 lbs), and height is 163 cm (64 in). The remainder of her physical exam is normal. Which of the following laboratory tests is most appropriate for the evaluation of this patient? A. 24-hour urine catecholamine levels B. 24-hour urine cortisol level C. Basic metabolic profile D. Lipid profile E. Thyroid stimulating hormone (TSH)

The correct answer is D. In the absence of symptoms and signs associated with secondary causes of obesity such as hypothyroidism and Cushing's disease, the initial laboratory evaluation in patients with obesity should be limited to assessment of co-existing risk factors for atherosclerotic cardiovascular disease (ASCVD), including dyslipidemia and diabetes. Screening guidelines vary regarding lipid screening in adults. The American Heart Association (AHA) recommends screening all adults aged 20 to 79 years old every four to six years in those without ASCVD. The U.S. Preventive Services Task Force (USPSTF) strongly recommends (grade A recommendation) screening in all men >35 years old and women >45 years old who are at increased risk for heart disease. The USPSTF also recommends (grade B recommendation) screening for men 20 to 35 years old and women 20 to 45 years old if they are at increased risk for heart disease.

A 57-year-old female comes in, concerned about a 1.5 cm dark multicolored mole lateral to her left eye that has been increasing in size over the past six months. A punch biopsy shows pathology indicative of squamous cell carcinoma. What is the most appropriate intervention? A. Three-month follow-up visits for the first year; then every six months B. Avoid artificial sources of UV light, such as indoor tanning C. Cryotherapy extending 4 mm beyond the lesion margins D. Mohs surgery E. Excisional biopsy extending 5 mm beyond lesional border

The correct answer is D. Lesions that are close to facial features such as the eyes and in cosmetically sensitive areas are best treated with Mohs surgery because it is tissue sparing and has better cosmetic results. A punch biopsy is done for diagnosis. It would not be effective at getting the required 4 mm margins so the patient would need further intervention before going to surveillance. Treatment with 5-FU or cryotherapy is not indicated for squamous cell carcinoma. Excisional biopsy is a potential treatment, but it would require removal of a large area of tissue in a cosmetically sensitive area and may require additional surgery if the margins are not clear.

Ms Smith is a 38-year-old female with a significant past medical history of hypercholesterolemia who presents to her primary care physician after 4 weeks of episodic epigastric discomfort. She reports a recurrent ache-like sensation in the epigastric region occurring about 10 minutes after eating and lasting for several hours. The pain does not appear to be associated with any particular food. She denies any odynophagia or dysphagia. Ms Smith is a smoker, with a 30-pack year history. You are concerned about peptic ulcer disease. Which additional history finding would increase your level of concern? Choose the single best answer. A. Report of hoarseness B. Epigastric discomfort associated with recumbency or bending C. Bouts of recurrent laryngitis D. Return symptoms despite consistent use of antacids E. Sour or bitter taste in mouth

The correct answer is D. Symptoms that would increase the likelihood of PUD include pain that improves with meals (although some people experience the opposite), history of NSAID use, a pain described as "gnawing" or "hunger-like" and persistent symptoms despite adequate acid-blocking therapy. Hoarseness, epigastric discomfort with lying down, laryngitis, discomfort after large meals, and bitter taste are all typical symptoms of GERD.

It is November and you are working in a small, rural, emergency department serving a community who is currently going through a flu epidemic. Your next patient is a 4-year-old male who was brought in by his mother for a sore throat and fever that started two nights ago. She says he has a mild cough, and is complaining of headaches as well. Since last night, he has had a decreased appetite and hasn't been his normal, active self. She also wants you to know that he is allergic to eggs and latex, and uses an inhaler once a month for asthma like symptoms. On physical exam you note an erythematous throat, clear rhinorrhea, and rhonchi on auscultation. A rapid strep test was performed in the office and is negative. His last well child check was 14 months ago, and his mother says she knows he is due for another but her schedule has been too busy. What is the next best step in management? The best option is indicated below. Your selections are indicated by the shaded boxes. A. Flu Shot B. Amoxicillin therapy C. FluMist (influenza vaccine, live intranasal) D. Tamiflu (oseltamivir phosphate) E. Influenza test. SUBMIT

The correct answer is D. Tamiflu can be prescribed in patients who show signs of flu like symptoms without further testing in communities with known flu outbreaks. Tamiflu can reduce symptoms of the flu, and is recommended only in patients who are still within a 48 hour window since symptoms began. In a population that has a known flu outbreak, patients presenting with symptoms compatible with the flu do not require a nasal swab for influenza unless it will change the treatment. Amoxicillin could be a potential answer if you were treating strep throat, but a negative strep test and only moderate probability make this a less likely choice. The Flu Shot is not recommended in patients with active illness/fever. While research has shown that the flu shot can be given to patients with egg allergies, the live virus form of the vaccine given in Flu mist should be avoided in this population. Also, Flu Mist is currently not approved for patients with respiratory disease such as asthma.

James is brought in by his mother and father today for a well-child visit. His mother had a normal term pregnancy and uncomplicated spontaneous vaginal delivery. Thus far James has been meeting all his developmental milestones. Today James can feed himself, easily pedal a tricycle, uses three-five word sentences, can be understood by strangers 75% of the time, can copy a square, takes turns in games, and engages in fantasy play with his siblings. At what developmental age is James? The best option is indicated below. Your selections are indicated by the shaded boxes. A. 1 year B. 18 months C. 2 years D. 3 years E. 6 years

The correct answer is D. The milestones are consistent with a developmental age of 3 years. The milestones are most consistent with a developmental age of 3 years. (A) is incorrect because a 2-year-old child will most commonly play beside other children, rather than with other children. Two-year-olds typically have two- to four-word sentences. While a 2-year-old may copy a straight line or circle, a 2-year-old is unlikely to copy a square and pedaling at this age would be an emerging skill. (B) is incorrect as an 18-month-old may have several single words, but does not yet speak in sentences. 18-month-old children may scribble, but cannot copy a square or pedal a tricycle. (C) is incorrect because a 1-year-old cannot pedal a tricycle, copy a square, or take turns playing with others. (E) is incorrect because a 6-year-old should be able to speak clearly and tell stories in full sentences. More details on developmental milestones are available here: https://www.cdc.gov/ncbddd/actearly/milestones/index.html.

A 34-year-old male comes to the clinic complaining of recurrent abdominal pain. He says the pain has been bothering him for the past eight months. He reports episodes of diarrhea and constipation, with more episodes of constipation. He states he has noticed an increase in flatulence. He denies any nausea or vomiting. He has noticed mucus in his stools, but no blood. He states that he cannot recall if anything aggravates the pain, but admits to being under more stress than usual, due to his mother-in-law moving in with him and his wife. Vital signs show a blood pressure of 124/76 mmHg, pulse of 74, respirations of 16, a temperature of 97.9, and oxygen saturation of 98% on room air. Physical exam is unremarkable. The most appropriate initial step in management is: A. Scheduling the patient for a colonoscopy to look for colon cancer B. Discussing the patient's diet, and educating him about avoiding dairy products C. Performing a CBC, TSH, complete metabolic panel, and stool studies D. Offer behavioral therapies and exercise to help with symptoms E. Scheduling the patient for a CT scan of the abdomen to rule out small bowel obstruction

The correct answer is D. The patient most likely has a diagnosis of irritable bowel syndrome (IBS). IBS is diagnosed based on clinical history, physical exam, and absence of alarm symptoms suggesting other pathology. The Rome IV criteria are often used to aid diagnosis of adult IBS: Recurrent abdominal pain, on average ≥ 1 day per week in past three months with ≥ 2 of following features: 1) related to defecation; 2) associated with change in stool frequency; 3) associated with change in stool form (appearance). Laboratory studies are not required to make this diagnosis if other diagnoses are unlikely, as in this case. Offering behavioral therapies and exercise to help with symptoms would be appropriate first steps to help with irritable bowel syndrome. Scheduling a colonoscopy to search for colon cancer is inappropriate, as he is not presenting with bloody stool or significant weight loss. The patient is too young to undergo a screening colonoscopy without a history of a first-degree relative who was diagnosed with colon cancer at an age 10 years older than his current age. Discussing the patient's diet is suitable; a trial of avoidance of dairy products would be dependent on history, and a recommendation that includes this strategy may or may not be appropriate. Scheduling a CT scan of the abdomen to rule out a small bowel obstruction is inappropriate. The patient is not complaining of nausea or vomiting, and has no abdominal tenderness or hyperactive bowel sounds on physical exam, all of which would be suspicious for small bowel obstruction.

Sally is a generally healthy 27-year-old female graduate student who presented to your office twelve weeks ago with episodic post-prandial epigastric burning. This had been bothering her for nearly six months but she had been busy with her thesis and was unable to find the time necessary for an appointment. She reported this year has been particularly stressful, with limited time resulting in increased consumption of coffee and take-out fast food. At that time, she began an eight-week trial of omeprazole. She returns now with no improvement of her symptoms. She discontinued the omeprazole one month ago because she ran out of the medication. She has no additional symptoms and physical exam is unremarkable. Which of the following is the most appropriate next step in her management? Choose the single best answer. A. Refer her for an upper gastrointestinal endoscopy B. Begin treatment with triple therapy of pantoprazole, clarithromycin, and azithromycin for 14 days C. Continue omeprazole for another 4 weeks with close follow up D. Administer a urea breath test E. Switch PPI from omeprazole to pantoprazole

The correct answer is D. This patient exhibits no alarm symptoms, but does have persistent symptoms despite adequate empiric therapy. This case warrants testing for H. pylori as a cause of symptoms. One test which is sensitive and specific for H. pylori infection is the urea breath test. The antibiotic regimen listed in choice B is for treatment of H. pylori, but a diagnosis must be made before instituting such a treatment regimen. Switching PPI's or doing a longer treatment of the same medication would not be optimal choices, since the patient hasn't found any relief thus far with the medication. While an upper endoscopy might assist with diagnosis, it is more invasive and in the absence of alarm symptoms, a less invasive test for H. pylori (urea breath test) is acceptable.

You are seeing a 63-year-old male with hypertension, diabetes, and a history of an NSTEMI two years ago. His most recent echocardiogram reveals mild hypokinesis of the inferior wall of the left ventricle and a LV ejection fraction of 40%. Shortly after his MI, he was treated for symptoms of congestive heart failure, but he has not has any such symptoms since then. His exercise tolerance is excellent. Today his physical exam is completely unremarkable. The correct pairing of NYHA functional class and ACCF/AHA Stage of CHF is which of the following for this man? A. NYHA II / Stage C B. NYHA I / Stage A C. NYHA I / Stage B D. NYHA I / Stage C E. NYHA II / Stage B

The correct answer is D. This patient has no current physical limitations from his CHF, so his NYHA class is I, but he has a history of symptomatic CHF, thus his ACCF/AHA stage is C. NYHA relies purely on someone's current functional status. Thus a person with known CHF but who has no symptoms or limitations (such as this one) is NYHA class I. The ACCF/AHA Staging is based on the stage of illness. Any person who has ever had symptoms or physical findings of CHF is at least Stage C. If their symptoms are unable to be controlled, they are Stage D. Stage A indicates a patient at risk of developing CHF, but with no known findings or symptoms. Stage B is for patients with evidence of decreased cardiac function (eg. decreased ejection fraction) but who have never experiences symptoms of congestive heart failure

You are seeing a 33-year-old male who presents with several weeks of gradually-worsening headaches that he describes as severe and upsetting. They tend to occur three to four times per week and get worse as the day goes on. He describes them as feeling "like a vice squeezing my whole head." He has tried ibuprofen for them, and it has helped some. Unfortunately he never has ibuprofen with him at work where the headaches are more common, because his new boss does not allow medications to be kept at work without a doctor's note. He denies nausea, vomiting, lacrimosis, and photophobia. He is not awakened from sleep by the headaches. He appears mildly distressed, and his vital signs are normal. Examination of his head and neck, fundic discs, and cranial nerves is broadly normal. What is the most appropriate step in management of his headaches? The best option is indicated below. Your selections are indicated by the shaded boxes. A. Instruct him to discontinue ibuprofen, as he is having rebound headaches. B. Order a CT scan of his head to rule out intracranial pathology. C. Prescribe propranolol to be taken as migraine prophylaxis. D. Prescribe ibuprofen to be taken as needed and provide a note for work. E. Prescribe sumatriptan as needed and provide a note for work.

The correct answer is D. This patient is experiencing tension-type headaches, which are classically described as a tight sensation surrounding the head and are often made worse with activity and stress (eg. a new boss). The most appropriate first-line treatment for tension-type headaches is an NSAID, such as ibuprofen (D) on an as-needed basis. Rebound headaches are common in patients who are taking medications for headache every day for an extended period, unlike this patient (A). Head imaging, such as a CT scan (B), would be inappropriate without red-flag characteristics, such as abrupt onset, recent trauma, or neurologic findings. Prophylactic medications such as propranolol (C) and episodic medications such as sumatriptan (E) are appropriate for the treatment of migraine headaches, not tension-type headaches.

After reviewing the treatment options in the preceding list, considering all aspects of Sarah's history and exam, which would be the best initial treatments for prevention of her migraine headaches if her lifestyle measures fail to reduce her symptoms to the acceptable level? Select all that apply. A. Propranolol B. Amitriptyline C. Divalproex sodium D. Topiramate E. Feverfew

The correct answers are A, B.

A 28-year-old male comes to the Emergency Department with shortness of breath, cough and wheezing for the last 4 hours. He states that he was diagnosed with asthma recently, and is currently using inhaled corticosteroid with a long acting B2-agonist daily to control his symptoms. His RR is 34/min; Temp: 98.8 degrees F; O2sat: 88%; BP: 130/85 mmHg. What treatment should be given to this patient first? A. Albuterol breathing treatment using a nebulizer B. Intravenous corticosteroids C. Ipratropium breathing treatment using a nebulizer D. Oxygen E. Intravenous third generation cephalosporin

The correct answer is D. This patient is in acute respiratory distress with a decreased oxygen saturation and increased respiratory rate. The first treatment this patient should receive is oxygen supplementation. After giving the patient oxygen, you can then continue treatment for his acute asthma exacerbation with albuterol and ipratropium breathing treatments and intravenous or oral corticosteroids. Antibiotics are not indicated for treatment of asthma exacerbations.

A 59-year-old patient comes to the local ER with a swollen, tender knee that started yesterday. He returned home two days ago from a 5-day hike on the Appalachian Trail. He denies any recent or previous injury to the knee and any history of previous inflammatory joint disease. Vital signs: temperature is 97.7, pulse is 80, BP is 139/75, and RR is 22. His lower extremities are marked with abrasions in various stages of healing. He holds the knee in full extension. The knee is swollen, reddened, tender, and it feels warm. Which of the following is the most appropriate next step? A. Prescription for full-dose acetaminophen B. X-ray of the knee, including sunrise and standing films C. Magnetic resonance imaging (MRI) of the knee D. Knee aspiration E. Prescription for colchicine

The correct answer is D. This patient is presenting with an acutely inflamed knee which could represent an initial presentation of an inflammatory rheumatalogic disease such as rheumatoid arthritis (less likely), gout, or a joint infection. The latter diagnosis can be rapidly destructive of the joint and needs to be ruled out urgently with a joint aspiration and fluid analysis. This would also help distinguish between infection and gout. Knee imaging would be useful for identifying bony (X-ray) or soft tissue (MRI) pathology, but would not determine if this patient has a septic joint. Colchicine is a treatment for gout which would only be appropriate if this condition had been definitively diagnosed.

A 56-year-old female with a past medical history significant for hypothyroidism and recurrent urinary tract infections presents to the office to discuss dyspareunia and hot flashes that began approximately one year ago. Her last menstrual cycle was 14 months ago, and her family history is significant for breast cancer in her sister, diagnosed at the age of 47. She has become increasingly bothered by the frequency of her hot flashes and has stopped attending social events with her friends due to these symptoms. Which one of the following is true regarding the risks and benefits of hormone-replacement therapy (HRT)? A. Use of combined estrogen and progesterone therapy decreases the risk of breast cancer. B. Use of unopposed estrogen in patients with an intact uterus decreases the risk of endometrial cancer. C. Use of hormone therapy for less than five years will not affect a patient's risk of coronary artery disease. D. Use of hormone therapy decreases the risk of osteoporotic fractures. E. Both systemic and topical estrogens are equally effective for the treatment of vasomotor symptoms.

The correct answer is D. Use of combined estrogen and progesterone beyond three years increases the risk of breast cancer. Use of unopposed systemic estrogen in females with an intact uterus increases endometrial cancer risk. Beginning HRT after age 60 increases the risk of coronary artery disease. Systemic estrogen is most effective for treatment of vasomotor symptoms. HRT has been shown to decrease the risk of osteoporotic fractures.

A 24-year-old G1P0 patient at 14 weeks presents with vaginal bleeding and abdominal cramping. On examination her vital signs are: blood pressure 120/75 mmHg, pulse rate 74/minute, temperature 98.4 degrees F taken orally, respiratory rate 18/minute, and oxygen saturation 99% on room air. On pelvic examination, there is a small amount of blood in the vagina, the cervical os is open, and there is no cervical or adnexal tenderness noted. On pelvic ultrasound, an intrauterine gestational sac with a yolk sac is seen. What is her diagnosis? A. Complete abortion B. Threatened abortion C. Missed abortion D. Inevitable abortion E. Septic abortion

The correct answer is D. When the cervical os is open or dilated, bleeding in early pregnancy is classified as an inevitable abortion. When the cervical os is closed with bleeding before 20 weeks, the correct diagnosis is a threatened abortion. A missed abortion is fetal demise without cervical dilation. Complete abortion occurs when products of conception have been completely expelled from the uterus. Septic abortion is highly unlikely in the setting of her being afebrile and pain free.

A 21-year-old female comes into your family medicine office complaining of an abnormal discharge with a foul odor. She has been sexually active with four to five partners over the past year, uses condoms inconsistently. Which of the following can be seen on a wet prep? A. Chlamydia B. Gonorrhea C. Syphilis D. Trichomonas E. Herpes

The correct answer is D. While WBC's may be evident on the wet prep in chlamydia and gonorrhea, for diagnosis these require nucleic acid amplification tests done on urine, endocervical or urethral samples to be detected. RPR and FTS-ABS are the ways to detect syphilis. Herpes is usually diagnosed clinically or through a PCR blood test; herpes is not visible on a wet prep.

You are on call over the weekend for the pediatric clinic and a concerned mother calls in. She has three children at home, her youngest being 5-weeks-old. She's very busy and about to leave town on Monday to visit relatives out of state. She tells you that for the last week her two oldest children have had a cough, runny nose, and one or two fevers as high as 101.2 F, but these symptoms have improved in both children with Tylenol. Her youngest child is now having the same symptoms and has a temperature of 100.5 F. Because they're about to go out of town, mom wants to know if you can call in something stronger like an antibiotic so her youngest can feel better sooner. What is the most appropriate step in managing this child? A. Call in a prescription for amoxicillin as this will cover the likely causative organisms for this infection B. Continue to use Tylenol as needed for fevers and supportive care C. Educate mom that this is likely a viral infection and it will have to run its course D. Recommend that the infant be evaluated immediately in your office or the nearest emergency department E. Call in a prescription for a cough syrup so both child and mom can get more sleep.

The correct answer is D. While the baby may well have the same viral URI given the likely exposure to her older siblings with similar symptoms, a 5-week-old with a fever should be evaluated for possible sepsis. Young infants are at risk of developing invasive bacterial infection due to immaturity of the immune system. Additionally, they may not manifest focal symptoms or findings other than fever. Thus, all febrile neonates (within the first three months of life) should be evaluated clinically prior to initiating treatment. It would be inappropriate to reassure the mother that the cause is a virus with only symptomatic care and Tylenol or treat with antibiotics empirically before a full evaluation.

What is the most appropriate next step? Choose the single best answer. A. Give empiric antibiotics for strep pharyngitis. B. Do a rapid antigen detection test for strep pharyngitis and do not do a strep culture if the test is negative. C. Do a throat culture (sent to lab and results in two to three days). D. Do a rapid antigen detection test for strep pharyngitis and a strep culture if the test is negative. E. Symptomatic treatment only. Antibiotics are not indicated in this patient.

The correct answer is D. You review Althea's symptoms and clinical findings again with Dr. Medel. You note the lack of physical findings to support a diagnosis of infectious mononucleosis or the presence of a retropharyngeal abscess, as well as her recent exposure to a friend with strep pharyngitis. She has a Modified Centor score of 4 (one each for fever, her age, cervical lymphadenopathy, and tonsillar erythema). The most appropriate next step is a rapid antigen detection test (RADT), and send a strep culture if the RADT is negative (D). Children over age 3 with a negative RADT should have a follow-up strep throat culture for further evaluation. Children less than age 3 should not routinely be tested for strep throat, as rheumatic fever is unlikely in this age group. (A) is incorrect as empiric antibiotics are not warranted for strep throat. (B) is incorrect as a negative RADT in children over age 3 requires a confirmatory culture. (C) is incorrect given the high Centor score and potential for unnecessary treatment delay while waiting for culture results. (E) is incorrect as this patient may have strep throat, and if so, antibiotics are appropriate.

A 64-year-old male comes in for a routine physical examination. He notes that over the past few months he has had to get up to urinate in the middle of the night. Benign prostatic hypertrophy (BPH) is on your differential. What other symptom is consistent with BPH? A. Cloudy penile discharge B. Erectile dysfunction C. Hematuria D. Urinary urgency E. Dysuria

The correct answer is D. Urinary urgency is a common symptom of BPH. Cloudy penile discharge is more consistent with urethritis, prostatitis, gonorrhea or chlamydia. Erectile dysfunction: this differential is wide, including psychosocial, endocrine, vascular, structural, medication side effect, chronic disease. While symptoms of BPH might influence dysfunction, it is not a primary cause. Hematuria would be more likely in infections, urinary tract or kidney stones, trauma or cancer. Dysuria can be related to a UTI, STI, topical or systemic irritant, and is uncommn in BPH.

If a patient asked you about a test result and you knew the test was abnormal but did not know how to interpret it, what would you feel most comfortable doing? (This may or may not actually reflect what you think you should do!) Choose the single best answer. A. Deny knowing anything about the test results and go ask Dr. Medel. B. Give the result (i.e., 1:100 risk for Down syndrome) without any additional information. C. Reassure the patient that it's no big deal. D. Ask the patient why they haven't gotten the results before now. E. Advise the patient that the results are back, but you aren't certain what they mean.

The correct answer is E.

Which of the following conditions may present with shoulder pain? Choose the single best answer. The best option is indicated below. Your selections are indicated by the shaded boxes. A. Myocardial infarction B. Cancer of the lung C. Cholecystitis D. Ruptured ectopic pregnancy E. All of the above

The correct answer is E.

Which of the following treatments are clearly safe and probably effective for the management of hot flashes according to the best available evidence? Select all that apply. A. Mind and body practices such as yoga, tai chi, qi gong, and acupuncture B. Phytoestrogens such as soy and red clover C. Botanicals such as black cohosh, don quai, and kava D. Selective serotonin reuptake inhibitors (SSRIs) and serotonin-norepinephrine reuptake inhibitors (SNRIs) E. Bioidentical hormone replacement therapy and DHEA F. Prescription medications such as clonidine and gabapentin

The correct answers are D, F.

Working at your clinic, you receive a call from a patient of yours, Mr Smith, a 45-year-old male who was seen three days ago complaining of lower back pain. At that time he had no history of trauma, pain that improved while lying down and no neurologic deficits. He works as a truck driver. He was treated conservatively along with pharmacologic intervention with NSAIDs and muscle relaxants. He calls your office now due to only minimal improvement. And although his symptoms have not changed, he is frustrated with the slow progress, needs to get back to work as soon as possible, and is concerned this might be "something serious." Which of the following is the most appropriate next step in management? A. Obtain a plain film x-ray B. Order an MRI C. Ask him to double the dosage of his muscle relaxants D. Schedule him for an appointment immediately E. Reassure him and schedule a follow-up appointment in a few days

The correct answer is E. Given this clinical presentation, the likelihood of this being an episode of lumbar sprain/strain is high, and the odds of this being "something serious" (nerve root compression, malignancy, infection) is still low. The original plan is a good one, and should continue. No new meds or imaging studies would help, and an urgent appointment will not change the anticipated course. Some physicians might choose to involve a physical therapist at this time, but this option is not available for this question. QUESTION 5

A 31-year-old female, G1P0 at 26 weeks gestation, presents to the clinic for her gestational diabetes screening. Her pregnancy has been uncomplicated. Vital signs are stable. Which of the following would require a follow up three-hour glucose tolerance test? A. Fasting serum glucose concentration of 91 mg/dL B. Fasting serum glucose concentration of 112 mg/dL, and a one-hour serum glucose concentration of 128 mg/dL C. Two-hour serum glucose concentration of 146 mg/dL D. Fasting serum glucose concentration of 113 mg/dL, and a two-hour serum glucose concentration of 134 mg/dL E. Fasting serum glucose concentration of 130 mg/dL, and a one-hour serum glucose concentration of 158 mg/dL SUBMIT

The correct answer is E. Screening for gestational diabetes is usually performed between 24 and 28 weeks gestation, using a fasting glucose and a one-hour glucose following a 50g glucose load. If the fasting glucose is greater than 126 mg/dL, OR the one-hour glucose is greater than 130mg/dL (90% sensitivity) or 140mg/dL (80% sensitivity), then the patient is considered to have a positive result. In the case of a positive one-hour glucose, the patient should undergo a three-hour GTT with a 100g glucose load. Only answer choice 'E' has a result that would require a follow up three-hour GTT. 80% sensitivity and 90% sensitivity from: Gestational Diabetes Mellitus; American Diabetes Association; Diabetes Care, Vol 26, Supp 1, Jan 2003.

Mr. Brown is a 42-year-old male accountant with a significant past medical history of obesity who presents to his primary care physician after one week of lower back pain. After moving into a new home three days ago, he woke up the next morning with bilateral lower back pain without any radiation. He denies any recent trauma, fever, chills, numbness, tingling, or incontinence. He has not had any urinary frequency or dysuria. He takes no medications and has no significant past medical history. Which additional findings in his history or physical exam would make the diagnosis of lumbosacral sprain/strain more likely? A. Increased pain with coughing B. Abnormal gait C. Point tenderness on spinous processes D. Loss of ankle jerk E. Spasm of paraspinous muscles

The correct answer is E. Spasm of the paraspinous muscles suggests lumbosacral sprain/strain. Increased pain with coughing, abnormal gait and loss of ankle jerk point to conditions that compress a regional nerve root, while point tenderness on the spinous processes often indicates an origin in the vertebra (osteoporotic fracture, malignancy, etc.).

A 72-year-old female with a 30-year history of Type 2 diabetes returns to your office for routine visit. She is taking 20 units of insulin glargine every morning and five units of insulin aspart with meals. She is on atorvastatin 40 mg daily. She is on no other medications. Her A1c is 6.5%, and her BP today is 145/90. The patient notes blurry vision for the past several months and a few days of dark spots in her vision. She denies headaches or nausea. What is the most appropriate next step to slow down the progression of diabetic retinopathy? A. Increase her insulin aspart from five units to seven units with meals. B. Increase her insulin glargine to 23 units every morning. C. Perform a fundoscopic examination and make no changes to her regimen today. D. Start her on a baby aspirin. E. Start her on a calcium channel blocker.

The correct answer is E. The patient's symptoms describe diabetic retinopathy which affects 40% of people with diabetes who are on insulin after five years (25% of those on oral agents). Proliferative retinopathy is prevalent in 25% of the diabetes population with ≥ 25 years of diabetes, but many patients have retinopathy much earlier. Patients with diabetes need to see an ophthalmologist regularly for a dilated retina exam and should not rely on an undilated fundoscopic exam by a primary care physician (C) as effective screening. Increasing either her mealtime insulin (A) or her basal insulin (B) would be inappropriate, as her hemoglobin A1C is at target. Aspirin (D) has not been demonstrated to have an impact on diabetic retinopathy. This patient's blood pressure is above her target of 130/80 mmHg. Starting her on any of the first-line options for blood pressure management, such as a calcium channel blocker (E), would be appropriate. There is fair evidence that in addition to decreasing cardiovascular risk, blood pressure control may prevent progression of diabetic retinopathy.

What is 2nd step after history for breast lump

The next step is a thorough breast exam: Certain characteristics on physical exam increase the suspicion of malignancy.

A 19-year-old female with no significant past medical history is involved as the driver in a motor vehicle accident and brought to the Emergency Department by EMS. She is complaining of severe pain in her right lower extremity that has been worsening since the accident. In addition, she has started to notice what she describes as "burning and tingling" in her right foot. On physical exam, her right calf is edematous and tender with tense overlying skin. There is no swelling or tenderness of the right foot or ankle but the right dorsalis pedis and posterior tibial artery pulses are barely palpable. She cannot confirm light touch of the foot and cannot wiggle her toes on command. What is the next best step in the management of this patient? A. Reassurance and icepacks q 2 hours B. Immobilize leg and ankle with a cast C. Urgent EMG of the right lower extremity D. Diagnostic imaging of right foot and ankle E. Emergent fasciotomy

The correct answer is E. This clinical scenario describes acute compartment syndrome which is a vascular emergency. Emergent fasciotomy is the treatment of choice to relieve pressure in the calf and, if not performed, the limb could be lost due to acute ischemia. While emergent radiographs of the tibia and fibula are appropriate to evaluate for co-existent fracture, x-rays of the foot and ankle are not indicated. Reassurance, ice packs, urgent EMG and immobilization are all incorrect treatments and place the patient at risk of serious permanent adverse outcome.

A 52-year-old woman with a history of diabetes and rheumatoid arthritis presents for her annual examination. She works in an office 10 hours a day, and rarely gets exercise. Her BMI is 23 and her blood pressure is 152/85. Her previous visit two months ago showed blood pressure of 148/82. Her father had a history of diabetes and her maternal grandmother died of rheumatic heart disease at the age of 42. She admits to marijuana drug use in the past and is a nonsmoker. Which of the following is a major risk factor for coronary heart disease (CHD) that this patient has? A. Age B. Family history C. Rheumatoid arthritis D. Obesity E. Hypertension

The correct answer is E. With two elevated blood pressure readings, this patient meets the criteria for Stage 2 Essential Hypertension (E), which is one of the major risk factors for CHD. Other major risk factors include diabetes (which this patient has), smoking, and elevated lipids. These data points, which are included in the ASCVD risk took, capture most of the variability in a patient's risk for CHD. This person also has other risk factors for CHD, but ones that are not included among the major risk factors. These include her sedentary lifestyle and her rheumatoid arthritis (RA). RA (C) and other inflammatory conditions increase a patient's risk of CHD, but to a lesser extent than the major risk factors. Age (A) becomes a risk factor over age 55 for women and 45 for men. The family history (B) becomes a risk factor if a first-degree relative has CHD male < 55 and female < 65. This patient is not obese (D).

A 61-year-old G4P4 female presents to a local emergency room with vaginal bleeding. The bleeding began two days ago and is described as spotting. She began her menses at age 16 and had regular menstrual cycles until the age of 59. She endorses smoking ½ pack per day for the last 23 years and drinks 1-2 glasses of wine with dinner every evening. She denies any abdominal pain or dysuria. She is 5'7" and weighs 112 lbs. You perform a physical exam, including a vaginal exam, pap smear, and bimanual exam. Which one of the following characteristics of this patient increases her risk of endometrial cancer? A. Smoking B. Multiparity C. Body habitus D. Age of menarche E. Age of menopause

The correct answer is E. Any characteristic that increases exposure to unopposed estrogen increases the risk of endometrial cancer. This includes tamoxifen therapy, obesity, anovulatory cycles, no prior history of pregnancy, early menarche (before age 12), and late menopause (after age 52). Smoking decreases estrogen exposure, thereby decreasing risk. Oral contraceptives increase progestin levels, thus providing protection against endometrial cancer.

A 68-year-old male with GOLD Stage 3, Group D, COPD requiring 2L of oxygen at nighttime presents to clinic complaining of increasing lower extremity edema over the past few weeks. He also thinks his nighttime cough might be worse. His physical exam reveals distant breath sound with scattered rhonchi, a normal cardiac exam, and 2+ bilateral pitting edema in his legs up to his mid shins. What is the most likely mechanism of disease underlying his lower extremity edema? A. Decreased blood flow to the lower extremities due to thromboembolism B. Hepatomegaly from infiltration of the liver with granulomas C. Irregular heart rate due to atrial fibrillation D. Overexpansion of lower extremity veins due to incompetent venous valves E. Pulmonary hypertension causing right heart failure

The correct answer is E. Chronic hypoxia causes pulmonary vasoconstriction that increases blood pressure in the pulmonary vessels. This elevation in blood pressure causes permanent damage to the vessel walls and leads to irreversible hypertension. The right heart eventually fails because the pump cannot sustain flow effectively against this pressure. Right heart failure leads to an increase in preload, with peripheral edema and increased jugular venous distention. Given this patient's history of advanced COPD, E is the most likely cause. Thromboembolism would cause painful necrotic areas in the distal extremities but is not a common cause of peripheral edema. Hepatomegaly can cause venous congestion, but there is nothing in this patient's history to suggest this is likely. New onset atrial fibrillation can cause left-sided heart failure and dependent edema. This patient's cardiac exam is described as normal which indicates he is not in atrial fibrillation currently (though he could have paroxysmal AFib). He also does not present with lung findings suggestive of pulmonary edema such as crackles. His lung findings are typical for a patient with COPD. Venous stasis from incompetent venous valves is a common cause of dependent edema. This is frequently the result of a deep vein thrombosis, and thus is more typically unilateral. In this patient with advanced COPD, pulmonary hypertension is a much more likely explanation.

A 21-year-old G1P0 female present to clinic as a new patient to establish prenatal care. Which statement represents something that would not be expected to be a benefit of group prenatal care for this patient? A. Decreases the likelihood of preterm delivery B. Increases physician contact C. Increases support network D. Shared education between patients E. Increases adherence to techniques for pain management during labor

The correct answer is E. Clear, health literate communication about options for pain management are a benefit of group prenatal care, not adherence to a particular pain management plan. Answer choices A, B, C and D are benefits of group prenatal care. Advantages of group prenatal visits: increase prenatal knowledge, labor preparedness, support network, shared education between patients, and patient satisfaction provide more time for prenatal education and anticipatory guidance improve provider efficiency preterm delivery less likely (more significant for African-American women) -preterm birth is the number one cause for neonatal death in African-American infants increased birth weight of preterm infants (a significant survival determinant) combat racial disparities used for a variety of medical conditions, including diabetes and chronic pain.

A 7-year-old male is brought to your clinic with a fever of 102-103° F (38.9-39.4° C) for the last three days. He is up-to-date on all vaccinations and has no significant medical history. His mother notes that he has not had a cough but is eating and drinking less because "it hurts to swallow." On examination of his neck you notice tender cervical lymphadenopathy bilaterally, and auscultation of his back shows clear lung sounds on both sides. His oropharyngeal exam shows erythematous throat, but no tonsillar exudates. What would be the most appropriate next step? A. Empiric Amoxicillin therapy B. Empiric Levofloxacin therapy C. Empiric Tamiflu therapy D. Chest x-Ray E. Rapid Strep Test

The correct answer is E. In addressing this child presenting with fever, symptoms of pharyngitis (sore throat), and the absence of cough, the physician needs to consider the possibility of strep throat. Predictive scoring tools such as the Modified Centor criteria are useful in determining which patients need testing for strep, but should not be used to make a positive diagnosis of strep throat. This child would have a score of 4 (one each for fever, absence of cough, high-risk age group, and tender cervical lymphadenopathy) and should be tested for strep using a rapid strep test. Empiric antibiotics are not appropriate here. In the case of a positive strep test, amoxicillin would be an appropriate choice, but levofloxacin would not. Influenza often presents with fever and sore throat, but typically involves cough as a prominent symptom. This patient has no lung findings, so a chest x-ray is unlikely to be of benefit.

Gallstones found without inflammation What is the most effective therapeutic/management option at this point? Choose the single best answer. The best option is indicated below. Your selections are indicated by the shaded boxes. A. Trial of ursodiol B. Emergent cholecystectomy C. Hepatobiliary Iminodiacetic Acid scan (HIDA Scan) D. Expectant management (aka watchful waiting) E. Surgical consultation for cholecystectomy F. Endoscopic retrograde cholangiopancreatography (ERCP)

The correct answer is E. In this case, Mr. Keenan has biliary colic without acute cholecystitis. If the patient instead had acute cholecystitis, urgent cholecystectomy may be required. Typically, a less urgent surgical approach is undertaken with a transient episode of biliary colic. A surgical referral with followup in the near future (i.e. on the scale of weeks) is appropriate in this case.

A 48-year-old female with no smoking history comes to the emergency department with a concern of sore throat for the last two days. She does not have a thermometer at home, but states she has been feeling hot and her children have also been out from school for a fever and sore throat. Her children are now staying with their father who she is separated from, and he notified her yesterday that they were on antibiotics for their symptoms. She was holding off coming to the doctor because she wasn't coughing or having any problems with swallowing until breakfast this morning. You note that she has a fever of 38.6 degrees Celsius measured by the nurse, and on physical exam you observe an erythematous throat with exudate, and bilateral cervical lymphadenopathy. What is the next best step? A. Amoxicillin therapy B. Levofloxacin (Levaquin) therapy C. Tamiflu therapy D. CXR E. Rapid Strep Test SUBMIT

The correct answer is E. Modified Centor Criteria (also called McIsaac Score) allows you to calculate the need for various interventions in a patient who presents with signs/symptoms of strep throat. One point is assigned each for fever greater than 38.0 degrees Celsius, absence of cough, tonsillar exudates, cervical lymphadenopathy, and age less than 15 years old. One point is removed for age older than 45 years old. Her total points is 4 for temp, cervical lymphadenopathy, no cough, and tonsillar exudates, minus 1 point for age < 45. This gives you a total score of 3, indicating the need for rapid strep test to guide further treatment.

What are environmental factors for BC

Therapeutic radiation Obesity in postmenopausal women Excessive alcohol intake Smoking

Considering Mrs. Tomlin's history which is the best treatment option at this time for her diagnosis? Choose the single best answer. A. Acupuncture B. Combined hormonal birth control (pills, patch, Nuva-ring) C. Depo-Provera D. Hysterectomy E. Progesterone-releasing intrauterine device (Mirena) F. ParaGard intrauterine device G. Uterine artery embolization

The correct answer is E. Mrs. Tomlin's desire to have children in a couple of years weighs heavily in the decision of the best treatment option for her. TEACHING POINT Treatment for Leiomyomas and Associated Symptoms Progesterone-releasing intrauterine device (IUD) (Mirena) is an effective option for reducing menstrual blood flow in women with menorrhagia secondary to fibroids. Another appeal is that it can be left in for five years. There are potential complications, but after appropriately discussing these with a patient it is a viable option. In studies the progesterone-releasing IUD (levonorgestrel-releasing intrauterine system) has clearly demonstrated decreased menstrual flow in women with fibroids. In one smaller study, the device decreased overall uterine volume. However, it does not decrease the size of individual fibroids already in the uterus. Through decreasing uterine volume and endometrial atrophy, the progesterone-releasing IUD can also decrease dysmenorrhea. In women who hope to maintain fertility for the future yet control their symptoms now, this is one of the best options with fewest side effects. Irregular vaginal bleeding, especially initially, is a common side effect of the progesterone-releasing IUD. Other potential side effects are lower abdominal pain and breast tenderness. The risk of uterine perforation is more likely at the time of insertion. The risk of infection is within the first 20 days of insertion.

Based on her answers, what is your level of concern for her safety and what would you recommend? Choose the single best answer. The best option is indicated below. Your selections are indicated by the shaded boxes. A. Report this abuse to social services. B. Call her husband in for a family/couple's counseling session. C. Encourage her to escape and stay with family members. D. Have police or someone go and take the gun out of the house if she decides to go home. E. Provide information on resources and options and offer her an opportunity to contact local domestic violence experts from the office.

The correct answer is E. Ms. Bell is at significant risk because of the history of violence, alcohol use, and weapons in the house. It isn't clear whether the level of violence is increasing as it is common for there to be a "honeymoon period" after an episode of violence. Providing information on resources and options for the victim (E) allows the victim to make the best choices for themselves.

The Aronsen family bring their 3-week old newborn for an acute visit because his parents express worry that "something is not right." They report vomiting, which seems more than the post-feeding "dribbles" they were used to with their prior children. The baby seems more irritable and less easily consoled. They deny stool changes, fever, rash, sick contacts, or decrease in frequency of feeds. They notice that he has fewer wet diapers overall. You are concerned that, on exam, the infant does not appear as well as one week prior when he underwent his routine 2-week well baby visit. His weight has increased slightly over the past week, but not as much as expected on the growth curve. He appears to be mildly dehydrated and lethargic, but you find no other distinct physical exam signs. What is the most likely diagnosis? A. Colic B. Failure to thrive C. Gastroesophageal reflux D. Intussusception E. Pyloric stenosis

The correct answer is E. Pyloric stenosis, the most common cause of non-bilious vomiting in infants, is four times more common in male infants. It presents from three weeks on, though it can present as early as the first week of life. Signs can include projectile vomiting, but parents may not note this. A firm, olive-shaped mass in the mid-epigastric region is present in 70% of cases. You would order an ultrasound to confirm the diagnosis, which has 95% sensitivity when a mass is not palpable. Gastroesophageal reflux (C) is very common; hallmark is effortless dribbling of milk following feeds. Infant should show no sign of distress. Colic (A) is unexplained paroxysmal bouts of fussing and crying that lasts at least three hours a day, at least three times a week, for longer than three weeks, not characterized by vomiting. Intussusception (D) typically presents after three months of life, and is characterized by sudden onset of severe, paroxysmal, colicky pain, recurring at frequent intervals. If not diagnosed and treated, it may develop progressive lethargy, weakness, fever, and shock. Failure to thrive (B) typically presents later in life and is manifest by failure to gain weight or a visible drop off of the normal growth chart curve.

A 58-year-old man presents to clinic with right upper quadrant pain. The patient has a history of hypertension and GERD, but denies any past surgeries. His abdominal exam reveals a positive Murphy's sign. What would be the next best step to confirm your working diagnosis? A. Complete blood count B. Liver tests C. CT abdomen without contrast D. CT abdomen with contrast E. Abdominal ultrasound SUBMIT

The correct answer is E. Real-time ultrasonography is the preferred study to evaluate right upper abdominal quadrant because it is inexpensive, noninvasive and widely available. US provides good evaluation of gallbladder and is accurate in detection of gallstones as well as dilation of biliary tree. A CT with contrast may give you some evidence of gallbladder disease but is not the preferred test when suspecting biliary colic or cholecystitis. A CT without contrast is used primarily when there is a suspicion for kidney stones. The other tests would help with your diagnosis, but US is preferred method for biliary colic/gallstones.

Tim is a 15-year-old who comes to clinic with his father, who is concerned Tim has Strep throat. Tim has felt tired for the past day, has a sore throat, scratchy eyes, and dry cough. On exam, his temperature is 101.5 F, heart rate 75, respiratory rate 14, and blood pressure 110/65 mmHg. His pupils are equal, round, and reactive and his conjunctiva are clear. He has increased tearing in both eyes. His nares are patent with mild erythema. His oral pharynx is erythematous without tonsillar enlargement or exudate. He does not have tender or swollen anterior cervical lymph nodes. What is the most appropriate course of action? A. Empiric antibiotic therapy for Strep pharyngitis B. Heterophile antibody test for infectious mononucleosis C. Rapid Antigen Detection Test for Strep pharyngitis D. Strep throat culture E. Symptomatic management without further testing

The correct answer is E. The correct answer is (E) because this patient's modified Centor score of 1 and symptoms consistent with a viral syndrome do not warrant further testing at this time. (B) is incorrect as the absence of palatal petechiae or anterior cervical lymphadenopathy make infectious mononucleosis unlikely and the heterophile antibody test is not indicated for illness less than 7 days in duration. (C) is incorrect because RADT is not indicated for a modified Centor score of 1. (D) is incorrect because a strep throat culture is not indication in a patient with a modified Centor score of 1 and symptoms consistent with a viral syndrome.

Mr Gill is a 27-year-old male who presents to his primary care physician with post-prandial epigastric burning. The burning is episodic, without associated hematemesis, dysphagia, or odynophagia. In the past, he has episodically used an OTC chewable calcium carbonate to provide relief. Over the past month, he has had to increase the frequency of its use to four times daily. His blood pressure is 120/71 mm Hg, heart rate at 75/min and regular. Physical exam reveals only minimal epigastric tenderness on palpation but is otherwise normal. Which of the following is the most appropriate next step in management? Choose the single best answer. A. Upper endoscopy B. H. pylori IgG serologic testing C. Fecal immunochemical testing (FIT) D. Fecal occult blood testing (FOBT) E. 8 week trial of pantoprazole

The correct answer is E. This patient displays no red flag symptoms at this time. The most widely accepted initial intervention in a patient like this is empiric treatment with a histamine-2 receptor blocker or a proton pump inhibitor such as pantoprazole. Upper endoscopy would be indicated if there were alarm symptoms or if empiric therapy did not resolve symptoms. H. pylori testing might be indicated if symptoms persisted after empiric therapy. There is not a specific concern for rectal bleeding at this time, so fecal testing for blood is not indicated.

A 68-year-old male becomes confused and agitated on post-operative day three after open cholecystectomy. He does not recall his surgery or where he is, and he wants to leave the hospital. Physical exam shows temperature of 99.9 Fahrenheit, blood pressure of 143/89 mmHg, heart rate of 90 beats/minute, respiratory rate of 13 breaths/minute, and O2 sat of 98% on room air. He appears agitated and uncomfortable. He has a Foley catheter. His physical exam is unremarkable. Labs: WBC-11,000, Urinalysis: 2+ nitrites and 2+ leukocyte esterase, 10 WBC/hpf. Glucose finger stick: 80, ECG: normal sinus rhythm at 88 beats per minute. Which of the following is the best initial step to help relieve the patient's symptoms? A. Give IV ceftriaxone B. Give oral ceftriaxone C. Order urine culture D. Order urine gram stain E. Remove urine catheter

The correct answer is E. This patient has an UTI which is the most common cause of delirium in the elderly. Catheters increase the incidence of UTIs and removal (E) would be the appropriate first step to help relieve his symptoms. Ordering a urine gram stain (D) and urine culture (C) would be helpful for identification of pathogen and ideally collection of these materials should be obtained after removal of catheter with a mid-stream specimen. Administering IV ceftriaxione for his UTI (A) is an appropriate first-line antibiotic for his complicated UTI, and can be narrowed after speciation of bacteria. Antibiotics would be most effective after removal of the infected catheter. Ceftriaxone (B) is incorrect, as ceftriaxone is not available as an oral option.

You are seeing a 72-year-old woman in a family medicine office who presents reporting having had a period of facial drooping and left upper extremity weakness that lasted approximately two hours the prior day. She currently has no symptoms. She has a past medical history of hypertension and Type 2 diabetes, for which she takes losartan and metformin daily. Her family history is positive for coronary artery disease in her father. She is a daily smoker, and she does not drink alcohol. On exam, she has normal vital signs including a blood pressure of 122/74 mmHg. Her cardiac exam reveals a regular rate without murmurs. Her neurological exam is completely normal. Which of the following is the most likely pathophysiology for her presenting symptoms? A.Hemorrhage of a cerebral vessel B. Transient hypotension leading to cerebral ischemia C. Hypertensive urgency that has since resolved D. Acute hypoglycemia E. Blood clot in a cerebral vessel

The correct answer is E. This patient is presenting having had a transient ischemic attack (TIA). The most common cause of a TIA is an acute clot in a cerebral vessel (most commonly embolic, but also may be due to plaque rupture in a small cerebral vessel). If the clot resolves and circulation is restored soon enough, the stereotypical cerebrovascular ischemic symptoms resolve and we deem the episode a TIA. Cerebral hemorrhage can produce the symptoms this patient experienced. However, hemorrhage is unlikely to lead to symptoms that resolve as hers have. Once the bleeding has happened, patients typically experience the more permanent deficits of a stroke. Transient hypotension can cause neurologic symptoms, but these are not focal in nature (as this patient experienced) and typically involve changes in a patients overall sensorium. A hypertensive urgency can cause generalized neurologic symptoms such as agitation or stupor, or it can cause an acute ischemic or hemorrhagic stroke with focal findings. It is a conceivable explanation for a TIA, but it is less likely in this patient with very well-controlled hypertension on only one medication. Acute hypogycemia causes generalized neurologic findings and delirium which quickly resolve with the correction of the serum glucose. Focal neurologic findings such as this patient's would not be caused by hypoglycemia. Furthermore, metformin used alone does not cause hypoglycemia as an adverse reaction.

56-year old male with a past medical history of hypertension and type 2 diabetes mellitus presents with progressive dyspnea on exertion, orthopnea, paroxysmal nocturnal dyspnea, and dependent edema over the prior 4 days. His social history is notable for a 30 pack-year tobacco history and occasional marijuana use. His family history is positive for bypass surgery in his mother at the age of 62. On exam, his vitals include a blood pressure of 166/86 mmHg, pulse of 98 beats/minute, respirations of 21/minute and a BMI of 32 kg/m^2. He appears mildly dyspneic. His neck reveals an elevated jugular venous pulse, his chest exam reveals bilateral crackles in the lower lung fields, and his cardiovascular exam reveals mild tachycardia, a regular rhythm, and no murmurs. His abdominal exam is unremarkable, and he has 2+ edema in his legs midway to his knees. An EKG reveals sinus tachycardia with a left axis deviation and Q waves in leads II, III, and AVF. A BNP is measured and is found to be 510 pg/ml. Which of the following is the most likely cause of this patient's heart failure? A. Atrial fibrillation B. Non-ischemic cardiomyopathy C. Valvular disease D. Pulmonary embolism E. Ischemic cardiomyopathy

The correct answer is E. This patient, with several major risk factors for CAD (hypertension, diabetes, smoking, and a family history in his mother), is most likely to be presenting with heart failure due to ischemic heart disease. While we do not have information about his echocardiogram or cardiac stress testing, his risk factors make E the most likely diagnosis. Atrial fibrillation can cause CHF, but his exam and EKG showing sinus rhythm make this less likely. Non-ischemic cardiomyopathy is also a possible diagnosis, but it is less common than ischemic cardiomyopathy. Valvular disease is less likely in the patient with no audible murmurs. Pulmonary embolism can cause dyspnea and dependent edema due to right heart failure. In such patients, pulmonary edema does not usually occur unless there is also a history of left sided heart failure. The crackles on this patient's exam makes PE less likely.

Samantha and Joe bring in their newly adopted 5-year-old son, Matthew to your office with the chief concern of sore throat, without cough, that began two days ago. He was born in the United States and was neglected by his biological parents. Samantha and Joe believe Matthew lived in an old building and ate mostly non-nutritious meals. They are unsure if his immunizations are up to date. His immunization record is attached. Upon physical exam he is playful and interactive. His vital signs are: Temperature: 99.0 F; heart rate: 80; respiratory rate: 19; blood pressure: 95/63 mmHg. He has tender anterior cervical lymphadenopathy, tonsillar exudates and palatal petechiae. In addition to MMR and influenza, and Hep B, which other immunizations does Matthew need? A. Hib, DTaP, PCV, IPV B. RV, IPV, Hep A, varicella C. Varicella, meningococcal D. Varicella, HPV, DTaP E. Varicella, IPV, DTaP, Hep A

The correct answer is E. Varicella, IPV, DTaP, and Hep A vaccines (E) are appropriate. (A) is incorrect as this child has completed his PCV and Hib vaccine series. (B) is incorrect as rotavirus vaccine is not indicated at his age and he has completed the vaccine series. (C) is incorrect as the meningococcal vaccine is not indicated in otherwise healthy children until age 11. (D) is incorrect as the HPV vaccine is not indicated until age 11.

Which of the following is the best next step to manage Mr. Fitzgerald's skin lesion? Choose the single best answer. maybe cancer A. Observation B. Trial with antifungal or corticosteroid cream C. Trial with a combination of antifungal and corticosteroid cream D. Excisional biopsy E. Shave biopsy F. Incisional biopsy or punch biopsy SUBMIT

The correct answer is F.

Which of the following conditions are very important not to miss as they might be severe or life-threatening? Select all that apply. A. Appendicitis B. Hepatitis C. Ovarian cyst D. Pancreatitis E. Pelvic inflammatory disease (PID) F. Pregnancy G. Ectopic pregnancy H. Trauma

The correct answers are A, B, C, D, E, F, G, H.

Although Savannah is not planning on getting pregnant, you have informed her about folic acid supplements because any sexually active woman should be aware of preconception counseling in case she unintentionally becomes pregnant. What other issues would you want to address with women who are trying to get pregnant? Select all that apply: A. Family history of cystic fibrosis B. Risk factors for HIV C. Reviewing current medications D. Assessing current weight and nutritional status E. Assessing and advising against alcohol and tobacco use F. Screening for safety and domestic violence

The correct answers are A, B, C, D, E, F.

When screening for the likelihood of alcohol abuse and/or dependence, what specific areas could be interrogated? Select all that apply. A. Quantity of alcohol consumed per week B. Admission of a drinking problem C. Evidence or admission of an annoyance reaction to criticisms about drinking D. Use of alcohol in the morning E. Guilt about the alcohol consumption F. The desire to decrease the amount of current amount of drinking

The correct answers are A, B, C, D, E, F.

Which of the following are independent risk factors for coronary heart disease (CHD)? Select all that apply. The best options are indicated below. Your selections are indicated by the shaded boxes. A. HDL cholesterol < 40 mg/dL B. Diabetes C. Smoking D. History of premature coronary heart disease (CHD) in a first-degree relative. E. Sedentary lifestyle F. Obesity

The correct answers are A, B, C, D, E, F.

Which of the following are long-term effects from asthma? Select all that apply. The best options are indicated below. Your selections are indicated by the shaded boxes. A. Airway remodeling B. Subepithelial fibrosis C. Angiogenesis D. Airway smooth muscle hypertrophy E. Mucous gland hyperplasia and hypersecretion F. Incomplete reversibility of airway obstruction

The correct answers are A, B, C, D, E, F.

Dr. Medel asks you, "What additional tests would be appropriate to order at this time?" Select all that apply. A. Blood type/Rh status B. Chlamydia screening (urine or swab) C. HIV test D. Urinalysis E. Hepatitis B surface antigen F. Herpes I/II antibody G. Screening for bacterial vaginosis H. Hemoglobin/hematocrit I. Ultrasound (for dating) J. Toxoplasmosis K. Hepatitis C antibody

The correct answers are A, B, C, D, E, H. Incorrect options for Ms. Rios: Hepatitis C antibody screening Most clinicians would not routinely screen Ms. Rios for hepatitis C given her apparently stable relationship without other risk factors. Herpes I/II antibody (F) Type-specific HSV serology may be appropriate in some patients with a history of HSV, but not in a woman with no prior history of herpes. Bacterial vaginosis (G) Screening for bacterial vaginosis is not recommended. Ultrasound (I) Since Ms. Rios has reliable LMP dates, a screening ultrasound would not be necessary at this time. Toxoplasmosis (J) Universal screening for toxoplasmosis, cytomegalovirus, and parvovirus is not recommended.

What are the risk factors for testicular tumors? Select all that apply. The best options are indicated below. Your selections are indicated by the shaded boxes. A. Genetics B. Family history C. Cryptorchidism D. Environment E. Prior testicular cancer

The correct answers are A, B, C, D, E.

Which of the following are triggers that may increase the frequency and severity of Sarah's headaches? Select all that apply. A. Sleep disturbance B. Emotional stress C. Caffeine D. Aspartame and phenylalanine E. Intense exercise

The correct answers are A, B, C, D, E.

Which of the following are true regarding the natural history of H. pylori infection? Select all that apply. The best options are indicated below. Your selections are indicated by the shaded boxes. A. H. pylori is spread through human saliva and feces and via food and water sources. B. Ninety percent of patients worldwide with duodenal ulcers are infected with H. pylori. C. The incidence of H. pylori is declining worldwide. D. The gastric mucosa offers H. pylori protection from host immune mechanisms. E. The strongest evidence to support the role of H. pylori as an etiology of PUD is the elimination of ulcer recurrence after eradication.

The correct answers are A, B, C, D, E.

As you step out of the room, you realize that you just gave Chantel a lot of information. It's hard for patients to remember everything that they hear in a doctor's visit. Which of the following would you do to ensure the patient understands the information you have related? Select all that apply. A. Give her a handout with the appropriate anticipatory guidance. B. Have the nurse review the guidance points with her. C. Refer her to a website that has all the information that she needs. D. Give her the name of some books that have the information in it. E. Have the patient repeat back the information so that you know what she understands and remembers.

The correct answers are A, B, C, D, E. All of these could be effective strategies.

After you have finished obtaining the history of present illness, you begin to realize that the patient may be having episodes of wheezing, in addition to his cough. Which of the following can cause wheezing? Select all that apply. The best options are indicated below. Your selections are indicated by the shaded boxes. A. Asthma B. Chronic obstructive pulmonary disease C. Congestive heart failure D. Foreign body aspiration E. Pneumonia F. Persistent bronchitis G. Upper airway cough syndrome H. Epiglottitis I. Vocal cord dysfunction J. Obstructive sleep apnea K. Pulmonary embolism

The correct answers are A, B, C, D, F, G, I, K. Asthma (A) is the most common cause of persistent cough and wheezing. The following answers are incorrect: Pneumonia (E) may present with rales (wheezing can be present, but is less classic). Epiglottitis (H) may present with stridor (wheezing can be present, but is less classic). Obstructive sleep apnea (J) may present with daytime sleepiness, nocturnal snoring and irregular breathing when asleep. Causes of Wheezing Asthma is the most common cause of persistent cough and wheezing. Other causes of wheezing to consider include: Chronic obstructive pulmonary disease Congestive heart failure Foreign body aspiration Persistent bronchitis Upper airway cough syndrome Vocal cord dysfunction Pulmonary embolism

Which of the following are milestones of normal development that you might expect to see in a four-week-old infant? Select all that apply. A. Increased alertness and responsiveness. B. Move hands to mouth and eye range. C. Diminishment of jerky upper extremity motions. D. Can pick up head and move from one side to other when lying prone. E. Can flip from back to front while supine. F. Increased ability to focus on faces and show preference for human faces. G. Hearing maturation; beginning to turn head toward familiar sounds.

The correct answers are A, B, C, D, F, G. Flipping from back to front is generally not achieved until at least four months old (E). For more information on neonatal developmental milestones, see the Teaching Point below.

What are alternative forms of pain management for women who wish to avoid epidural use in labor? Select all that apply. A. Intradermal sterile water injections in the back B. Self-hypnosis C. Acupuncture D. IV opioids E. Oral benzodiazepines F. Water immersion G. Medical marijuana H. Inhaled nitrous oxide

The correct answers are A, B, C, D, F, H. Benzodiazepines and medical marijuana do not have any role in the management of labor pain. For more detailed teaching about the correct options, see the Teaching Point below

Which of the following are advantages of using LMWH, instead of unfractionated heparin? Select all that apply. A. LMWH has a longer biologic half-life so it can be administered subcutaneously once or twice daily. B. Laboratory monitoring is not required for LMWH. C. Thrombocytopenia is less likely with LMWH. D. Dosing is fixed with LMWH. E. LMWH reduces re-occurrence of clots more than unfractionated heparin. F. LMWH may be used in the outpatient setting.

The correct answers are A, B, C, D, F. LMWH has several advantages over unfractionated heparin: Longer biologic half-life so it can be administered subcutaneously once or twice daily Laboratory monitoring is not required Thrombocytopenia is less likely although periodic monitoring of platelets may be needed Dosing is fixed Bleeding complications are less common LMWH may be used in the outpatient setting; whereas unfractionated heparin requires hospitalization as it is administered intravenously with the dosage based on body weight and titrated based on the activated partial thromboplastin time. One advantage to unfractionated heparin is that it can be immediately shut off and reversed in the case of bleeding due to its very short half-life. In a patient with a significant bleeding risk (e.g. recent admission for gastrointestinal bleeding), it is advisable to choose unfractionated heparin over low molecular weight heparin, which has a much longer half-life once injected.

From the following, select the top six diagnoses on your differential. She has cough A. Pneumonia- bacterial B. Pneumonia- viral C. Bronchitis- acute D. Influenza E. Upper respiratory tract infection F. Asthma exacerbation G. Streptococcal pharyngitis H. Pneumonia- atypical I. Bronchiolitis

The correct answers are A, B, C, D, G, H.

Which of the following are necessary to evaluate when you suspect benign prostatic hyperplasia (BPH)? Select all that apply. A. Presence of classic lower urinary tract symptoms (LUTS). B. Examination of prostate. C. Urinalysis. D. Serum prostate specific antigen (PSA). E. Residual volume of urine. F. Urine flow rate. G. Serum BUN and creatinine.

The correct answers are A, B, C, D, G.

"What are some of the things you think I should have covered in my visit with Ms. Rios yesterday?" Select all that apply. postpartum after baby A. Mood B. Contraception C. Sexual activity D. Breastfeeding E. Glucose tolerance test

The correct answers are A, B, C, D.

Dr. Nayar tells you, "Ms. Hailey asked why Andrew's testis has torsed, and as I told her, the cause isn't usually clear. However, there are some causes we do know of for testicular torsion." Which of the following are causes of testicular torsion? Select all that apply. The best options are indicated below. Your selections are indicated by the shaded boxes. A. Congenital anomaly B. Undescended testicle C. Trauma D. Exercise

The correct answers are A, B, C, D.

What strategies can be used to prevent osteoporosis? Select all that apply. The best options are indicated below. Your selections are indicated by the shaded boxes. A. Smoking cessation B. Calcium intake of 1200 mg per day C. Weight bearing exercise

The correct answers are A, B, C, D.

Which of the following interventions or tools have been shown to be likely beneficial during labor and delivery? Select all that apply. A. Having a written birth plan B. Having a doula, birth coach, or other support person present during labor and delivery C. Attending childbirth education classes during pregnancy D. Delivery in a home-like environment

The correct answers are A, B, C, D.

Which of the following instruments may be used for screening pregnant and postpartum women for depression? Select all that apply. A. PHQ-2 B. PHQ-9 C. Edinburgh Postnatal Depression Scale D. A followed by B if positive E. GAD-7

The correct answers are A, B, C, D. There are a variety of tools available for screening for postpartum depression. The Edinburgh Postnatal Depression Scale (C) is a tool validated for use expressly to detect postpartum depression. Other acceptable options include typical screening tools for depression such as the PHQ-2 (A) and the PHQ-9 (B). If using the PHQ-2, positive responses are typically followed upon with a full PHQ-9 (D). The GAD-7 is a screening tool for anxiety-spectrum disorders and would not be useful in this setting

All doctors must take time to assess their patient's understanding of their illness for which of the following reasons? Select all that apply. A. It allows the physician to offer medical plans that resonate with the patient's priorities. B. It encourages partnership with the patient in their care. C. It reduces inappropriate clinical diagnosis and management. D. It has no effect on patient adherence to medical regimens. E. It reduces risk of misunderstanding between physicians and their patients. F. It reduces patient satisfaction with their physician.

The correct answers are A, B, C, E. "The physician serves as the expert on disease, whereas the patient experiences a unique illness. Even when the patient's and the physician's socio-cultural backgrounds are similar, substantial differences may exist because of these separate perspectives." (Ann Intern Med. 1999)

Which of the following are pain characteristics that speak against the likelihood of angina? Select all that apply. A. Pleuritic pain B. Pulsating pain C. Positional pain D. Pressure E. Palpation reproduced pain

The correct answers are A, B, C, E. Pleuritic -- worsened by respiration and may be exacerbated when lying down. Causes of pleuritic chest pain include pulmonary embolism, pneumothorax, viral or idiopathic pleurisy, pneumonia, and pleuropericarditis. Pulsating. Positional -- could be pericarditis, which typically improves with sitting up and leaning forward; pleuritic chest pain, which may be exacerbated when lying down; or musculoskeletal pain, which body position or movement may exacerbate. Reproduced by Palpation -- musculoskeletal pain.

For patients like Mr. Keenan with Alcohol Use Disorder, what are the best options for intervention and treatment? Select all that apply. A. Brief intervention by the family physician B. Referral for cognitive behavioral therapy (CBT) C. Consider treatment with medication D. Inpatient treatment program E. Intensive outpatient treatment program F. Voluntary participation in mutual support group (e.g., Alcoholics Anonymous (AA)) G. Motivational Enhancement Therapy (MET) SUBMIT

The correct answers are A, B, C, F, G. Incorrect answers include: Inpatient treatment program (D). This patient's pattern of alcohol use and absence of withdrawal symptoms suggests he does not need an intensive inpatient program. Intensive outpatient treatment program (E). An intensive program is typically used for patients with more extensive psycho-social co-morbidities.

Which of the following are appropriate options for the next step in managing Mr. Martin's blood pressure? Select all that apply. A. Initiate a calcium channel blocker B. Continue lifestyle modifications C. Continue the chlorthalidone at the current dose (25mg/day) D. Discontinue the chlorthalidone E. Increase the chlorthalidone to 50mg/day F. Initiate a beta blocker G. Initiate an angiotensin converting enzyme inhibitor (ACEI)

The correct answers are A, B, C, G. Approach to the Treatment of Hypertension Goal blood pressure for patients with hypertension is <130/80 mmHg. If a diuretic has been started and this goal is not met at follow-up visits, sequential addition of medications from the other preferred classes (calcium channel blockers, ACE-Inhibitors, Angiotensin Receptor Blockers) should be added. IMPORTANT: Always avoid combined use of an ACE-I and an ARB to avoid the risk of hyperkalemia and worsening kidney function. Throughout treatment, lifestyle modifications should be continually discussed and encouraged. There are other medications (for example: loop diuretics, beta blockers, alpha blockers) that may be used if patients are unable to tolerate medications from the preferred classes, or if goal blood pressure has not been reached despite their use (in this case, a search for causes of secondary hypertension may also be warranted).

Boy with scrotal pain Which of the following conditions are the four most likely diagnoses on your differential at this point? The best options are indicated below. Your selections are indicated by the shaded boxes. A. Trauma B. Testicular torsion C. Epididymitis D. Inguinal hernia E. Hydrocele F. Henoch-schonlein purpura G. Torsion of the appendages H. Tumor I. Varicocele J. Referred pain

The correct answers are A, B, C, G. A panel of experts selected trauma (A), testicular torsion (B), epididymitis (C), and torsion of the testicular appendages (G)as the four most likely diagnoses at this point. These will be discussed in more detail on the next card. Less Likely Diagnoses for Andrew's Groin Pain: Inguinal hernia: Andrew's history of constant exquisite pain radiating to the scrotum does not fit with a diagnosis of inguinal hernia. Hydrocele:A hydrocele is an unlikely diagnosis for Andrew as he is having acute tenderness and has no mass on exam. Henoch-Schönlein purpura (HSP): Andrew does not display any other symptoms of HSP, so this is an unlikely diagnosis for him at this point. Testicular tumor: Although important to consider in any adolescent male with a scrotal enlargement, it is unlikely to be the cause for Andrew's scrotal swelling because of the acuity of his symptoms and the absence of a mass. Varicocele:This does not explain Andrew's history of severe, acute pain. Referred pain:This is not a likely diagnosis for Andrew at this time, since he has had a previous appendectomy and has no other systemic symptoms.

Which of the following conditions are most appropriate to include on the differential diagnosis of Mr. Chen's shoulder pain at this time? Choose the top four answers. A. Rotator cuff tendinopathy B. Impingement / Subacromial bursitis C. Rotator cuff tear D. Biceps tendinopathy E. Degenerative glenohumeral arthritis F. Shoulder instability G. Acromioclavicular (AC) joint separation H. Labral tear I. Adhesive capsulitis SUBMIT

The correct answers are A, B, C, H. An appropriate differential diagnosis for Mr. Chen's shoulder pain includes: Rotator cuff tendinopathy (A) Impingement or subacromial bursitis (B) Rotator cuff tear (C) Labral tear (H)

From the following, select the top four diagnoses on your differential for low back pain in this patient. A. Lumbar strain B. Disc herniation C. Spinal stenosis D. Spinal fracture E. Cauda equina syndrome F. Pyelonephritis G. Malignancy H. Ankylosing spondylitis I. Spondylolisthesis J. Degenerative arthritis K. Prostatitis

The correct answers are A, B, C, J.

What are the management options for an inevitable abortion? Select all that apply. A. Expectant management B. Medical therapy C. Surgical management

The correct answers are A, B, C.

What are the treatment options for Mr. Payne? Select all that apply. A. Surgery B. It has only been five weeks, continue with current treatment C. Acupuncture D. Epidural injection

The correct answers are A, B, C. If a patient has been in pain for five weeks with progression of neurological deficit (such as absent reflex at the ankles) and poor pain control, it is reasonable to refer him to a spine surgeon for surgical consultation. If the patient doesn't have any red flags, continuation of conservative treatment is also an option. However, if the patient has already been getting PT, more PT is not likely to help. There is some evidence that acupuncture can be helpful in low back pain.

Which of the following are appropriate therapies for thrombotic TIA and stroke? Select all that apply. A. Aspirin B. Aspirin with dipyridamole (Aggrenox) C. Clopidogrel (Plavix)

The correct answers are A, B, C. Aspirin alone, aspirin with dipyridamole (Aggrenox), and clopidogrel (Plavix) are all used in the treatment of both embolic and non-embolic TIA and CVA. For aspirin, a low dose of 81 mg is recommended as higher doses (eg. 325 mg) increase side effects without substantially decreasing the risk of stroke. The addition of clopidogrel to aspirin is not recommended for stroke prevention as evidence shows that it dramatically increases gastrointestinal bleeding without a large enough benefit to warrant the addition. Most providers use aspirin as the first choice for platelet inhibitors, reserving clopidogrel for patients unable to tolerate aspirin.

Which pharmacologic and non-pharmacologic measures have roles in the treatment of Alzheimer's dementia and its symptoms? Select all that apply. A. Cholinesterase inhibitors B. Vitamin E C. Cognitive rehabilitation therapy D. Memantine E. Respite care F. Atypical antipsychotics for behavioral disturbances G. Ginkgo biloba

The correct answers are A, B, D, E, F.

While the patient is standing, what should be examined? Select all that apply. A. Inspect the curvature of the spine B. Palpate the paraspinal muscle C. Ask the patient to run D. Check flexion of the spine E. Ask the patient to walk F. Ask the patient to squat

The correct answers are A, B, D, E, F.

Which of the lung findings listed below would signify that there is a lung consolidation if you and Dr. Lee were to hear them? Select all that apply. A. Egophony B. Increased tactile fremitus C. Wheezing D. Dullness to percussion E. Crackles F. Rhonchi G. Whispered pectoriloquy

The correct answers are A, B, D, E, G.

Man with fatigue and red blood per rectum in spots Dr. Medel then asks you to consider a differential diagnosis for Mr. Cunha's fatigue based on your findings from his history and physical exam. From the following, select the top five diagnoses on your differential. The best options are indicated below. Your selections are indicated by the shaded boxes. A. Depression B. Obstructive sleep apnea C. Diabetes mellitus D. Anemia E. Occult malignancy F. Sleep restriction (i.e., physiologic fatigue) G. Hypothyroidism H. Coronary artery disease I. Chronic fatigue syndrome SUBMIT

The correct answers are A, B, D, E, H.

Patient not getting better on PPIs: Which of the following are appropriate next steps in diagnostic and therapeutic planning? Select all that apply. The best options are indicated below. Your selections are indicated by the shaded boxes. A. Ask about current NSAID and/or aspirin use B. Perform a digital rectal examination and guaiac-based fecal occult blood test (FOBT) C. Start tricyclic antidepressants to modulate visceral sensation and minimize pain D. Order H. pylori IgG serology E. Order an H. pylori fecal antigen test F. Obtain a complete blood count (CBC) G. Refer for a urease breath test

The correct answers are A, B, D, F The H. pylori IgG serologic test only confirms evidence of past infection and an immunologic response to H. pylori . In a population with a high prevalence of active H. pylori infection, it is a useful first-time test. However, if the prevalence of active infection is low, then the test may yield a high number of false-positive results. It should not be used to confirm eradication of H. pylori after treatment as it can remain positive for years. The urea breath test accurately detects active infection but is more expensive than serologic testing. It is less accurate during PPI therapy, and patients would need to stop the PPI for at least two weeks before a urea breath test (bismuth and antibiotics should also be stopped for at least two weeks before a urea breath test.). However, urea breath testing may be appropriate depending on the characteristics of the population being tested. The stool antigen test for H. pylori is also accurate and widely available, but it is more expensive and less convenient than serologic testing. The stool antigen and urease breath tests may also be used as confirmatory tests after a positive serologic test.

Which of the following is in your differential diagnosis for palpitations in this 50-year-old woman? Select all that apply. A. Dysrhythmia B. Valvular heart disease C. Hypothyroidism D. Hyperthyroidism (or over-replacement) E. Cushing's disease F. Anxiety / panic disorder G. Coronary heart disease (CHD) H. Vasomotor symptoms of climacteric I. Anemia J. Pulmonary embolus K. Prescription or illicit drug use

The correct answers are A, B, D, F, G, H, I, K.

For 30 yo woman with palpitations: What are the top five diagnoses on your differential at this point? Select all that apply. A. Cardiac dysrhythmias B. Anxiety / panic disorder C. Dehydration / hypovolemia D. Anemia E. Aortic stenosis F. Hyperthyroidism G. Drug abuse (including caffeine)

The correct answers are A, B, D, F, G. All of the above diagnoses should be considered given the key findings of the case to this point. From what we know so far, hyperthyroidism seems most likely. Regarding Ms. Waters' specific symptoms: Cardiac arrythmias are NOT associated with systemic symptoms such as weight loss, oligomenorrhea, and loose stools, so the presence of systemic symptoms makes this a less likely sole explanation for Ms. Waters' symptoms. Ms. Waters may have anxiety, but it doesn't explain all her symptoms at this time. Ms. Waters has none of the other symptoms of substance abuse, and it seems unlikely although would need to be considered if no other cause for her palpitations were found. She has reported a fairly low caffeine intake, but keep in mind patients may overlook sources of caffeine other than coffee and tea such as soda. Incorrect answers: Dehydration and hypovolemia (C) can cause tachycardia. However, this is usually an acute presentation and is also associated with orthostatic symptoms such as dizziness. Although aortic stenosis (E) can rarely cause palpitations, it most commonly causes syncope, exertional chest pain and lower extremity edema which are all absent here.

Dr. Nayar asks you, "What ECG changes would suggest that Mr. Reynolds has existing coronary artery disease?" Select all that apply. A. ST segment depression or downsloping ST segment B. Q waves C. U waves D. Convex ST segment elevation E. Short PR interval

The correct answers are A, B, D. The presence of a down-sloping or depressed ST segment (A) or convex ST elevation (D) suggest ischemia or injury to the myocardium respectively. Q waves would suggest that Mr. Reynolds has had an infarction. U waves (C) and a short PR interval (E) do not suggest coronary artery disease.

From the following, select the top three diagnoses on your differential. A. Knee sprain B. Osteoarthritis C. Lyme disease D. Rheumatoid arthritis (RA) E. Gout/Pseudogout F. Septic arthritis G. Baker's cyst H. Psoriatic arthritis I. Systemic lupus erythematosis

The correct answers are A, B, D. In Ms. Roman's case, a knee sprain (A), osteoarthritis (B) and rheumatoid arthritis (D) are the most likely causes of her sub-acute to chronic knee pain. Knee sprains are very common. Though they are typically associated with a history of trauma, the trauma may be minor enough that a patient may not recall it. Osteoarthritis is the most likely cause of her pain. Rheumatoid arthritis is slightly less common than OA but still can cause pain with a mild effusion (as she has). Her mention of wrist pain could also indicate that she has more joints involved, which would be consistent with RA. If this were the summer in a region where Lyme disease is endemic, Lyme would need to be considered a more likely possibility. Gout or Pseudogout typically cause acute episodes of severe pain with acute redness and swelling not seen in her case. Septic arthritis similarly causes an acutely red and warm joint. Baker's cysts are common and could present with Ms. Roman's symptoms. However, she does not have any palpable fullness in the popliteal fossa, which is the principal clinical finding of this condition. Psoriatic arthritis is less likely in a patient without findings of psoriatic plaques on her skin exam or a history of known psoriasis.

Looking at Mrs. Parker's medical record, you see that her last Pap test and mammogram were two years ago. Which of the following are recommended screening exams for women in their fifties without any specific symptoms or risk factors? Select all that apply. A. Mammogram B. Pap smear C. DEXA scan D. Colon cancer screening E. CA-125

The correct answers are A, B, D. Mammogram (A) Since Mrs. Parker is 55 and her last mammogram was 2 years ago, it is recommended that she have a mammogram. Colon cancer screening (D) Mrs. Parker should have colon cancer screening based on her age. Pap smear (B) Mrs. Parker has had no abnormal Pap smears and is at low risk for cervical cancer. Since her screening two years ago was with cytology alone, she is not due for screening with either the Pap alone, or co-testing with cytology and HPV next year.

Which of the following are the the top five items on your differential diagnosis? The best options are indicated below. Your selections are indicated by the shaded boxes. A. Chronic obstructive pulmonary disease B. Nonasthmatic eosinophilic bronchitis C. Congestive heart failure D. Pulmonary embolism E. Asthma with uncontrolled allergic rhinitis and possible chronic sinusitis F. Gastroesophageal reflux disease G. Vocal cord dysfunction

The correct answers are A, B, E, F, G. Asthma with uncontrolled allergic rhinitis and possible chronic sinusitis (E) is the most likely diagnosis as the patient presents with upper respiratory symptoms and a new onset of wheezing. Other possible, though less likely, causes of his symptoms include chronic obstructive pulmonary disease (A), nonasthmatic eosinophilic bronchitis (B), and vocal cord dysfunction (G), since all of these conditions can cause coughing and/or wheezing. Another possible cause is gastroesophageal reflux (F). Asymptomatic gastroesophageal reflux (confirmed by esophageal pH studies) can occur and can be associated with asthma.

Select the lab tests that comprise a cost-effective initial diagnostic strategy for evaluation of Mr. Cunha's fatigue. Select all that apply. A. Complete blood count (CBC) B. Serum glucose C. Liver tests D. Sleep study E. Erythrocyte sedimentation rate (ESR) F. Thyroid stimulating hormone (TSH) G. Epstein Barr virus titers H. Iron studies with ferritin

The correct answers are A, B, E, F, H. Evaluation of Mr. Cunha's Fatigue: A CBC (A) is important because of Mr. Cunha's blood with stools and pallor on exam. Mr. Cunha has no signs or symptoms of connective tissue diseases or chronic infection, but since occult malignancy is a possibility, an ESR (E) may be helpful here. Also, given his suspected chronic blood loss, iron studies (H) are appropriate. It would also be reasonable to wait for the results of the CBC, and only order these if he were anemic. The latter option is potentially cost-saving, but adds another step to the workup. Mr. Cunha's BMI is 24.1 kg/m2. Hence, he is not obese (BMI of 30 kg/m2 or higher) or overweight (BMI of 25 kg/m2 or higher, but below 30 kg/m2) and he has no family history of diabetes, so diabetes is slightly less likely than other explanations. However, diabetes is very common and can lead to fatigue. Thus a serum glucose (B) is appropriate here. Liver tests (C) are not generally part of the initial evaluation of fatigue. Mr. Cunha has no symptoms of liver disease such as abdominal distension, jaundice, or light-colored stools, and he lacks common risk factors for viral hepatitis. In Mr. Cunha's case, he presents with early exhaustion with exercise and adequate sleep, making a primary sleep disorder less likely. In Mr. Cunha's case, he lacks any other symptoms of hypothyroidism or hyperthyroidism. Additionally, hypothyroidism is ten times more common among women than men, making it unlikely in this case.

Given her intermediate pre-test probability, which of the following tests have an established role in the diagnosis of CAD in Mrs. Hernandez? Select all that apply. A. Exercise treadmill testing (ETT) B. Stress echocardiography C. Coronary artery calcium scoring (CAC Score) D. CT angiography E. Nuclear stress testing (with either Thallium or Sestamibi)

The correct answers are A, B, E.

Dr. Hill asks, "What do you think are the three most likely causes of Savannah's vaginal bleeding?" Select all that apply. A. Spontaneous abortion (miscarriage) B. Ectopic pregnancy C. Molar pregnancy (gestational trophoblastic disease) D. Vaginal trauma E. Cervical abnormalities (e.g., excessive friability, malignancy, polyps, trauma) F. Idiopathic bleeding in a viable pregnancy

The correct answers are A, B, F. There are many important causes of bleeding in early pregnancy, but the three most common are spontaneous abortion, ectopic pregnancy, and idiopathic bleeding in a viable pregnancy.

Which of the following would you order at this point. Select all that apply. A. Complete blood count B. Electrolytes, glucose, creatinine, and blood urea nitrogen (BUN) C. Thyroid studies D. Sedimentation rate E. C-reactive protein F. Arterial blood gas G. Chest x-ray H. Hemoglobin A1C

The correct answers are A, B, H. CBC (A) Leukocytosis might make you consider an infectious process Electrolytes, glucose, BUN & creatinine (B) Evaluate diabetic control and renal function Hemoglobin A1C (H) Determine diabetic control Thyroid studies (C) are unlikely to provide any useful information about the absence of signs and symptoms of thyroid disease. Sedimentation rate (D) and c-reactive protein (E) might be elevated, but would be in both cellulitis and DVT, so it is not particularly useful in determining a diagnosis at this point. Arterial blood gas (F) or chest x-ray (G) in a patient without symptoms of respiratory difficulty is unlikely to be useful.

Dr. Nayar asks you which of the following you recommend for Mr. Reynolds now. Select all that apply. A. Low dose aspirin (81 mg daily) B. Moderate- to high-intensity statin C. High sensitivity C-reactive protein (HS CRP) D. Electron-beam computerized tomography scan for coronary calcium (EBCT) E. Carotid ultrasound F. Abdominal ultrasound of aorta

The correct answers are A, B. An ASCVD risk of 17.4% places Mr. Reynolds at high risk for an ASCVD event in the next 10 years. He needs to take a low-dose aspirin daily (A) and moderate- to high-intensity statin (B) to lower his lipids. Since Mr. Reynolds is already at high risk and warrants aggressive therapy to lower his lipids, HS CRP (C) or EBCT (D) results would not change his management. USPSTF review in 2014 gave a D recommendation for carotid ultrasound in asymptomatic individuals (E). USPSTF review in 2014 gave a B recommendation for one-time abdominal ultrasound of the aorta (F) in males ages 65 to 75 who have smoked. Mr. Reynolds is only in his 50s, so it is not recommended now for Mr. Reynolds.

In addition to physical therapy, what treatments do you plan to recommend to Mr. Chen today for his shoulder condition? Select all that apply. Has rotator cuff sprain and impingement syndrome A. Relative rest from aggravating factors such as his softball playing and painting B. Pain medication as needed in topical and/or oral form C. Sling immobilization D. Urgent orthopedic surgical consultation for suspected complete tear of the rotator cuff E. X-ray of the shoulder F. MRI of the shoulder G. Subacromial injection SUBMIT

The correct answers are A, B. Recommended therapies: Relative rest (A) If Mr. Chen does not rest from some of his repetitive overhead activity, he runs the risk of worsening impingement, tendinopathy and ultimately a tear of the rotator cuff. Pain medication as needed in topical and/or oral form (B) Mr. Chen reports occasional acetaminophen use-you could trial him on a maximal dose of acetaminophen (noting no known liver disease) of 1000mg four times daily as needed and tolerated; or, alternatively trial him on a NSAID such as ibuprofen.

Mr. Payne asks Dr. Lee: "What's the likelihood that this pain will go away completely?" Select all that apply. A. Most back pain is improved in 4 to 6 weeks. B. It is more common for patients with psychosocial distress to recover. C. Longer time to recovery is associated with older patients. D. Recurrence rate for back pain varies from 35% to 75%.

The correct answers are A, C, D

From the following, select the top six diagnoses on your differential diagnosis. Brown discharge in pregnant woman after having sex A. Low-lying placenta and/or placenta previa B. Premature rupture of the membranes (PROM) C. Placental abruption D. Bacterial vaginosis E. Vaginal candidiasis F. Urinary tract infection G. Cervical trauma H. Preterm labor I. Uterine rupture

The correct answers are A, C, D, E, F, G. After considering your differential, you determine that it is most likely that the discharge is from bacterial vaginosis caused by a shift in the normal bacterial flora of the vagina. Recent history of intercourse and the presence of dysuria support this diagnosis. It is also possible that Ms. Rios has vaginal candidiasis or a urinary tract infection, as both of these diagnoses could present with dysuria in the setting of recent sexual intercourse. Brownish vaginal discharge is not typically seen with either of these conditions, however could indicate concomitant intercourse-related cervical trauma. Ms. Rios' brownish discharge could also indicate third trimester bleeding, which complicates 3-4% of pregnancies, so you keep placental abruption and placenta previa on the differential because both of these can be life-threatening to both mother and fetus. However, these conditions typically cause bright red blood and neither cause dysuria. PROM is less likely here since Ms. Rios doesn't describe loss of fluid and your exam reveals slightly watery brownish, rather than clear, fluid in the vault. Ms. Rios' brownish vaginal discharge could indicate a bloody show associated with preterm labor, but she is not experiencing uterine contractions, so this is unlikely. Ms. Rios does not have risk factors (such as a prior cesarean section) and would be expected to demonstrate heavier bleeding associated with pain if she were experiencing a uterine rupture.

You ponder for a moment how her treatment would be different if she had systolic heart failure (sometimes referred to as Heart Failure with Reduced Ejection Fraction, or HFrEF). Select from the following list all the pharmacologic treatments that have a role in the management of Grade C systolic heart failure (i.e. structural heart disease with prior or current symptoms of HF). Select all that apply. The best options are indicated below. Your selections are indicated by the shaded boxes. A. Angiotensin converting enzyme (ACE) inhibitors B. Amlodipine (Norvasc) C. Angiotensin receptor blockers (ARBs) D. Digoxin E. Loop diuretics F. Metoprolol succinate (Toprol-XL) G. Pioglitazone (Actos) for improved glucose control in type II diabetes H. Eplerenone

The correct answers are A, C, D, E, F, H. The 2017 guidelines recommend as the mainstay of medical therapy for patients with chronic HFrEF inhibition of the renin-angiotensin system with ACE inhibitors, ARBs, or the newer angiotensin receptor-neprilysin inhibitor (ARNI) in conjunction with beta blockers, and aldosterone antagonists in selected patients, to reduce morbidity and mortality.

Dr. Hill then asks you to consider your differential diagnoses for Mr. Fitzgerald's skin condition on his left forearm (oval-shaped, erythematous 18 mm x 16 mm patch), based on your findings from his history and physical examination. From the following, select the top six diagnoses on your differential. A. Eczema (dermatitis) B. Psoriasis C. Squamous cell carcinoma of the skin D. Actinic keratosis E. Basal cell carcinoma of the skin F. Melanoma G. Lichen planus H. Seborrheic Keratosis I. Fungal skin infection

The correct answers are A, C, D, E, F, I. Psoriasis is unlikely in this case because. Mr. Fitzgerald's skin lesion is unilateral and on the flexor aspect of his left forearm. Lichen planus is unlikely because Mr. Fitzgerald's skin lesion is a unilateral, well demarcated patch. Mr. Fitzgerald's lesion is not characteristic of seborrheic keratoses.

What are important guidelines to use when documenting a case of suspected domestic abuse? Select all that apply. A. Clearly state names of all involved in note and utilize "patient states" to provide clear documentation in the event of a court case. B. Do not include abuse history in subjective as there is no way to verify. C. Utilize a body map to document injuries on physical exam using a blue pen. D. Maintain a separate file for all laboratory and radiology tests ordered as these could be used against the patient in a court case. E. Document recommendations for support and follow up as well as materials given to the patient. F. Do not document any specific plans for referral, support or materials given as this could increase the patient's risk.

The correct answers are A, C, E.

Which of the following are symptoms of acute sinusitis? Select all that apply. The best options are indicated below. Your selections are indicated by the shaded boxes. A. Fever B. Clear nasal drainage C. Headaches D. Facial Pain E. Toothache F. Gradual worsening five days after the onset of a viral upper respiratory infection G. Failure to respond to decongestants H. Initial improvement after a viral upper respiratory infection and then a reoccurrence of worsening symptoms I. Nasal congestion or obstruction for more than 12 weeks

The correct answers are A, C, D, E, G, H. Fever (A), headache (C), facial pain (D), toothache (in the maxillary teeth) (E), failure to respond to decongestants (G), and double worsening (the pattern of initial improvement from a viral infection followed by an acute worsening) (H) are all symptoms consistent with acute sinusitis, though none are pathognomonic for the condition. Clear nasal discharge (B) is not typical of acute sinusitis. Gradual worsening in the first five days after the onset of a URI (F) is fairly typical for the common cold, not sinusitis. Nasal congestion lasting more than 12 weeks (I) is consistent with chronic sinusitis, not acute.

Dr. Medel asks you, "Which elements of an initial focused history are most important to gather in a patient with a possible new diagnosis of hypertension?" Select all that apply. A. History or symptoms of congestive heart failure B. History of cancer C. History or symptoms of cardiovascular disease D. History of diabetes E. History of high cholesterol F. History of glaucoma G. Family history of cardiovascular disease H. Family history of colon cancer I. Medications, including over-the-counter and complementary medications J. Smoking, alcohol, and/or drug use

The correct answers are A, C, D, E, G, I, J. History of cancer (B), history of glaucoma (F), and family history of colon cancer (H) are not essential elements of an initial focused history for a patient with a possible new diagnosis of hypertension. These questions are appropriate, however, for a more general medical history.

For which of the following cancers does the USPSTF recommend screening (with an A or B level of recommendation)? Select all that apply. The best options are indicated below. Your selections are indicated by the shaded boxes. A. Breast cancer B. Prostate cancer C. Lung cancer D. Colorectal cancer E. Cervical cancer F. Pancreatic cancer G. Testicular cancer H. Skin cancer SUBMIT

The correct answers are A, C, D, E.

Which of the following are TRUE regarding dyspepsia due to GERD or PUD? Select all that apply. A. GERD can be distinguished from other gastrointestinal disorders with reasonable accuracy on the basis of symptoms. B. Eating and drinking make GERD symptoms improve and PUD symptoms worse. C. Peptic ulcers may be associated with nausea and vomiting that can occur anytime shortly after eating up to several hours later. D. Non-erosive reflux disease (NERD) is the most common form of GERD. E. Patients with GERD report lower health-related quality of life than patients with heart failure.

The correct answers are A, C, D, E.

From the following, select the top four diagnoses on your differential. Painful menstruation and menorrhagia A. Adenomyosis B. Cervical stenosis C. Chronic pelvic inflammatory disease D. Endometriosis E. Fibroids F. Inflammatory bowel disease G. Irritable bowel syndrome H. Ovarian cysts I. Psychiatric problems J. Uterine polyps

The correct answers are A, C, D, E. Most Likely / Important Diagnoses A panel of experts selected the following as the most likely diagnoses at this point: adenomyosis (A), chronic pelvic inflammatory disease (C), endometriosis (D), fibroids (E). More about these conditions on the following page.

What are the possible etiologies for new onset CHF? From the following, select the top four diagnoses on your differential for Mrs. Hernandez's presenting symptoms and signs on history and physical exam. A. Myocardial infarction (MI) B. Anemia C. Arrhythmia D. Ischemic cardiomyopathy E. Non-ischemic cardiomyopathy F. Pulmonary embolism G. Diastolic dysfunction due to uncontrolled hypertension H. Thyroid dysfunction I. Valvular disease

The correct answers are A, C, D, G. Mrs. Hernandez may have CHF due to ischemic cardiomyopathy given her history of multiple CAD risk factors (age, poorly controlled hypertension and diabetes). Her CHF could be due to diastolic dysfunction only (especially considering her non-adherence to anti-hypertensives). She may have suffered a silent MI that precipitated the development of her symptoms, or she may be suffering from an intermittent cardiac arrhythmia that is impairing the pump function of her heart.

Dr. Lee then asks you: "Which of the following findings would support the diagnosis of disc herniation?" Select all that apply. A. Pain worse with sitting B. Pain worse with standing C. Pain worse with cough and sneezing D. Drop foot E. Urinary retention

The correct answers are A, C, D.

What are the major anatomic stabilizers of the shoulder joint? Select all that apply. A. Labrum B. Teres major muscle C. Rotator muscle group D. Glenohumeral ligaments E. Brachial plexus

The correct answers are A, C, D.

Which of the following are effective treatments for premenstrual syndrome? Select all that apply. The best options are indicated below. Your selections are indicated by the shaded boxes. A. Danazol B. Hysterectomy C. Oral contraceptives D. Selective serotonin reuptake inhibitors (SSRIs) during menses E. Spironolactone F. Vitamin B6

The correct answers are A, C, D. Danazol is an androgenic medication with progesterone effects. It lowers estrogen and inhibits ovulation. However, its multiple androgenic side effects, including weight gain, suppressing high density lipids, and hirsutism, limit its desirability among patients. GnRH agonists, such as leuprolide, are effective at treating premenstrual syndrome through ovulation inhibition. However, their anti-estrogen effects, including hot flashes and vaginal dryness, make these not as popular. Oral contraceptives are effective treatment for dysmenorrhea, anovulation, and in some cases menorrhagia. While not always effective for premenstrual syndrome, they are a good place to start. It would be appropriate to try this in a woman also needing birth control. The most favorable pill is the formulation containing ethinyl estradiol and drospirenone. One study demonstrates potential improved effectiveness by decreasing the placebo pills to four days from seven. Selective serotonin reuptake inhibitors during menses are an effective treatment of premenstrual syndrome, especially if severe or mood symptoms predominate. There are three effective regimens for SSRI use. One regimen is continuous daily treatment. Another is intermittent treatment, which is just as effective as daily treatment for decreasing both psychologic and physical symptoms. There are two types of intermittent treatment. One method is to start therapy 14 days prior to menses (luteal phase of cycle) and continue until menses starts. The second method is to start on the first day a woman has symptoms and continue until the start of menses or three days later. Many randomized trials have used fluoxetine and sertraline. Venlafaxine can be used as well. Lower doses are effective. If one medication does not work, another in the same class should be tried prior to considering the treatment a failure. Follow-up should occur after two to four cycles. Intermittent treatment is associated with fewer side effects and lower cost.

Which of the following are indicators that an antibiotic would be helpful for a patient with a diagnosis of an acute exacerbation of COPD? Select all that apply. The best options are indicated below. Your selections are indicated by the shaded boxes. A. Increased dyspnea B. Low-grade fever C. Increased phlegm production D. Change in sputum color E. O2 saturation of 92% with walking 100 feet F. Pleuritic chest pain SUBMIT

The correct answers are A, C, D. Anthonisen et al. studied patients presenting with an acute exacerbation of chronic bronchitis (AECB). They examined how the symptoms of increased dyspnea (A), increased sputum (phlegm) production (C), and a change in color of sputum (D) predicted response to an antibiotic. This classic study found that in the group who presented with an exacerbation characterized by the presence of all three symptoms, 44% improved in the antibiotic group versus 31% in the placebo group. Also, only 10% of patients in the antibiotic group experienced deterioration of condition compared to 22% in the placebo group. Low-grade fever (B), decreased O2 saturation of 92% with walking 100 feet (E), and pleuritic chest pain (F) are not indications for antibiotic treatment without making a diagnosis in addition to an acute exacerbation of chronic bronchitis. A fever might indicate pneumonia, which should be confirmed by chest x-ray, though a low-grade fever could represent a viral infection. And decreased oxygen saturation might also indicate pneumonia. Antibiotics would be prescribed for a pneumonia, but- these findings in the context of AECB do not support use of an antibiotic. Pleuritic chest pain, especially in concert with acute hypoxemia, could indicate a pulmonary embolism

Which of the following elements may comprise a comprehensive foot examination for a diabetes patient? Select all that apply. A. Evaluate the feet for sensation to touch and vibration B. Elicit patellar reflexes bilaterally C. Palpate dorsalis pedis and posterior tibial pulses bilaterally D. Measure the surface temperature of the feet bilaterally E. Assess the skin for color and temperature changes, hair loss and scaling F. Inspect the feet for abrasion, callus formation ulceration, infection G. Inspect the feet for bony abnormalities H. Elicit Achilles reflexes bilaterally

The correct answers are A, C, E, F, G, H. Eliciting ankle jerk reflexes is an important part of a comprehensive diabetic foot exam. A decline in patellar reflexes (B) would not occur due to diabetic neuropathy until very late in the course of the disease and is not a part of a diabetic exam. Measuring skin surface temperature (D) is not a reliable finding. For more about the comprehensive diabetic foot exam, see the Teaching Point below.

In the hallway, Dr. Wilson asks you, "Since Ms. Sanchez has not been seen for her diabetes for over a year. Which of the following diagnostic tests are appropriate with respect to her diabetes?" Select all that apply. A. Hemoglobin A1C B. 24-hour urine protein and glucose C. Spot urine albumin/creatinine ratio D. Fingerstick blood glucose E. Serum creatinine and calculated GFR F. Serum B12 levels G. Thyroid-stimulating hormone (TSH) H. Fasting lipid profile (total cholesterol, LDL- and HDL-cholesterol and triglycerides)

The correct answers are A, C, E, F, G, H. Ms. Sanchez would be due for a hemoglobin A1C (A), spot urine albumin/creatinine ratio (C), serum creatinine and GFR (E), Serum B12 level (F), TSH (G), and lipid profile (H). Lipid profiles have historically been done fasting, though there is growing evidence that they may be done without this restriction. For more detailed information, see the Teaching Point below.

What labs and radiology studies would you like to order now? Select all that apply. The best options are indicated below. Your selections are indicated by the shaded boxes. A. Complete blood count B. Computed tomography (CT) scan C. Human chorionic gonadotropin D. Magnetic resonance imaging (MRI) E. Pelvic ultrasound F. Thyroid-stimulating hormone G. Von Willebrand's testing

The correct answers are A, C, E, F.

Which of the following features would suggest a diagnosis of Down syndrome? Select all that apply. A. Flat facial profile B. Normal Moro reflex C. Slanted palpebral fissures D. Hypertonia E. Anomalous ears F. Transverse palmar crease SUBMIT

The correct answers are A, C, E, F. Down syndrome is characterized by a number of physical features, many of which affect the appearance of the head, neck, and extremities. Not all features are present in a given individual, but most infants with this syndrome have between 4 and 6 of the following signs: Flat facial profile Poor Moro reflex Excessive skin at nape of the neck Slanted palpebral fissures Hypotonia Hyperflexibility of joints Dysplasia of pelvis Anomalous ears Dysplasia of midphalanx of fifth finger Transverse palmar (Simian) crease

Which of the following agents have been proven to cause or contribute to the development of peptic ulcer disease? Select all that apply. A. Aspirin B. Acetaminophen C. Ibuprofen D. Psychosocial stress E. Moderate physiologic stress F. Caffeine G. Cigarette smoking H. Helicobacter pylori

The correct answers are A, C, E, G, H. Aspirin and other non-steroidal anti-inflammatory drugs (NSAIDs) are the predominant pharmacologic agents that contribute to the development of PUD. Classically, the elderly are at the highest attributable risk of ulceration and perforation due to chronic NSAID use. Chronic NSAID use is a leading cause of morbidity in the elderly. Moderate to severe physiologic stress may lead to stress ulceration, predominantly in patients in the intensive care unit (ICU). Colonization of the stomach by H. pylori renders the underlying mucosa more vulnerable to peptic acid damage by disrupting the mucous layer, liberating enzymes and toxins, and adhering to the gastric epithelium. In addition, the body's immune response to H. pylori incites an inflammatory reaction that contributes to tissue injury and leads to chronic gastritis. In most individuals the chronic gastritis is asymptomatic and does not progress. In some cases, however, altered gastric secretion coupled with tissue injury leads to peptic ulcer disease. In other cases, gastritis progresses to mucosal atrophy, intestinal metaplasia, and eventually gastric carcinoma. Rarely, persistent immune stimulation of gastric lymphoid tissue can lead to gastric lymphoma. There is no evidence to support a cause-and-effect association between cigarette smoking and PUD. However, cigarette smoking does decrease vascularity to gastric mucosal cells, resulting in decreased rates of mucosal healing after insult, and in combination with NSAID use or H. pylori infection, increases the risk of ulceration.

Based on your findings so far, select the top four diagnoses on your differential diagnosis. The best options are indicated below. Your selections are indicated by the shaded boxes. A. Acute bronchitis B. Angina C. Asthma D. Bronchiectasis E. Chronic obstructive pulmonary disease (COPD) F. Congestive heart failure (CHF) G. Lung cancer H. Panic disorder I. Pneumonia J. Pulmonary embolism (PE) K. Pulmonary tuberculosis

The correct answers are A, C, E, G.

Which immunizations would be appropriate to administer to her at this time? When reviewing the immunization record, assume that it is Fall 2009 and Savannah is 16 years old. Select all that apply. A. Tetanus, Diphtheria, Pertussis (Tdap) B. Human Papillomavirus (HPV) C. Meningococcal (MCV) D. Hepatitis A E. Hepatitis B F. Inactivated Polio Virus (IPV) G. Measles, Mumps, and Rubella (MMR) H. Varicella

The correct answers are A, C, H. Appropriate Vaccination Administration Tetanus, Diphtheria, Pertussis (Tdap) (A)- It is time for Savannah's Td booster. The Tdap would be recommended to provide additional coverage for Pertussis. Meningococcal (MCV) (C) - A recommended vaccine for Savannah since she did not have this vaccine at her 11-year-old visit. Varicella (H)- A second dose is recommended Savannah had her HPV, Hep A, Hep B series, as well as her MMR and IPV series (Answer options B, D, E, F, &G).

Which of the following provides evidence that a woman may be in active labor? Select all that apply. The best options are indicated below. Your selections are indicated by the shaded boxes. A. Strong regular contractions that are difficult to walk or talk through. B. Cervical exam with greater than 4 cm of dilation C. Cervical change in the setting of regular contractions D. Normal fetal heart tones without decelerations SUBMIT

The correct answers are A, C. It is important to diagnose women with active labor prior to admitting them to the hospital for expectant management. Admitting women prior to active labor, in the latent labor stage, is associated with more interventions, including augmentation of labor (e.g. with medications such as oxytocin or other interventions) and cesarean delivery. Active labor is associated with strong regular contractions every three to five minutes and a cervical dilation of more than 6 cm in the setting of contractions. The fetal heart tracing does not impact the diagnosis of active labor.

What do you think are absolute contraindications to a digital cervical examination? Select all that apply. A. Patient report of vaginal bleeding with an undocumented placental location (or a known low-lying placenta or placenta previa.) B. An abnormal fetal heart rate tracing with multiple late decelerations. C. Patient report of leaking vaginal fluid with known prematurity (or known preterm premature rupture of membranes - PPROM.) D. Active labor in a patient with very painful contractions.

The correct answers are A, C. Neither an abnormal fetal heart tracing (B) nor painful contractions (D) are reasons to defer a digital cervical exam, and, in fact, may be reason to do an exam to gather important information that will impact the management of the patient in either case. Both patient reports of vaginal bleeding with an undocumented placental location (or known previa) AND leaking vaginal fluid with prematurity (or known PPROM) are absolute contraindications to digital cervical exam secondary to harm that may be caused-worsening of bleeding in the first case, and introduction of bacteria into the uterus potentially leading to infection, in the second.

Which of the following are risk factors that Mrs. Parker has for endometrial cancer? Select all that apply. A. Obesity B. Diabetes mellitus C. Arterial hypertension with or without atherosclerotic heart disease D. History of breast or colon cancer

The correct answers are A, C. Obesity (A) and hypertension (C) are risk factors that Mrs. Parker has for endometrial cancer. Although Mrs. Parker has a family history for diabetes (B), and it is a risk factor for endometrial cancer, she does not herself have diabetes at this time. History of breast or colon cancer (D) are risk factors for endometrial cancer that Mrs. Parker does not have. Other risk factors for endometrial cancer for Mrs. Parker that were not listed in the question are HRT, early menarche (age 11), and age.

Which of the following physical examination findings support the diagnosis of COPD? Select all that apply. The best options are indicated below. Your selections are indicated by the shaded boxes. A. Increased AP diameter B. 2/6 systolic murmur loudest at the right upper sternal border (RUSB) C. 1+ pretibial pitting edema D. Laryngeal height 2 cm above the sternal notch, upon full expiration E. End-expiratory wheezing

The correct answers are A, D, E.

Which of the following can be helpful in establishing breastfeeding between a mother and infant? Select all that apply. The best options are indicated below. Your selections are indicated by the shaded boxes. A. Avoiding pacifier use. B. Avoiding too many weight checks and office visits to give mother and baby time to establish the breastfeeding relationship at home. C. Delaying skin-to-skin contact until after the first bath. D. Educating the patient and her partner about breastfeeding, especially early in pregnancy when the decision to breastfeed is often made. E. Lactation consultation with a certified lactation consultant, especially if low milk supply is an issue.

The correct answers are A, D, E. To encourage breastfeeding, pacifiers and supplements should be avoided in the first few weeks of life, mothers and newborns should ideally stay in the same room in the hospital, and infants should only be fed on demand. Education of expectant women and their partners (like breastfeeding classes during pregnancy) has been shown to help increase the rate of breastfeeding initiation and exclusive breastfeeding at six months. Lactation consultation by a certified lactation specialist can also be very helpful for breastfeeding mothers, and should be considered routinely for first-time breastfeeders, small for gestational age infants, preterm infants, and any other circumstance where breastfeeding could be challenging. Breastfeeding infants with slow or no weight gain should be seen frequently to asses for failure to thrive, jaundice or other problems. Frequent office visits can also provide additional support and education for women desiring to breastfeed their infants. Immediate skin-to-skin contact after delivery is a very important step in establishing breastfeeding in mother-baby pairs, and there is good evidence to support this practice. Early skin-to-skin contact and early initiation of breastfeeding is associated with increased rates of breastfeeding at hospital discharge and later in infancy.

What are the most common causes of dementia? Choose the three best answers. The best options are indicated below. Your selections are indicated by the shaded boxes. A. Alzheimer's disease B. Parkinson's disease C. Pick's disease D. Vascular dementia E. Huntington's disease F. Creutzfeldt-Jakob disease G. Dementia with Lewy bodies

The correct answers are A, D, G.

In patient with PUD Next, she asks: "Which two of the following options are the most appropriate first steps in diagnostic testing and therapeutic planning for this patient?" Choose the two best answers. The best options are indicated below. Your selections are indicated by the shaded boxes. A. Using an empiric treatment strategy with a proton pump inhibitor (PPI) B. Referring the patient for an upper endoscopy (esophagogastroduodenoscopy / EGD) C. Ordering an upper GI series (barium swallow radiograph) D. Focusing on lifestyle modifications to promote symptomatic improvement E. Referring the patient for a 24-hour pH probe

The correct answers are A, D.

What are always abnormal findings on a pelvic exam? Select all that apply. A. 12-week size uterus when not pregnant B. Mildly tender ovaries to palpation C. Nabothian cysts on the cervix D. Nonmobile uterus E. Scant white vaginal discharge

The correct answers are A, D. Unless a woman is pregnant, a normal uterus in not larger than eight weeks in size, approximately the size of a clenched fist. A normal uterus may be mildly tender on exam just prior to or during menses. A normal uterus can be tilted anteriorly (anteverted or anteflexed), midline, or tilted posteriorly (retroverted or retroflexed). An anteflexed or retroflexed uterus may be difficult to assess for size because of its position. The uterus should be smooth in contour around the entire surface area. Serosal fibroids or large mucosal fibroids may cause a "knobby" feel to the uterus.

Which of the following are characteristics of migraine headaches? Select all that apply. A. Moderate to severe pain. B. Pressing or tightening bilateral pain. C. Alleviated by physical activity. D. Accompanied by nausea and vomiting. E. Last 30 minutes to seven days. F. Ten episodes are necessary for diagnosis.

The correct answers are A, D. - last from 4-72 hours

Prior to her discharge from the hospital, how would you approach Mrs. Gold regarding her breastfeeding concerns? Select all that apply. She feels she isn't producing milk and hasn't breast fed A. "Giving the baby formula could delay your milk production further because the baby will be too full to nurse." B. "Go ahead and give your baby a little formula if you're worried. Then you'll be able to sleep a little more as well." C. "I wouldn't give your baby any formula, or you'll never be able to breast feed." D. "Tell me more about your concerns. Did you breastfeed your last baby? What was your experience then? What was your mother's experience with breastfeeding?"

The correct answers are A, D. Addressing Breastfeeding Concerns Ideally, include the patient and her mother or other support people that the patient may turn to for advice. Educate them about the process of breastfeeding initiation. Approach A gives clear advice and explains the reasoning behind it. Giving supplemental formula prematurely can actually impair women's full milk production because when the infant gets full from formula, he will not nurse as much at the breast and the decreased stimulation will lead to decreased milk production. Approach D is a nice start in that it is an attempt to find out more about where the patient is coming from so that you can give her directed education. Approach B is not recommended, as giving formula in the hospital when not indicated for severe weight loss or other concerns can decrease breastfeeding rates. Approach C is probably a little too authoritative and does not really explain why you are recommending no formula to the patient.

Right-hand dominant patients who have strokes in the area of the brain that contributes to the behavior of Mr. Wright are likely to also have which of the following associated behaviors or functional impairments? Select all that apply. He is displaying neglect A. Left hemiplegia B. Impairment of spontaneous respirations C. Receptive aphasia D. Expressive aphasia E. May attempt to read while holding books upside down F. Inattention to areas of a room G. Denial of stroke disability H. Left facial weakness

The correct answers are A, E, F, G, H. Right-hand dominant patients with strokes in the area of the brain are likely also to have left hemiplegia. Patients with right middle cerebral infarcts affecting the right parietal hemisphere may have difficulties with their spatial and perceptual abilities, which causes them to misjudge distances, or they may attempt to read holding books upside down. They may ignore people or objects in their left visual field or not pay attention to that area of the room. They may also not recognize their functional impairments (denial of stroke disability).

"Now that you have a diagnosis of disc herniation with radiculopathy for Mr. Payne, let's discuss what would you like to do for him." Which of the following are indicated at this time? Select all that apply. A. Prescribe NSAID and muscle relaxant B. Strict bed rest C. Referral to a back surgeon D. Order a MRI E. Moist heat F. Referral to physical therapy

The correct answers are A, E, F. Mr. Payne has no red flags and has had pain for only two weeks, so conservative therapy is appropriate for him.

What are the risk factors for primary dysmenorrhea? Select all that apply. A. Depression or anxiety B. Increasing parity C. Lower socioeconomic class D. Menarche late in adolescence E. Smoking

The correct answers are A, E.

For pregnant patient at 10 weeks, Which of the following would be appropriate to complete or give today? Select all that apply. A. Influenza vaccination B. Rhogam immunization C. MMR (measles, mumps, rubella) immunization D. Hepatitis B core antigen testing E. Repeat HIV testing F. Routine ultrasound for anatomy G. Serum marker screening for neural tube defects H. Serum marker screening for trisomy 21 I. Serum marker screening for trisomy 18 J. Counseling about screening and prenatal diagnosis of fetal anomalies

The correct answers are A, J. Vaccinations for Ms. Rios: Influenza vaccine (A) is recommended for all pregnant women. Ms. Rios is Rh negative and needs immunization to prevent the development of antibodies to a fetus with Rh positive blood in a future pregnancy if her current fetus is Rh positive (B). However, now is not the appropriate time for this. Rubella immunization (C) after completion of her current pregnancy. Counseling and screening for fetal anomalies for Ms. Rios: Although serum screening is not usually performed until approximately 15-21 weeks' gestation, it would be appropriate to begin a conversation today about whether or not Ms. Rios would like to have screening performed when the time comes. Further Hepatitis B serology testing or HIV testing are not necessary unless new risk factors arise.

Mr. Roberts is a 78-year-old male with a significant past medical history of chronic kidney disease stage II, coronary artery disease, and hypertension who presents lumbar back pain. He has also been feeling general malaise and chills over the past few days. On review of symptoms he reports having some difficulty urinating with hesitancy and pain on urination. Currently, his chronic conditions are well managed with metoprolol, lisinopril, and aspirin. Vital signs are temperature 100.2 F, blood pressure 135/75, pulse 76/min, and respiratory rate 15/min. Given this history, which of the following physical exam maneuvers are most helpful in making the diagnosis? Select all that apply. A. Abdominal palpation B. Costovertebral angle percussion C. Straight leg test D. Digital rectal exam E. Pinprick sensation of the legs

The correct answers are B and D. In an older male patient, prostatitis may present with low back pain. This patient's symptoms-general malaise, chills, hesitancy and pain on urination-and signs (fever) suggest acute bacterial prostatitis. Patients with acute bacterial prostatitis will often have exquisite tenderness over the prostate on rectal exam. This patient could also have pyelonephritis, which often goes along with costovertebral angle tenderness.

When Savannah has left, Dr. Hill asks you, "What lab tests do you think I should order as part of the initial pregnancy workup plan?" Select all that apply. A. Serum pregnancy test B. Complete blood count C. Rubella antibodies D. Hepatitis B surface antigen E. Type & screen F. RPR G. HIV H. Ultrasound

The correct answers are B, C, D, E, F, G.

In addition to the pelvic ultrasound, which of the following laboratory tests would be reasonable to obtain at this juncture? Select all that apply. Has 1st trimester bleeding A. Type and screen (Rhesus typing) B. Complete blood count (CBC) C. PCR for gonorrhea and chlamydia D. Wet mount preparation E. Progesterone F. Quantitative beta-human chorionic gonadotropin (quant. beta-hCG)

The correct answers are B, C, D, E, F. its usefulness to detect infection (and thus a septic abortion) during pregnancy because most pregnant patients have a mild leukocytosis. Nevertheless, if significantly elevated, or associated with a bandemia, this test would need to be factored into the consideration of a septic abortion. Wet mount preparation for trichomonas, as well as PCR testing for gonorrhea and chlamydia: All sexually transmitted infections can cause vaginal bleeding. These tests should be obtained in this clinical context, despite a previously normal recent result. Progesterone: Laboratory testing for progesterone is most useful in extreme situations. If the result is >25, it is highly associated with a sustainable intrauterine pregnancy. If the result is <5, it is highly associated with an evolving miscarriage or ectopic pregnancy. Levels between 5 and 25 have minimal diagnostic value in distinguishing intrauterine from ectopic pregnancy. Algorithms for the diagnosis of ectopic pregnancy emphasizing progesterone measurements have been associated with a higher use of surgical management and often miss ectopic pregnancy since 85% of ectopic pregnancies will have a normal progesterone level. Nevertheless, the test remains valuable because of its positive and negative predictive value at the extremes of the reference range. In many labs, it is a common and quick test, which makes it frequently ordered. Quantitative beta-human chorionic gonadotropin (quant. beta-hcg): This test has enormous significance, and when combined with the pelvic ultrasound, they are the definitive diagnostic modalities. However, in isolation, one beta-hCG can be challenging to interpret, especially without the ultrasound results. Human chorionic gonadotropin is secreted by the trophoblastic cells very early in embryonic life (day 7, post-ovulation). Additionally, testing for the beta-subunit is exquisitely sensitive (down to 5 mIU/mL) and specific (the placenta is the only normal tissue that excretes beta-hCG). By the expected date of menses, the beta-hCG is usually > or = 100 mIU/mL.

Which of the following are topics or health conditions addressed by GAPS? Select all that apply. The best options are indicated below. Your selections are indicated by the shaded boxes. A. Preventing diabetes mellitus B. Preventing hypertension C. Preventing hyperlipidemia D. Promoting safety and injury prevention E. Promoting physical fitness

The correct answers are B, C, D, E.

Which of the following are treatment options for osteoporosis? Select all that apply. A. Prednisone B. Alendronate (Fosamax) C. Parathyroid Hormone (Forteo) D. Raloxifene E. Calcitonin SUBMIT

The correct answers are B, C, D, E.

Which of the following would be appropriate to the evaluation of Mrs. Parker's bleeding? Select all that apply. A. Follicle-stimulating hormone (FSH) and luteinizing hormone (LH) levels B. Transvaginal ultrasound C. Endometrial biopsy D. Complete blood count E. Thyroid-stimulating hormone (TSH) level

The correct answers are B, C, D, E.

What are the red flags or alarm symptoms that would suggest a more serious underlying condition causing his back pain? Select all that apply. A. The worst pain the patient has ever had B. Fever C. Loss of bowel/bladder control D. Severe pain that awakens him from sleep. E. Numbness of the leg F. Weight loss

The correct answers are B, C, D, F.

Which of the following interventions have been shown to have a role in the management of coronary artery disease? Select all that apply. The best options are indicated below. Your selections are indicated by the shaded boxes. A. Aim for a hemoglobin A1c (HgA1C) of 6% or less B. Continue treatment with angiotensin-converting enzyme (ACE) inhibitor C. When appropriate, treat with aspirin D. Treat with a statin E. Use ezetimibe to bring her LDL cholesterol to less than 70 mg/dL F. Encourage weight loss to bring her body mass index to 24 kg/m2 as soon as possible G. Provide a pneumococcal vaccine every five years H. Add a beta-blocker even if her blood pressure is normal

The correct answers are B, C, D, H.

Which of the following statements are true about the diagnosis of pregnancy? Select all that apply. A. Home pregnancy tests are not a reliable indication of pregnancy. B. The presence of a delayed menstrual period in a woman with previously regular periods and symptoms of nausea, breast tenderness, and fatigue are classical indications of pregnancy. C. Blood tests of human chorionic gonadotropin (hCG) can identify pregnancy as early as 8 days post-conception (about 1 week prior to a missed period). D. Transvaginal ultrasound is a useful method for dating early pregnancy, but is not typically used to diagnose pregnancy.

The correct answers are B, C, D.

What are common side effects associated with selective serotonin reuptake inhibitors (SSRIs) and serotonin-norepinephrine reuptake inhibitors (SNRIs)? Select all that apply. A. Cardiac arrhythmias B. Headaches C. Insomnia D. Nausea E. Arthralgias

The correct answers are B, C, D. Common side effects of SSRI/SNRIs include: Headaches Sleep disturbances (drowsiness and, less frequently, insomnia) Gastrointestinal problems such as nausea and diarrhea Sexual dysfunction Older antidepressants such as TCAs can cause arrhythmias. Citalopram and Escitalopram can cause QT interval prolongation at higher doses, especially in the face of hypokalemia and hypomagnesemia or when combined with other medication that have this same effect. Reports of symptomatc arrythmia is uncommon. Often patients with depression will present with arthralgias and myalgias, but SSRI/SNRIs do not cause arthralgias.

Post delivery, which of the following contraceptive methods are possible options now for Ms. Rios? Select all that apply. A. Combination oral contraceptive B. A progestin-only oral contraceptive C. A copper-containing intrauterine device D. Injectable medroxyprogesterone acetate (Depo-Provera)

The correct answers are B, C, D. Progestin-only pills, injectable progestin (Depo-Provera), and progestin implants (Implanon) can be started immediately post partum. Advantages to these methods include the minimal effect of progestins on blood pressure, coagulation factors or lipid levels, and the lack of increased risk of stroke, myocardial infarction, or venous thromboembolism with progestin-only contraceptives. Potential disadvantages include the need to take a daily pill at the same time every day (for maximum effectiveness) with a progestin-only pill and irregular bleeding, particularly within the first few months. In women who are breastfeeding, progestins do not appear to cause changes in the composition or volume of breast milk or have any negative effect on the infant. However, due to theoretical concerns about the role of progestins in the initiation and production of breast milk, agencies such as World Health Organization (WHO) and the American College of Obstetricians and Gynecologists (ACOG) recommend that women not begin progestin-only contraception until six weeks if exclusively breastfeeding. In clinical situations where follow-up is uncertain, it may be advisable to initiate contraception before discharge from the hospital. Copper-containing IUDs are the only type of IUD approved for use postpartum and may be inserted immediately. Immediate insertion has been shown to be safe and effective, although earlier insertion has a slightly higher expulsion rate compare to insertion at 4-6 weeks postpartum. Levonorgestrel-releasing IUDs (Mirena) should not be inserted until 6 weeks post-partum (full involution of the uterus) due to a higher risk of perforation and expulsion. Use of combined oral contraceptives postpartum and in lactating women is controversial. Women may benefit from reduced coagulation-related risks (such as DVT) by waiting four or more weeks postpartum, and combined oral contraceptives are known to suppress milk production in the early postpartum period. However, clinical trials have shown mixed results with regard to the effect of combined oral contraceptives on milk supply and infant growth, and a Cochrane review concluded that no evidence-based recommendation can be made about the use of combined oral contraceptives in lactating women.

Given all of these findings, which of the following actions would be appropriate at this visit? Select all that apply. She has a rash and week 36 A. Perform urine screening for Group B streptococcus. B. Perform vaginal-rectal swab screening for Group B streptococccus. C. Prescribe a low-potency topical steroid for her rash. D. Recommend lotion or skin emollients for her rash. E. Perform serum testing for electrolytes, renal function, and glucose. F. Re-check hemoglobin/hematocrit testing.

The correct answers are B, C, D. Since Ms. Rios is at 36 weeks', it would be appropriate to assess GBS status today. In addition to addressing Ms. Rios' GBS status at today's visit, it is also appropriate to treat her rash. It would not be appropriate at this visit to perform serum testing for electrolytes, renal function, and glucose (E); or to recheck her hemoglobin/hematocrit (F) as she does not have new symptoms or risk factors for gestational diabetes, renal problems, or change in blood counts.

Dr. Hill asks you to consider a differential diagnosis. Choose five of the following that are at the top of your differential given what you know of Mr. Smith so far. A. Muscle strain B. Lymphedema C. Cellulitis D. Popliteal cyst E. Deep venous thrombosis F. Venous insufficiency G. Peripheral arterial disease

The correct answers are B, C, E, F, G. Most Likely Diagnoses Mr. Smith's concern of swelling that is unilateral is an important finding to support the diagnosis of cellulitis, lymphedema, or DVT. In contrast, for venous insufficiency or PAD, one would expect bilateral leg swelling. Cellulitis and DVT are acute processes. Lymphedema, PAD, and venous insufficiency are less likely, given the acute nature of Mr. Smith's symptoms. Since PAD, venous insufficiency, and lymphedema are not infectious processes, the lack of fever in Mr. Smith's case speaks in favor of these diagnoses. In Mr. Smith's case, the acute unilateral swelling, erythema, and warmth in a diabetic patient, make cellulitis likely. The presence of a fever would support the diagnosis of an infectious process like cellulitis. But the fact that Mr. Smith does not have a fever doesn't rule out a diagnosis of cellulitis as it is certainly possible to have localized cellulitis without fever. In Mr. Smith's case, you feel that the acute nature of his symptoms, the unilateral swelling, and the presence of risk factors such as obesity, smoking, and diabetes - make DVT a very likely diagnosis. Mr. Smith does not have a fever, but the inflammatory process due to a thrombus in his vein could explain his unilateral lower extremity edema. The acute and unilateral nature of Mr. Smith's symptoms make you think a diagnosis of venous insufficiency is unlikely. In Mr. Smith's case, a diagnosis of PAD seems unlikely given the lack of claudication and normal pulses on examination. Although PAD may not be the primary cause of Mr. Smith's leg swelling and pain, it could potentially be contributing to it by increasing his risk for a foot ulcer. Mr. Smith's history of smoking and diabetes increase the likelihood that he has PAD. Less Likely Diagnoses Muscle strain (A) - Although this is a frequent cause of unilateral leg swelling and pain, in a patient with intact mental status, a history of trauma should be present to consider this. Popliteal cyst (D) - This should be palpable and rarely would cause the extensive swelling and pain seen in Mr. Smith.

Which of the following statements are true about thiazide diuretics? Select all that apply. A. A 50 mg dose of hydrochlorothiazide reduces blood pressure and decreases morbidity and mortality more than a 25 mg dose of hydrochlorothiazide. B. Thiazides may affect electrolyte levels. C. Thiazide diuretics should be used with caution in patients with a history of gout. D. Thiazide diuretics should be started at 25mg in older adult patients. E. Thiazide diuretics may cause older adult patients to become incontinent of urine.

The correct answers are B, C, E. Thiazides may affect electrolyte levels and cause hyponatremia or hypokalemia, and thus require periodic monitoring of blood electrolytes. Thiazide diuretics should be used with caution in patients with a history of gout as they may precipitate flares. Patients on uric acid lowering therapy are less likely to experience a flare. Thiazide diuretics may cause older adult patients to become incontinent of urine. Aging decreases bladder capacity, decreases time of awareness of need to void, increased detrusor instability decreased pelvic floor muscles, incomplete emptying, and atrophic urethral changes. Thiazide diuretics may exacerbate problems for those on the threshold or already incontinent of urine. Thiazides have demonstrated marked reduction in morbidity and mortality from hypertension in comparison to newer, more expensive antihypertensive medications. Studies have also demonstrated that despite the availability of 50 mg hydrochlorothiazide tablets, doses of hydrochlorothiazide above 25 mg do not decrease blood pressure further or further reduce morbidity and mortality rates. A 2004 JAMA meta-analysis of clinical studies indicated that low-dose (12.5 to 25 mg/d chlorthalidone or hydrochlorothiazide) and high-dose (50 mg/d or more of both drugs) diuretic therapy lowered blood pressure to a similar degree and exerted a similar benefit in reducing stroke, congestive heart failure, cardiovascular and total mortality, but only low-dose diuretic therapy significantly reduced coronary heart disease incidence. A 2009 Cochrane review reinforced that no other drug class improved health outcomes better than low-dose thiazides, and beta-blockers and high-dose thiazides were found to be inferior. Thiazide diuretics should be started at lower doses such as 6.25mg or 12.5mg/d in older adult patients, because this population may be more sensitive to this drug class and may experience hypotensive episodes or electrolyte abnormalities. Most other adults can start at 25mg/d.

Stressed woman with abdominal pain. In addition to the five diagnoses mentioned above, which three of the following conditions are at the top your differential diagnosis at this time? A. Gallbladder disease B. Gastritis C. Irritable bowel syndrome D. Inflammatory bowel disease E. Peptic ulcer disease F. Urinary tract infection G. Vaginitis

The correct answers are B, C, G.

You know that survivors of a transient ischemic attack (TIA) or stroke have an increased risk of another stroke, and much of the long-term treatment is aimed at secondary prevention of another stroke. You have already prescribed antithrombotic therapy. What else do you want to consider in your management and follow-up plan for Mr. Wright? Select all that apply. A. Moderate-intensity statin therapy B. Blood pressure lowering to a goal of 130/80 mmHg C. Blood pressure lowering to a goal of 150/90 mmHg D. Smoking cessation E. Mediterranean diet F. Exercise counseling G. Stroke education

The correct answers are B, D, E, F, G. Need high-intensity statin

Which of the following interventions have evidence of improving cardiovascular disease outcomes that matter to patients with diabetes? Select all that apply. A. Advising all patients to simply cut back on their smoking B. Lowering blood pressure in patients with blood pressure > 140/90 mmHg C. Screening asymptomatic diabetes patients for coronary heart disease (CHD) D. Adding a moderate-intensity statin for patients 40-75 yrs old with LDL-c >70 mg/dl E. Adding a high-intensity statin for patients 40-75 yrs old with LDL-c >70 mg/dL and ≥ 7.5% estimated 10-year ASCVD risk F. Treating dyslipidemia through lifestyle modification: diet and exercise G. Using aspirin as secondary prevention in diabetes patients with a history of CVD H. Controlling glucose as close to normal range as possible (A1C 4 to 6%)

The correct answers are B, D, E, F, G. For detailed information about cardiovascular risk management in diabetes, see Teaching Point below.

Dr. Medel asks you, "What treatment approaches would you recommend for a patient like Mr. Martin with newly diagnosed Stage 2 hypertension?" You respond that all patients with newly diagnosed hypertension (or elevated blood pressure) should be counseled about behavioral approaches that they can take to manage their blood pressure. Specifically, the new ACC/AHA guidelines recommend weight loss for those who are overweight or obese, a heart healthy diet like the DASH eating plan, sodium restriction, potassium supplementation through dietary measures, increased physical activity, and a limited alcohol intake. Dr. Medel answers, "Great summary! Would you recommend a medication?" Select all that apply. A. No antihypertensive medication indicated at this time B. Initiate a thiazide diuretic C. Initiate a beta-blocker D. Initiate a calcium-channel blocker E. Initiate an ACE-Inhibitor

The correct answers are B, D, E. For patients like Mr. Martin with Stage 2 hypertension, regardless of ASCVD risk, the ACC/AHA guidelines recommend that a medication be started at the time of diagnosis, along with counseling about lifestyle modifications. If the blood pressure is more than 20/10 mmHg above goal, two medications should be started. The meta-analysis conducted for these guidelines found a slight preference for thiazides (particularly chlorthalidone) as the first-choice medication. See the required Aquifer Hypertension Guidelines Module to learn more about lifestyle modifications and pharmaceutical management of hypertension.

Which hospital interventions can prevent or minimize delirium? Select all that apply. The best options are indicated below. Your selections are indicated by the shaded boxes. A. Place a Foley catheter to prevent the urge to urinate and incontinence B. One-to-one sitter C. Place patient in room at the end of the hall where it is dark and quiet at night D. Repeated reorientation E. Remove eyeglasses, so the patient is not likely to pick at and damage them F. Avoid sedative medications and medications with anticholinergic effects G. Range of motion exercises

The correct answers are B, D, F, G.

Dr. Lee tells you: "Working from such a broad list is difficult. Having a shorter list of working diagnoses will help you conduct a more focused initial history and physical exam. What are the three most common causes of back pain?" Select all that apply. A. Pyelonephritis B. Lumbar strain C. Spinal stenosis D. Disc herniation E. Kidney stones F. Degenerative joint disease

The correct answers are B, D, F.

"Which of the following are the best next steps in management?" Select all that apply. A. Check alpha-1 antitrypsin levels. B. Prescribe an albuterol metered-dose inhaler on an as-needed basis. C. Start prednisone at a low dose. D. Help the patient to quit or decrease smoking. E. Admit the patient to the hospital for intensive training and support.

The correct answers are B, D.

According to USPSTF, of the groups listed below, who should be screened for chlamydia? Select all that apply. A. All non-pregnant women B. All sexually active women under 25 years C. All pregnant women D. All sexually active women 25 years or older who are at high risk E. All sexually active men

The correct answers are B, D.

While Mr. Chen's pain does not sound emergent, you are ever mindful of conditions within your differential that would require immediate treatment to avoid catastrophic outcomes. Which of the following musculoskeletal causes of shoulder pain would merit urgent diagnosis and management? Select all that apply. The best options are indicated below. Your selections are indicated by the shaded boxes. A. Calcific tendinopathy B. Septic glenohumeral arthritis C. Impingement of the supraspinatus tendon D. Septic subacromial bursitis E. Adhesive capsulitis

The correct answers are B, D. Delay in recognition and treatment of a septic glenohumeral ("shoulder joint") arthritis or septic subacromial bursitis may lead to local tissue destruction and loss of function, extension of infection locally to deeper spaces such as bone (osteomyelitis) or more distant sites by way of bacteremia and that may progress to sepsis.

How would you interpret Savannah's vital signs and prenatal labs? Select all that apply. Rh negative, hr 85 A. The normal blood pressure suggests it is unlikely that she is bleeding now. B. The pulse is within normal limits, suggesting hemodynamic stability. C. The low-grade fever suggests infection. D. Rhesus immune globulin (RhoGam) is indicated.

The correct answers are B, D. The correct answers are that the pulse is within normal limits, suggesting hemodynamic stability (B) and that RhoGam is indicated (D). Dr. Hill continues: "Savannah's pulse and blood pressure indicate that she is hemodynamically stable. If she was hemodynamically unstable, it would change how urgently you need to begin to make your assessment and, in some cases, intervene. Similarly, the concern of significant pelvic abdominal pain associated with vaginal bleeding would increase the urgency and necessity of assessment. Additionally, her vital signs indicate that she is afebrile. This does not conclusively rule out infection, but it does make significant infection and/or sepsis less likely. "Finally, although this is not the most urgent issue, Savannah's blood type is important. She is Rh-negative and she should be administered a 50mcg dose of Rho(D) Immune Globulin (e.g., RhoGAM), regardless of the eventual diagnosis. RhoGAM is indicated for bleeding episodes during pregnancy, regardless of the gestational age."

Which treatments are recommended in the elderly? Select all that apply. A. Benzodiazepines B. Cognitive behavioral therapy C. Antihistamines D. Antidepressants E. Zolpidem

The correct answers are B, E.

You recognize that Sarah is asking for a specific diagnostic test and wants it to be done urgently. Based on her history, you don't believe that Sarah will need an MRI urgently. Which of the following are the next best steps? Select all that apply. A. Tell her up front that she doesn't need an MRI at this time. B. Identify and address her primary concerns. C. Try not to bring up brain tumors. D. Tell her you'll make sure she gets an MRI. E. Expand your history and perform your physical examination.

The correct answers are B, E.

Which of the following are approved initial treatment regimens for the eradication of H. pylori? Select all that apply. A. PPI therapy for an additional 4 weeks for 8 weeks of total therapy B. Addition of an H2 receptor antagonist to the PPI regimen for 8 weeks of total therapy C. Standard dose PPI + amoxicillin 1 gram + clarithromycin 500mg all twice daily for 10-14 days D. Standard dose PPI + trimethoprim/sulfamethoxazole double strength + erythromycin 500mg all twice daily for 10-14 days E. Standard dose PPI + amoxicillin 1 gram + levofloxacin 500mg all twice daily for 10 days F. Standard dose PPI once or twice daily + metronidazole 250 mg, tetracycline 500 mg, + bismuth subsalicylate 525 mg each four times daily for 10-14 days

The correct answers are C, F.

Which of the following statements about chronic pain treatment is/are correct? Select all that apply. A. The ultimate goal of long-term pain control should be the complete elimination of pain. B. The most common side effect of long-acting opioids is constipation. C. Depression and chronic pain are unrelated and antidepressants have no role in the management of chronic pain. D. Anticonvulsants have a very low side effect profile, and are effective in the treatment of neuropathic pain. E. Tricyclic antidepressants (TCAs) are associated with anticholinergic side effects, and are relatively contraindicated in patients with severe cardiovascular disease or conduction problems. SUBMIT

The correct answers are B, E. Some anticonvulsants require blood level monitoring and have severe side effects like megaloblastic anemia. Carbamazepine (Tegretol) can interfere with other medications because it is a cytochrome P-450 inducer, including decreasing the effectiveness of hormonal contraception. Several anticonvulsants are also known teratogens.

Which of the following patients with otherwise uncomplicated respiratory tract infections require treatment with antibiotics? Select all that apply. A. 18-year-old with 5 day history of acute maxillary sinusitis B. 2-month-old with acute otitis media C. 28-year-old with adenovirus D. 3-year-old with a fever and a two-day history of otitis media E. 58-year-old with bronchitis F. 32-year-old with community acquired pneumonia G. 40-year-old with streptococcal pharyngitis

The correct answers are B, F, and G. For more teaching about antibiotic use in upper respiratory infections, see the Teaching Point below.

Which of the following are major risk factors for atherosclerotic cardiovascular disease (ASCVD)? Select all that apply. The best options are indicated below. Your selections are indicated by the shaded boxes. A. Changes in vision B. Diabetes C. Elevated HDL cholesterol levels D. Father had a myocardial infarction at age 72 E. Female sex F. History of dizziness G. Hypertension H. Older age (men over 45, women over 55) I. Tobacco use

The correct answers are B, G, H.

Which of these patients might describe their dizziness as a feeling of lightheadedness or like they are going to faint? Select all that apply. The best options are indicated below. Your selections are indicated by the shaded boxes. A. 48-year-old alcoholic with cerebellar degeneration B. 36-year-old with aminoglycoside toxicity C. 58-year-old with a myocardial infarction D. 28-year-old with anemia secondary to acute bleeding from a gastric ulcer E. 68-year-old with atrial fibrillation and uncontrolled heart rate F. 54-year-old with peripheral neuropathy G. 26-year-old with thyroid storm H. 72-year-old with acute gastroenteritis SUBMIT

The correct answers are C, D, E, G and H. Dizziness that is described as "lightheaded" or "like I'm going to faint" is generally classified as presyncope and is usually caused by inadequate cerebral perfusion (C, D, E, G and H). Treatment, therefore, is dependent on identifying and correcting the specific underlying cause.

Of the following, which are considered Instrumental Activities of Daily Living (IADL)? Select all that apply. The best options are indicated below. Your selections are indicated by the shaded boxes. A. Transferring from bed to chair (or vice versa) B. Maintenance of continence C. Doing laundry D. Using a telephone E. Managing finances F. Bathing G. Preparing meals H. Dressing I. Managing medications

The correct answers are C, D, E, G. Functional status assessment is traditionally divided into evaluation of Activities of Daily Living (ADL, or skills required for basic living) and Instrumental Activities of Daily Living (IADL, or skills required for living independently).

What are the best medications to offer the patient at this point? Select all that apply. A. Cough suppressant such as dextromethorphan B. Cough suppressant such as codeine C. Oral antihistamine such as cetirizine D. Inhaled bronchodilator such as albuterol E. Nasal corticosteroid such as fluticasone F. Inhaled corticosteroid such as budesonide

The correct answers are C, D, E. Cough suppressants such as dextromethorphan (A), codeine (B), and benzonatate are treatments that can provide minimal relief in the setting of acute cough, such as from the common cold or acute bronchitis. They are not appropriate for chronic sources of cough such as Mr. Dennison is experiencing. An inhaled corticosteroid (F) is an appropriate treatment for diagnosed asthma. Asthma is high on the differential for Mr. Dennison, but it would be more appropriate to start with simply an albuterol inhaler and allergy control as his symptoms are relatively mild and his diagnosis has not been confirmed. For cough due to suspected asthma Before a diagnosis of asthma is confirmed, it is reasonable to offer patients a short-acting inhaled bronchodilator such as albuterol. While a Cochrane review (2015) did not find evidence to support the use of beta2-agonists routinely in patients with a cough, it did acknowledge that "quicker resolution of cough with beta2-agonists were those with a higher proportion of people wheezing at baseline."

Mr. Dennison is studying the "My Asthma Plan" and he asks you how he would feel when his peak flow reading is in the Red Zone. Which of the following are examples of symptoms and signs indicating he is in the Red Zone? Select all that apply. The best options are indicated below. Your selections are indicated by the shaded boxes. A. Can do some activities, but not all B. Quick-relief medication such as a short-acting beta2 agonist inhaler help temporarily C. Very short of breath, including when walking or talking D. Gray or blue lips or fingernails E. A peak flow reading that is < 50% of his personal best reading

The correct answers are C, D, E. Red Zone Signs and Symptoms As described in "My Asthma Plan", a patient in the red zone would be very short of breath, including when walking or talking (C) and signs of cyanosis (D) could be present and indicate severe respiratory distress. In addition, the peak flow reading would be < 50% of the personal best (E). A patient in the red zone would also be unable to perform usual activities and would not improve with quick-acting medications such as short-acting beta2-agonists. Therefore, answers A and B are not correct.

Which of the following are risk factors for osteoporosis that Mrs. Parker has? Select all that apply. A. Corticosteroid use B. Obesity C. Hormone therapy D. Family history E. Caucasian race

The correct answers are D, E. Risks: Corticosteroid use Family history of osteoporosis, especially if a first-degree relative has fractured a hip. Previous fragility fracture defined as a low-impact fracture Smoking Heavy alcohol use Lower body weight (weight < 70 kg) is the single best predictor of low bone mineral density. Obesity (B) does not put patients at risk for osteoporosis, but neither is obesity protective against osteoporosis. Caucasian race - At any given age, African-American women on average have higher bone mineral density (BMD) than white women. The USPSTF, while acknowledging that the data for non-white women is less compelling than for whites, recommends screening all women at age 65 or earlier if they have equivalent risk.

Among following conditions, which need to be treated with systemic antifungal agents? Select all that apply. A. Tinea pedis/tinea manuum B. Tinea corporis/tinea cruris C. Tinea capitis D. Tinea unguium (onychomycosis)

The correct answers are C, D. In this case, if Mr. Fitzgerald had a fungal infection, you would treat it with an antifungal cream. Systemic Therapy Tinea capitis Oral therapy is required to adequately treat tinea capitis, as they are able to penetrate the infected hair shaft where topical therapies cannot. Griseofulvin is the first-line oral antifungal treatment approved for use. Suggested dosing is 20-25 mg/kg/day using the microsize formulation, for 6-12 weeks. Where the ultramicrosize formulation is used, a dose of 10-15 mg/kg/day is suggested, as it is more rapidly absorbed than the microsize form. Terbinafine hydrochloide was also approved by FDA in 2007 for tinea capitis for children ages 4 years and older. The approved pediatric dose of terbinafine granule is 125 mg, 187.5 mg, or 250 mg for children weighing less than 25 kg, 25 to 35 kg, and more than 35 kg, respectively, once daily for 6 weeks. In multiple studies, terbinafine was consistently more effective than griseofulvin against tinea capitis caused by Trichophyton tonsurans. However, in children with microsporum infection, new evidence suggests that the effect of griseofulvin is better than that of terbinafine. Tinea unguium Though griseofulvin is approved for tinea infection of the nails, its affinity for keratin is low and long-term therapy is required. The oral therapy regimens for tinea unguium (onychomycosis)are as follows: terbinafine 250 mg/day for 12 weeks (toenails) or 6 weeks (fingernails only) itraconazole 200 mg twice daily as pulse therapy

Mr. Rodriguez still has PUD despite triple therapy eradication. Which of the following options are appropriate next steps in evaluation and treatment? Select all that apply. A. Prescribe a PPI once daily for 4 weeks, then re-evaluate B. Refer for an upper endoscopy/EGD and biopsy C. Obtain an H. pylori fecal antigen test D. Obtain a urea breath test E. Repeat a course of PPI triple therapy for 14 days F. Treat with salvage therapy for resistant H. pylori infection

The correct answers are C, D. It has now been over four weeks since Mr. Rodriguez was treated with initial pharmacotherapy for H. pylori gastritis.

Several tests are recommended to be performed routinely in patients with suspected ischemic stroke to identify systemic conditions that may mimic or cause stroke or that may influence therapeutic options. If you were to imagine that Mr. Wright were presenting with symptoms of an acute stroke, which studies would you order now for Mr. Wright? Select all that apply. The best options are indicated below. Your selections are indicated by the shaded boxes. A. Toxicology screen B. Blood alcohol level C. Noncontrast brain CT or brain MRI D. Blood glucose E. Serum electrolytes/renal function tests F. ECG G. Markers of cardiac ischemia H. Complete blood count, including platelet count I. Prothrombin time/international normalized ratio (INR) J. Activated partial thromboplastin time K. Oxygen saturation L. Chest radiography M. Lumbar puncture

The correct answers are C, E, F, G, H, I, J, K, L.

Which of the following actions would you suggest for Ms. Roman based on the USPSTF Preventive Services Database search? Select all that apply. A. Ultrasound to screen for abdominal aortic aneurysm (AAA) B. Counseling on low back pain prevention techniques C. Colorectal cancer screening D. Ultrasound to screen for carotid artery stenosis E. Exercise to prevent falls F. Thyroid screening with TSH G. Screening for depression H. Screening for hypertension I. Mammogram screening for breast cancer J. Screening for cervical cancer

The correct answers are C, E, G, H, and I. Colorectal cancer screening by various methods, including fecal occult blood test (FOBT), flexible sigmoidoscopy, colonoscopy, and FIT testing, is recommended for patients ages 50 to 75 years old. The USPSTF recommends screening adults for depression in clinical practices that have systems in place to assure accurate diagnosis, effective treatment, and followup. Patients 18 years old and older should be screened for elevated blood pressure. The USPSTF recommends biennial screening mammography for women ages 50 to 74. They state that evidence is insufficient to make any recommendation for screening over the age of 75. The USPSTF recommends physicians encourage exercise for community-dwelling patients over 65 in order to prevent falls.

What is the first step in managing Mr. Fitzgerald's prostate problem? (BPH) Select all that apply. A. Start with 5-alpha-reductase inhibitor. B. Start with combination treatment of an alpha-adrenergic antagonist and a 5-alpha-reductase inhibitor. C. Behavior modifications to decrease symptoms. D. Refer the patient to urology service for surgical intervention like TURP (transurethral resection of the prostate). E. Start with alpha-adrenergic antagonists.

The correct answers are C, E.

If Mr. Chen had restricted passive as well as active ROM of the shoulder, what type of problems involving the shoulder might you consider? Select all that apply. The best options are indicated below. Your selections are indicated by the shaded boxes. A. Rotator cuff tear B. Rotator cuff impingment C. Adhesive capsulitis D. Tendinopathy of the long head of the biceps E. Glenohumeral arthritis

The correct answers are C, E. A patient with loss of active and passive ROM is more likely to have joint disease; whereas a patient with loss of only active ROM is more likely to have an issue with muscle tissue. The following joint diseases will produce restricted active and passive ROM of the shoulder: Adhesive capsulitis, a condition common in patients with metabolic diseases such as diabetes and hypothyroidism in which there is contracture of the joint capsule Glenohumeral arthritis, a much less common site of osteoarthritis than the primary weight-bearing joints of the lower extremity The following muscle tissue issues will compromise only active ROM (although may still elicit passive ROM pain in the case of rotator cuff impingement): Rotator cuff tear and impingement (conditions that may well occur together)

In man with 1 year of abdominal pain who uses NSAIDS, From the following, select the top three diagnoses on your differential. A. Anxiety B. Pneumonia C. Gastroesophageal reflux disease (GERD) D. Abdominal wall muscle strain E. Acute Pancreatitis F. Peptic ulcer disease (PUD) G. Diverticulitis H. Angina pectoris I. Gastritis SUBMIT Answer Comment The correct answers are C, F, I.

The correct answers are C, F, I.

What are the most likely causes of Mr. Cunha's iron deficiency? Choose the two best answers. A. Hematuria B. Iron malabsorption C. Colorectal carcinoma D. Peptic ulcer disease E. Gastritis F. Adenomatous colonic polyp G. Inadequate dietary intake of iron

The correct answers are C, F.

Mr. Cunha says, "Do you think I have cancer, doc? I want you to tell me." From the list of options, select how you would respond to Mr. Cunha. Select all that apply. A. "Anything is possible, but most often rectal bleeding is not caused by cancer, so you probably don't have it." B. "I couldn't possibly tell you until you have the colonoscopy. That will give us the answer." C. "In truth I don't know, but it is a possibility. I certainly hope you don't have cancer, but if it turns out that you do, I want you to know that I will be there to help you." D. "If you had had a normal colonoscopy three years ago, I would feel comfortable saying that you probably don't have cancer now, but now I can't be so sure."

There is no single correct answer. All options are possibilities

Which response(s) do you feel would be the most helpful given Mr. Cunha's statements regarding screening? Select all that apply. A. "I understand what you are saying, but these tests are still very important for you." B. "I hear your concerns. I still recommend these tests for you at some point, and we can talk about them more whenever you'd like. May I ask you about them again sometime?" C. "I understand. Many patients are scared of screening tests." D. "I worry that you will regret this some day. The United States Preventive Services Task Force and the American Academy of Family Physicians both recommend all these tests."

There is no single correct answer. All options are possibilities.

From the options below, select the statement you feel would be the most helpful to Mr. Cunha, given what you know about his diagnosis. Select all that apply. The best options are indicated below. Your selections are indicated by the shaded boxes. A. "I really have no way of knowing at this point. We will just have to wait until more tests come back." B. "Unfortunately I can't answer that until I have more information. Fortunately, however, treatment for colon cancer has really advanced, and survival from early stage cancers is excellent." C. "I certainly can understand your desire to know, and I wish I could give you a better sense. Unfortunately, until I have more information I can't give you a good answer. I promise to give you one as soon as I can." D. "No one ever really knows how long someone has to live, so that is not a question I can answer." SUBMIT

There is no single correct answer. All options are possibilities. Answer (A) tells the truth to Mr. Cunha, but does so somewhat bluntly. Answer (B) also tells the truth, and goes on to try to give Mr. Cunha some hope. Often, physicians appropriately feel a need to give their patients hope, but sometimes their efforts leave patients with an unrealistic sense of optimism. In this case, Dr. Medel does not know if Mr. Cunha has an "early stage" cancer. Answer 'B' could easily lead Mr. Cunha to infer that he does and to think that his prospects are "excellent." Answer (C) expresses empathy about Mr. Cunha's desire to know what his prognosis is. It answers his question truthfully and extends the therapeutic relationship by promising to always tell him the truth. Answer (D) is a common response to such questions. While it is technically true that we usually don't know exactly how long a patient will live, this response fails to answer the essential gist of the question: "What is my prognosis?" It relies on a technicality to avoid discussing an uncomfortable and important topic.

Based on your findings from her history and physical exam, select from the following the two top diagnoses on your differential. Choose the two best answers. Girl with sore throat, fever A. Epiglottitis B. Viral pharyngitis C. Pertussis D. Peritonsillar abscess E. Group A Beta-hemolytic Streptococcus (GABHS) pharyngitis F. Retropharyngeal abscess G. Viral croup H. Allergic rhinitis/pharyngitis SUBMIT

This person's symptoms are most consistent with strep throat (E) or viral pharyngitis (B). Epiglottis (A) is less likely given lack of drooling, inspiratory stridor, or "hot potato" voice. Pertussis (C) is less likely given the presence of the rash and is more typically considered for persistent cough after acute illness. Peritonsillar abscess (D) is less likely in this patient due to her lack of drooling, her symmetric tonsils, and her midline uvula. Retropharyngeal abscess (F) is less likely given absence of dysphagia. Viral croup (G) is less likely given character of cough and lack of stridor or hoarse voice. Allergic rhinitis/pharyngitis (H) is less likely given her fever and acute presentation. See below for more teaching about pediatric pharyngeal infections.

When is breast US typically used

This tool is generally used for evaluation of suspected abnormalities.

What is USPSTF evidence for breast ultrasound

USPSTF guidelines says there is insufficient evidence to use this for screening in women with dense breasts. The American College of Radiology (ACR) notes that it increases breast cancer detection in women with dense breasts but also increases the risk of false-positives

Dr. Lee asks you what cancers (besides breast and cervical) Mrs. Payne should be screened for. Choose the single best answer. A. Malignant melanoma B. Ovarian cancer C. Lung cancer D. All of the above E. None of the above

none of the above

What are skin cancer screening recs

While skin cancer is the most common type of cancer, the USPSTF currently reports that there is insufficient evidence to recommend for or against skin cancer screening.

When stop cervical cancer screening

Women older than 65 years who have had adequate screening within the last ten years may choose to stop cervical cancer screening. Adequate screening is three consecutive normal pap tests with cytology alone or two normal pap tests if combined with HPV testing.

Based on her history, which of these can you use with Sarah? Select all that apply. migraines not controlled by NSAIDs A. Ergot alkaloids B. Triptans C. Acetaminophen/butalbital/caffeine D. Aspirin/butalbital/caffeine

You may use ergot alkaloids (A) or triptans (B). While Sarah has an IUD and is very unlikely to become pregnant, you will need to warn her about their use if she should choose to remove the IUD or becomes pregnant. Since you are concerned that Sarah's recent overuse of caffeine may be contributing to her worsening symptoms, it would not be advisable to prescribe a caffeine-containing medication, such as in choice (C) acetaminophen/butalbital/caffeine, or (D) aspirin/butalbital/caffeine. You would also want to explore with Sarah her recent use of hydrocodone, an opioid narcotic medication, before prescribing a combination product containing butalbital (C) and (D), a barbiturate, as both may be habit-forming.

You are seeing a 28-year-old female who presents with gradually worsening headaches. She has had headaches off and on since she was a teen, similar to her mother. She typically awakens with them, and they are associated with nausea and vomiting which can last for much of the day. The pain is throbbing and usually unilateral behind the right eye. Resting in a dark room often helps, and ibuprofen can help as well if she is able to take it at the onset of the headache. Recently, the headaches have become more common, happening two to three times per week. These have been interrupting her job as a paralegal, and she is worried she will be fired. She has tried yoga and mindfulness-based stress reduction, but these have not helped. She does not smoke, drink alcohol, or use illicit drugs. She recently stopped her oral contraceptive pills in an effort to get pregnant. Her vital signs and physical exam are perfectly normal. Of the following, which would be the most appropriate next step in managing this patient? A. Advise her to simply continue the ibuprofen as it has been helpful B. Discontinue the ibuprofen and prescribe acetaminophen instead C. Order a CT scan of her head D. Prescribe amitriptyline nightly E. Prescribe sumatriptan as needed SUBMIT

`The correct answer is D. This patient is experiencing migraine headaches that are frequent enough that she is at risk of developing rebound headaches from frequent headache medication use. Given that the headaches are also interrupting her work and have not responded to behavioral strategies, it is appropriate to prescribe migraine prophylaxis, such as amitripyline (D). Simply continuing the ibuprofen (A), as she has been taking, is not likely to help more than it has and may cause rebound headaches. For similar reasons, changing her headache treatment from ibuprofen to acetaminophen is unlikely to help (B). This patient has no red-flag characteristics to warrant brain imaging (C). Sumatriptan (E) is an appropriate treatment for migraines, and may work better than ibuprofen for some patients. However, it is contraindicated in pregnancy, which this patient seeking to become. Like with ibuprofen, there would be a concern for rebound headaches, given the frequency of her migraines currently.

A 64-year-old woman who is overweight with well-controlled hypertension comes to your office complaining of a lump in her breast that she noticed while showering. She denies any pain, tenderness, or skin changes. A pertinent review of systems is negative. Menarche began at the age of 10. Her first child was born when she was 31 and she had her second and last child at the age of 33. She experienced menopause at the age of 44. Her mother died of colon cancer when she was 65 and her father passed away from metastatic prostate cancer at the age of 70. She has no history of tobacco use ever and occasionally drinks a glass of wine with dinner. Her BMI is 34. Which of the information provided thus far puts the patient at decreased risk for breast cancer? A. Age B. Weight C. Age at first birth D. Age at menarche E. Age at menopause

age at menopause

What are the signs/symptoms that are included in the grading of ankle sprains? Select all that apply. A. Ligament tear B. Loss of functional ability C. Pain D. Swelling E. Ecchymosis

all

Which of the following are common symptoms in patients with hypothyroidism? Select all that apply. A. Weight gain B. Cold intolerance C. Pedal edema D. Heavy periods E. Fatigue F. Double vision

all except F All of these except double vision (F) are common in patients with a low thyroid level. Double vision is a possible symptom of Graves ophthalmopathy but does not suggest a particular thyroid level.

Besides amenorrhea, which of the following are signs and symptoms of pregnancy? Select all that apply. A. Nausea B. Vomiting C. Breast tenderness D. Urinary frequency E. Fatigue F. Softening of the cervix and uterus G. Cervix and vaginal walls have an increasingly purplish-blue hue H. Enlargement of the uterus I. Fetal heart tones J. Fetal movement

all of the above

What are the risk factors for developing breast cancer in the general population? Select all that apply. A. Family history of breast cancer in first-degree relative B. Prolonged exposure to estrogen C. Genetic factors D. Age E. Post-menopausal obesity F. Excessive alcohol intake

all of the above

Which of the following are appropriate next steps to offer Mr. Rodriguez at this time? Select all that apply. The best options are indicated below. Your selections are indicated by the shaded boxes. A. A follow-up appointment if his symptoms of dyspepsia recur B. A follow-up appointment for a health maintenance examination C. A visit with social worker to help him obtain medical insurance D. Positive reinforcement regarding smoking cessation E. Advisement to continue to minimize alcohol consumption to a maximum of two alcoholic beverages per day

all of the above

Which of the following can cause an enlarged thyroid, also known as goiter? Select all that apply. A. Lack of iodine B. Hypothyroidism C. Hyperthyroidism D. Nodules E. Cancer F. Pregnancy G. Thyroiditis

all of the above

Which risk groups have more cervical screening

women with compromised immunity, are HIV positive, have a history of cervical intraepithelial neoplasia grade 2, 3 or cancer, or have been exposed to diethylstilbestrol (DES) in utero

What TCA agent is preferred for sleeping in elderly

doxepin 3-6 mg

Which of the following are risk factors for osteoporosis? Select all that apply. A. Early menopause B. BMI > 30 C. Sedentary lifestyle D. African American people E. History of previous fracture as an adult

early menopause, sedentary lifestyle, hx of previous fracture

What does presence of a single, hard, immobile lesion of approximately 2 cm or larger with irregular borders indicate

increases the likelihood of malignancy.

Women with ASC-US and negative HPV testing - should repeat when?

repeat co-testing in 3 years

How effective are oral medications, such as bupropion (Wellbutrin, Zyban, Budeprion) or varenicline (Chantix), in helping smokers quit? Choose the single best answer. A. Not effective (Quit rate at 12 months is no higher than placebo quit rate.) B. Somewhat effective (Quit rate at 12 months is 1.5 to 3 times the placebo quit rate.) C. Moderately effective (Quit rate at 12 months is 3 to 5 times the placebo quit rate.) D. Very effective (Quit rate at 12 months is 5 to 10 times the placebo quit rate.)

somewhat effective

What orexin antagonist agent is preferred for sleeping in elderly

suvorexant (Belsomra)

A 34-year-old woman who has no past medical problems nor is currently taking any medications comes into your office because she noticed a tender lump in her left breast starting approximately one month ago. She is worried because she has an aunt who had breast cancer that was BRCA positive, though her mother is BRCA negative. Her periods have been regular since they started at the age of 13 and occur every 32 days. She is currently menstruating. She has three children aged 12, 9, and 4. On exam, her BMI is 32, up from 28 three years ago and her other vital signs are stable. On breast exam, you note a mobile rubbery mass of approximately 1 x 1cm and with regular borders that is tender to palpation. You appreciate no axillary adenopathy. The rest of her physical exam is unremarkable. Of the information provided, which of the following places this patient at increased risk for breast cancer? A. Age B. Weight C. Parity history D. Family history of cancer E. Age of menarche

weight


संबंधित स्टडी सेट्स

ACC 450 - Chapter 10, ACC 450 Ch 10 Part 1, ACC 450 Ch 10 Part 4, ACC 450 Ch 10 Full Set

View Set

Ch 6 Legal Office Procedures: Legal Fees

View Set

Human Resources: Chapter 10 Quiz

View Set

earth, environment, society exam 2

View Set

Foundations of Software Engineering

View Set

General Concepts & Life Insurance basics

View Set

palabras femeninas terminadas en o,r,n

View Set

18.1 The Indian Subcontinent Achieves Freedom

View Set